You are on page 1of 116

Not included q/#/ 74, 188, 294, 310, 372, 392, 406, 414, 490.

492, 494, 523, 533

105. Examination2006
QUESTION N 1. Which one of the following is a purely ketogenic essential amino acid?
Question type: 1. Choosing the only correct answer
Question value (points): 1
Answer variants:
1. Leucine
2. Cysteine
3. Tyrosine
4. Histidine
5. Proline
Correct answer: 1 Answer variants: 5
QUESTION N 2. The rate-limiting step in catecholamine biosynthesis is:
Question type: 1. Choosing the only correct answer
Question value (points): 1
Answer variants:
1. the hydroxylation of tyrosine
2. the hydroxylation of phenylalanine
3. the formation of dopamine
4. the reduction of dihydrobiopterin
5. the hydroxylation of dopamine
Correct answer: 1 Answer variants: 5
QUESTION N 3. The hereditary X-linked recessive condition known as Lesch-Nyhan syndrome is due to a
severe, or complete, deficiency of
Question type: 1. Choosing the only correct answer
Question value (points): 1
Answer variants:
1. 5'-phosphoribosyl-1 -pyrophosphate (PRPP) synthetase activity
2. ribonucleotide reductase activity
3. adenine phosphoribosyltransferase activity
4. hypoxanthine-guanine phosphoribosyltransferase (HGPRT) activity
5. purine nucleoside phosphorylase activity
Correct answer: 4 Answer variants: 5
QUESTION N 4. Bile pigments are (2):
Question type: 2. Choosing several possible correct answers
Question value (points): 2
Answer variants:
1. Bilirubin
2. Biliverbin
3. Cholic acid
4. Deoxy cholic acid
5. All of the above
Correct answers: 1; 2 Answer variants: 5
QUESTION N 5. In hemolytic jaundice, the direct fraction of bilirubin :
Question type: 1. Choosing the only correct answer
Question value (points): 1
Answer variants:
1. Increases
2. Decreases
3. Remains the same
4. None of the above
Correct answer: 1 Answer variants: 4
QUESTION N 6. Serum transaiminases in hepatocellular jaundice :
Question type: 1. Choosing the only correct answer
Question value (points): 1
Answer variants:
1. Remains Normal
2. Very high in first week
3. Moderately raised
4. None of the above
Correct answer: 2 Answer variants: 4
QUESTION N 7. Match the following symptoms of disorders of amino acid catabolism with the correct
treatment.
Question type: 4. Arranging the correspondences for the answers
Question value (points): 4
Answer variants:
1. A diet low in phenylalanine
2. A leucine-restricted diet and administration of glycine
3. Restricted intake of branched-chain amino acids
4. A low-protein diet supplemented with arginine
Answer variants: 4
Correspondence variants:
A. High serum levels of ammonia
B. High serum levels of isovaleric acid
C. High serum levels of phenylpyruvate
D. High serum and urine levels of valine, leucine, and isoleucine and their corresponding alfa-keto acids
Correct answers: 1-C; 2-B; 4-A; 3-D Correspondence variants: 4
QUESTION N 8. Indicate true or false of the following:
Question type: 4. Arranging the correspondences for the answers
Question value (points): 1
Answer variants:
1. Aromatic amino acids form glutathione
2. D-amino acids are absorbed by simple diffusion
3. Sometimes the whole protein is absorbed into the blood
Answer variants: 3
Correspondence variants:
A. True
B. False
Correct answers: 1-B; 2-A; 3-A Correspondence variants: 2
QUESTION N 9. Degradation of hemoglobin takes place in:
Question type: 1. Choosing the only correct answer
Question value (points): 1
Answer variants:
1. Reticuloendothelial cells
2. Erythrocytes
3. Mitochondria
4. Extramitochondria
5. All of the above
Correct answer: 1 Answer variants: 5
QUESTION N 10. In hepatocellular jaundice, the direct fraction of bilirubin :
Question type: 1. Choosing the only correct answer
Question value (points): 1
Answer variants:
1. Increases
2. Decreases
3. Remains the same
4. None of the above
Correct answer: 1 Answer variants: 4
QUESTION N 11. In hemolytic jaundice, urine bilirubin is:
Question type: 1. Choosing the only correct answer
Question value (points): 1
Answer variants:
1. Increased
2. Usually absent
3. Present in small amount
4. None of the above
Correct answer: 2 Answer variants: 4
QUESTION N 12. A patient suffers from kidney stones. The patient is diagnosed as having a genetic defect in
an amino acid transport system, which is the cause of this problem. Which one of the following amino acids
is likely to be in excess in the urine of this patient?
Question type: 1. Choosing the only correct answer
Question value (points): 1
Answer variants:
1. Lysine
2. Proline
3. Alanine
4. Tryptophan
5. Glutamic acid
Correct answer: 1 Answer variants: 5
QUESTION N 13. Which one of the following nutrients is a precursor for a cofactor that carries one-carbon
groups of different oxidation states?
Question type: 1. Choosing the only correct answer
Question value (points): 1
Answer variants:
1. Methionine
2. Thiamine
3. Folic acid
4. Biotin
5. Pyridoxine
Correct answer: 3 Answer variants: 5
QUESTION N 14. Most amino acids are substrates for transamination EXEPT:
Question type: 1. Choosing the only correct answer
Question value (points): 1
Answer variants:
1. Alanine
2. Threonine
3. Serine
4. Valine
Correct answer: 2 Answer variants: 4
QUESTION N 15. In hemoglobin degradation, the first bile pigment formed is:
Question type: 1. Choosing the only correct answer
Question value (points): 1
Answer variants:
1. Bilirubin
2. Biliverdin
3. Bile acids
4. Cholic acids
5. Deoxycholic acids
Correct answer: 2 Answer variants: 5
QUESTION N 16. In obstructive jaundice, the direct fraction of bilirubin is :
Question type: 1. Choosing the only correct answer
Question value (points): 1
Answer variants:
1. Increased
2. Decreased
3. Remains the same
4. None of the above
Correct answer: 1 Answer variants: 4
QUESTION N 17. In hepatocellular jaundice, urine bilirubin is:
Question type: 1. Choosing the only correct answer
Question value (points): 1
Answer variants:
1. Increased
2. Usually absent
3. Present in small amount
4. None of the above
Correct answer: 3 Answer variants: 4
QUESTION N 19. Epinephrine is formed from norepinephrine by which one of the following actions?
Question type: 1. Choosing the only correct answer
Question value (points): 1
Answer variants:
1. Hydroxylation
2. Decarboxylation
3. Oxidative deamination
4. O-Methylation
5. N-Methylation
Correct answer: 5 Answer variants: 5
QUESTION N 20. The action of allopurinol is best described as
Question type: 1. Choosing the only correct answer
Question value (points): 1
Answer variants:
1. leading to the inhibition of xanthine oxidase
2. inhibiting 5'-phosphoribosyl-1-pyrophosphate (PRPP) amidotransferase
3. increasing the solubility of uric acid in plasma
4. inhibiting the formation of PRPP
5. inhibiting leukocyte movement by affecting microtubule formation
Correct answer: 1 Answer variants: 5
QUESTION N 21. Biliverdin is converted to bilirubin by :
Question type: 1. Choosing the only correct answer
Question value (points): 1
Answer variants:
1. Reduction
2. Oxidation
3. Conjugation
4. Detoxification
5. All of the.above
Correct answer: 1 Answer variants: 5
QUESTION N 23. In hemolytic jaundice indirect fraction (unconjugated) of bilirubin:
Question type: 1. Choosing the only correct answer
Question value (points): 1
Answer variants:
1. Increases but more than the direct fraction
2. Increases
3. Increases but the direct fraction is more
4. None of the above
Correct answer: 1 Answer variants: 4
QUESTION N 25. During the degradation of heme, which one of the following actions occurs?
Question type: 1. Choosing the only correct answer
Question value (points): 1
Answer variants:
1. Heme is initially converted to bilirubin
2. Reduction of bilirubin yields biliverdin
3. Bilirubin is conjugated to glucuronic acid in the liver
4. Failure to conjugate bilirubin to glucuronic acid causes porphyria
5. Free (unconjugated) bilirubin may be reduced to urobilinogen in the liver
Correct answer: 3 Answer variants: 5
QUESTION N 26. What black hair and skin pigment can be formed from phenylalanine in human?
Question type: 5. Entering the answer manually using the keyboard
Question value (points): 1
Answer variants:
1. melanin
2. Melanin
All answers are correct Answer variants: 2
QUESTION N 27. Bilirubin is transported to liver in combination with :
Question type: 1. Choosing the only correct answer
Question value (points): 1
Answer variants:
1. Globulin
2. Albumin
3. Ceruloplasmin
4. Transferrin
Correct answer: 2 Answer variants: 4
QUESTION N 28. In hepatocellular jaundice, Vanden Bergh reaction is :
Question type: 1. Choosing the only correct answer
Question value (points): 1
Answer variants:
1. Indirect positive
2. Delayed positive
3. Direct positive
4. None of the above
Correct answer: 2 Answer variants: 4
QUESTION N 29. In obstructive jaundice, the urine bilirubin is :
Question type: 1. Choosing the only correct answer
Question value (points): 1
Answer variants:
1. Increased
2. Usually present
3. Present in small amount
4. None of the above
Correct answer: 1 Answer variants: 4
QUESTION N 30. Which one of the following amino acids can undergo deamination by dehydration?
Question type: 1. Choosing the only correct answer
Question value (points): 1
Answer variants:
1. Glutamine
2. Leucine
3. Serine
4. Valine
5. Lysine
Correct answer: 3 Answer variants: 5
QUESTION N 31. Match the following descriptions with the correct medical problem.
Question type: 4. Arranging the correspondences for the answers
Question value (points): 1
Answer variants:
1. Albinism
2. Jaundice
3. Adrenal tumor
4. Homocystinuria
5. Protoporphyria
Answer variants: 5
Correspondence variants:
A. May be characterized by high urinary levels of vanillylmandelic acid
B. May be caused by a defect in tyrosinase
C. Is one consequence of a vitamin B12 deficiency
D. None of above
Correct answers: 1-B; 2-D; 3-A; 4-D; 5-C Correspondence variants: 4
QUESTION N 32. In liver, bilirubin is conjugated with:
Question type: 1. Choosing the only correct answer
Question value (points): 1
Answer variants:
1. Glucose
2. Glucuronic acid
3. Cholic acid
4. Water
5. Bile acids
Correct answer: 2 Answer variants: 5
QUESTION N 33. In obstructive jaundice, Vanden Bergh reaction is:
Question type: 1. Choosing the only correct answer
Question value (points): 1
Answer variants:
1. Indirect positive
2. Delayed positive
3. Direct positive
4. None of the above
Correct answer: 3 Answer variants: 4
QUESTION N 34. In hemolytic jaundice, fecal stercobilinogen is:
Question type: 1. Choosing the only correct answer
Question value (points): 1
Answer variants:
1. Normal
2. Absent
3. Much more than normal
4. None of the above
Correct answer: 3 Answer variants: 4
QUESTION N 35. A patient with a genetic defect in the enzyme that produces N-acetylglutamate would
present with
Question type: 1. Choosing the only correct answer
Question value (points): 1
Answer variants:
1. elevated levels of argininosuccinate
2. no detectable citrulline
3. elevated levels of arginine
4. elevated levels of urea
5. no detectable ornithine
Correct answer: 2 Answer variants: 5
QUESTION N 36. Match the following descriptions to the correct amino acid.
Question type: 4. Arranging the correspondences for the answers
Question value (points): 4
Answer variants:
1. Arginine
2. Tyrosine
3. Tryptophan
4. Glycine
Answer variants: 4
Correspondence variants:
A. The precursor of thyroid hormones
B. The precursor of the potent vasoconstrictor serotonin
C. The precursor of nitric oxide
D. The precursor of heme
Correct answers: 1-C; 2-A; 3-B; 4-D Correspondence variants: 4
QUESTION N 37. Direct bilirubin is:
Question type: 1. Choosing the only correct answer
Question value (points): 1
Answer variants:
1. Water soluble
2. Bilirubin diglucuronide
3. Gives pink colour with diazo reagent
4. All of the above
5. None of the above
Correct answer: 4 Answer variants: 5
QUESTION N 38. In hepatocellular jaundice. Indirect fraction of bilirubin:
Question type: 2. Choosing several possible correct answers
Question value (points): 1
Answer variants:
1. Increases, but is more than the direct fraction
2. Increases
3. Increases, but the direct fraction is more
4. None of the above
Correct answers: 1; 2 Answer variants: 4
QUESTION N 39. In hemolytic jaundice, urine urobilinogen is:
Question type: 1. Choosing the only correct answer
Question value (points): 1
Answer variants:
1. Absent
2. Normal
3. Much more than normal
4. None of the above
Correct answer: 3 Answer variants: 4
QUESTION N 40. Match the following statements regarding disorders of amino acid catabolism with the
correct cofactor.
Question type: 4. Arranging the correspondences for the answers
Question value (points): 1
Answer variants:
2. Tetrahydrobiopterin
3. Thiamine
4. Vitamin B
Answer variants: 3
Correspondence variants:
A. The unavailability of this cofactor causes certain symptoms consistent with a diagnosis of phenylketonuria
B. Some patients with symptoms of maple syrup urine disease respond to therapeutic doses of this compound
C. Some patients with methylmalonic aciduria respond to therapeutic doses of this compound
Correct answers: 2-A; 3-B; 4-C Correspondence variants: 3
QUESTION N 41. Match the figures with the letters.
Question type: 4. Arranging the correspondences for the answers
Question value (points): 1
Answer variants:
1. DNA
2. RNA
3. Both
4. None
Answer variants: 4
Correspondence variants:
A. Hydrogen bonds are formed between nitrogen bases.
B. Hydrogen bonds are formed between C and T and between A and G.
C. Its molecule consists of one polynucleotide chain.
D. The chains in its molecule are antiparallel.
Correct answers: 1-D; 2-C; 3-A; 4-B Correspondence variants: 4
QUESTION N 42. Match the figures with the letters. The nucleic acids in the cell may be:
Question type: 4. Arranging the correspondences for the answers
Question value (points): 1
Answer variants:
1. DNA
2. mRNA
3. tRNA
4. rRNA
Answer variants: 4
Correspondence variants:
A. Adaptors of amino acids to mRNA codons.
B. Structural components of ribosomes.
C. Templates for protein synthesis.
D. Carriers of genetic information.
Correct answers: 1-D; 2-C; 3-A; 4-B Correspondence variants: 4
QUESTION N 43. In obstructive jaundie indirect fraction of bilirubin:
Question type: 1. Choosing the only correct answer
Question value (points): 1
Answer variants:
1. Increases but more than the direct fraction
2. Increases
3. Increases but the direct fraction is more
4. None of the above
Correct answer: 3 Answer variants: 4
QUESTION N 44. In obstructive jaundice, urine urobilinogen is:
Question type: 1. Choosing the only correct answer
Question value (points): 1
Answer variants:
1. Absent
2. Normal
3. Much more than normal
4. None of the above
Correct answer: 1 Answer variants: 4
QUESTION N 45. In obstructive jaundice, fecal stercobilinogen is:
Question type: 1. Choosing the only correct answer
Question value (points): 1
Answer variants:
1. Normal
2. Absent
3. Much more than normal
4. None of the above
Correct answer: 2 Answer variants: 4
QUESTION N 47. What compound is end product of amino acid nitrogen metabolism in human?
Question type: 1. Choosing the only correct answer
Question value (points): 1
Answer variants:
1. Bile acids
2. Ketone bodies
3. Urea
4. Barium sulphate
Correct answer: 3 Answer variants: 4
QUESTION N 48. Hemoglobin contains the number of gram atoms of iron per mole in the ferrous state:
Question type: 1. Choosing the only correct answer
Question value (points): 1
Answer variants:
1. 1
2. 2
3. 3
4. 4
Correct answer: 4 Answer variants: 4
QUESTION N 49. Which of the following compounds are ROS formed in human body (3)?
Question type: 2. Choosing several possible correct answers
Question value (points): 2
Answer variants:
1. Ozone
2. Superoxide anion radical
3. Carbon dioxide
4. Hydroxide radical
5. Sulphur dioxide
6. Alkoxi radica
Correct answers: 2; 4; 6Answer variants: 6
QUESTION N 50. Biorole of amino acids are determined of their utilization in the synthesis of (3)
Question type: 2. Choosing several possible correct answers
Question value (points): 2
Answer variants:
1. Heme
2. Proteins
3. Biogenic amines and hormones
4. Fatty acids and fats
5. Glucose
Correct answers: 1; 2; 3Answer variants: 5
QUESTION N 51. The end product of amino acid nitrogen metabolism in uricotelic organisms
Question type: 1. Choosing the only correct answer
Question value (points): 1
Answer variants:
1. Bilirubin
2. Urea
3. Uric acid
4. Biliverdin
Correct answer: 3 Answer variants: 4
QUESTION N 52. The porphyrins are cyclic compounds formed through methylene bridges by the linkages of
pyrrol rings number:
Question type: 1. Choosing the only correct answer
Question value (points): 1
Answer variants:
1. 4
2. 3
3. 2
4. 1
Correct answer: 1 Answer variants: 4
QUESTION N 53. Choose lipids that are degradable by ROS in physiological conditions (2):
Question type: 2. Choosing several possible correct answers
Question value (points): 2
Answer variants:
1. Cholesterol
2. Dipalmitoyl phospatidylethanolamine
3. 1-Palmitoyl 2-linoloyl phospatidylcholine
4. 1,3-Dipalmitoyl 2-oleyl glycerol
5. Arachidonic acid
6. Methyl stearate
Correct answers: 3; 5 Answer variants: 6
QUESTION N 54. Match the letters with the figures:
Question type: 4. Arranging the correspondences for the answers
Question value (points): 3
Answer variants:
1. Trypsin
2. Pepsin
3. Aminopeptidase
4. Chymotrypsin
5. Carboxypeptidase
Answer variants: 5
Correspondence variants:
A. Endopeptodases
B. Exopeptidases
Correct answers: 1-A; 2-A; 3-B; 4-A; 5-BCorrespondence variants: 2
QUESTION N 55. Match the letters with the figures:
Question type: 4. Arranging the correspondences for the answers
Question value (points): 2
Answer variants:
1. Belong to the class of hydrolases
2. Hydrolyze peptide bonds in which the carboxyl group is provided by an aromatic A.A.
3. Hydrolyze peptide bonds in which the carboxyl group is provided by hydrophobic A.A.
4. Belong to exopeptidases
Answer variants: 4
Correspondence variants:
A. Pepsin
B. Chymotrypsin
C. Both
D. None
Correct answers: 1-C; 2-A; 3-B; 4-D Correspondence variants: 4
QUESTION N 56. Note localisations of urea biosynthesis enzymes in liver cells
Question type: 2. Choosing several possible correct answers
Question value (points): 2
Answer variants:
1. Cytosol
2. Mitochondria
3. Microsomes
4. Nuclei
Correct answers: 1; 2 Answer variants: 4
QUESTION N 57. Heme is synthesized by the incorporation of ferrous ion (Fe++) into protoporhyrin IX being
catalyzed by the enzyme:
Question type: 1. Choosing the only correct answer
Question value (points): 1
Answer variants:
1. Ferroxidase
2. Ferroreductase
3. Ferrochelatese
4. None of the above
Correct answer: 3 Answer variants: 4
QUESTION N 58. Choose correct characteristics (2) of the human cytochrome P450:
Question type: 2. Choosing several possible correct answers
Question value (points): 2
Answer variants:
1. It is hemoprotein
2. It is flavoprotein
3. It catalyzes redox reactions
4. It is localized in mitochondria only
5. It is a protein with transport function
Correct answers: 1; 3 Answer variants: 5
QUESTION N 59. Match the letters with the figures:
Question type: 4. Arranging the correspondences for the answers
Question value (points): 1
Answer variants:
1. Positive nitrogen balance
2. Negative nitrogen balance (3)
3. Nitrogen equilibrium
Answer variants: 3
Correspondence variants:
A. Grow old
B. Adult, normal nutrition
C. Long severe diseases
D. Time of grow up
E. Starvation
Correct answers: 1-D; 2-A, C, E; 3-B Correspondence variants: 5
QUESTION N 60. If one amino acid is fed in excess, the absorption of another is:
Question type: 1. Choosing the only correct answer
Question value (points): 1
Answer variants:
1. Slightly accelerated
2. Moderately accelerated
3. Highly accelerated
4. Retarded
Correct answer: 4 Answer variants: 4
QUESTION N 61. Glutathione is a:
Question type: 1. Choosing the only correct answer
Question value (points): 1
Answer variants:
1. Dipeptide
2. Tripeptide
3. Polypeptide
4. None of the above
Correct answer: 2 Answer variants: 4
QUESTION N 62. Full up the blank of the following:Glutamate is synthesized in animals by .(abbreviated).
Question type: 5. Entering the answer manually using the keyboard
Question value (points): 1
Answer variants:
1. GDH
2. glutamatedehydrogenase
All answers are correct Answer variants: 2
QUESTION N 63. Oxidative conversion of many amino acids to their corresponding - keto acids occurs in
mammalian:
Question type: 1. Choosing the only correct answer
Question value (points): 1
Answer variants:
1. Liver and kidney
2. Adipose tissue
3. Pancreas
4. Intestine
Correct answer: 1 Answer variants: 4
QUESTION N 64. The activity of mammalian L-amino acid oxidase, an FMN-flavoproteins, is quite:
Question type: 1. Choosing the only correct answer
Question value (points): 1
Answer variants:
1. Slow
2. Rapid
3. Both of the above
4. None of the above
Correct answer: 1 Answer variants: 4
QUESTION N 65. The transaminase activity needs the coenzyme:
Question type: 1. Choosing the only correct answer
Question value (points): 1
Answer variants:
1. ATP
2. PLP
3. FAD
4. NAD+
Correct answer: 2 Answer variants: 4
QUESTION N 66. Transamination is a:
Question type: 1. Choosing the only correct answer
Question value (points): 1
Answer variants:
1. Irreversible process
2. Reversible process
3. Both of the above
4. None of the above
Correct answer: 2 Answer variants: 4
QUESTION N 67. Match the corresponding amino acids formed by transamination of each of the following
a-keto acids
Question type: 4. Arranging the correspondences for the answers
Question value (points): 1
Answer variants:
1. 2-ketoglutaric acid
2. 3-hydroxy phenyl pyruvic acid
3. 2-keto isovalerate
4. phenyl pyruvic acid
5. 2- keto isocaproate
Answer variants: 5
Correspondence variants:
A. Glu
B. Tyr
C. Val
D. Phen
E. Leu
Correct answers: 1-A; 2-B; 3-C; 4-D; 5-E Correspondence variants: 5
QUESTION N 68. The metabolism of protein is integrated with that of carbohydrate and fat through:
Question type: 1. Choosing the only correct answer
Question value (points): 1
Answer variants:
1. Oxaloacetate
2. Citrate
3. Isocitrate
4. Malate
Correct answer: 1 Answer variants: 4
QUESTION N 69. The major mechanism of the removal of ammonia in the brain is:
Question type: 1. Choosing the only correct answer
Question value (points): 1
Answer variants:
1. Glutamine formation
2. Urea formation
3. Uric acid production
4. Creatinine formation
5. All of the above
Correct answer: 1 Answer variants: 5
QUESTION N 70. The major mechanism of removal of ammonia in liver is:
Question type: 1. Choosing the only correct answer
Question value (points): 1
Answer variants:
1. Glutamine formation
2. Urea formation
3. Uric acid formation
4. Creatinine formation
5. All of the above
Correct answer: 2 Answer variants: 5
QUESTION N 71. The major mechanism of the removal of ammonia in kidney is:
Question type: 1. Choosing the only correct answer
Question value (points): 1
Answer variants:
1. Ammonia formation
2. Urea formation
3. Uric acid formation
4. Creatinine formation
5. All of the above
Correct answer: 1 Answer variants: 5
QUESTION N 72. The major source of nitrogen in urinary urea is:
Question type: 1. Choosing the only correct answer
Question value (points): 1
Answer variants:
1. Glutamine
2. Citrulline
3. Purine breakdown
4. Amino acid deamination and transamination
5. Ammonia synthesised in distal tubles of kidneys
Correct answer: 4 Answer variants: 5
QUESTION N 73. Uremia occurs in:
Question type: 1. Choosing the only correct answer
Question value (points): 1
Answer variants:
1. Cirrhosis of the liver
2. Nephritis
3. Diabetes Mellitus
4. Coronary thrombosis
Correct answer: 2 Answer variants: 4
QUESTION N 74. Full up the blank of the following:Glutamate is synthesized in animals in presence of the
coenzyme (or) ... (abbreviated).
Question type: 5. Entering the answer manually using the keyboard
Question value (points): 1
Answer variants:
1. NAD or NADP
2. NAD NADP
3. NAD, NADP
All answers are correct Answer variants: 3
QUESTION N 75. What of following compounds can influence on carbamoyl phosphate synthetase activity
and structure?
Question type: 1. Choosing the only correct answer
Question value (points): 1
Answer variants:
1. N-Acetyl glutamate
2. N-Acetyl aspartate
3. Neuraminic acid
4. Oxalate
Correct answer: 1 Answer variants: 4
QUESTION N 76. Synthesis of 1 mol of urea needs in ... macroenergic bonds:
Question type: 1. Choosing the only correct answer
Question value (points): 1
Answer variants:
1. 2
2. 3
3. 4
4. 5
Correct answer: 3 Answer variants: 4
QUESTION N 77. Urea biosynthesis occurs mainly in the liver involving thee number of amino acids:
Question type: 1. Choosing the only correct answer
Question value (points): 1
Answer variants:
1. 3
2. 4
3. 5
4. 6
Correct answer: 4 Answer variants: 4
QUESTION N 78. Note normal range of daily urea excretion with urine (expressed as grams)
Question type: 1. Choosing the only correct answer
Question value (points): 1
Answer variants:
1. 10 to 20
2. 15 to 25
3. 20 to 30
4. 25 to 35
Correct answer: 3 Answer variants: 4
QUESTION N 79. In severe acidosis, rate of urea excretion is:
Question type: 1. Choosing the only correct answer
Question value (points): 1
Answer variants:
1. Decreased
2. Slightly increased
3. Highly increased
4. Moderately increased
Correct answer: 1 Answer variants: 4
QUESTION N 80. Uremia occurs in:
Question type: 1. Choosing the only correct answer
Question value (points): 1
Answer variants:
1. Cirrhosis
2. Nephritis
3. Diabetes mellitus
4. Coronary thrombosis
Correct answer: 2 Answer variants: 4
QUESTION N 81. Arginine is converted to ... by arginase with urea release.
Question type: 5. Entering the answer manually using the keyboard
Question value (points): 1
Answer variants:
1. ornithine
2. ORNITINE
3. Ornitine
All answers are correct Answer variants: 3
QUESTION N 82. Indicate true or false of the followings:
Question type: 4. Arranging the correspondences for the answers
Question value (points): 1
Answer variants:
1. Arginine is an aromatic amino acid.
2. In ammoniotelic organisms, uric acid is the end product of amino acid nitrogen metabolism.
3. The amino acids ornithine and citrulline occurs in protein.
Answer variants: 3
Correspondence variants:
A. true
B. false
Correct answers: 1-B; 2-B; 3-B Correspondence variants: 2
QUESTION N 83. Match the letters with the figures:
Question type: 4. Arranging the correspondences for the answers
Question value (points): 4
Answer variants:
1. Plant urease.
2. Ammonia intoxication.
3. Use of antibiotics.
4. In hyperammonemia type II.
Answer variants: 4
Correspondence variants:
A. Patients suffer from a deficiency of ornithine transcarbamoylase.
B. Leads to decreasing the absorption of ammonia formed by bacterial decomposition in the intestine.
C. Is caused by all disorders of urea synthesis.
D. Splits urea into ammonia and carbon dioxide at the optimum temperature and pressure.
Correct answers: 1-D; 2-C; 3-B; 4-A Correspondence variants: 4
QUESTION N 84. Complete this sentence: DOPA is converted to dopamine by a decarboxylase with ... as
coenzyme.
Question type: 5. Entering the answer manually using the keyboard
Question value (points): 1
Answer variants:
1. PLP
2. pyridoxal phosphate
3. plp
4. Pyridoxal phosphate
All answers are correct Answer variants: 4
QUESTION N 85. What compound formed by 5-hydroxytryptophan decarboxylation is main human
vasoconstrictor?
Question type: 5. Entering the answer manually using the keyboard
Question value (points): 1
Answer variants:
1. serotonin
2. Serotonin
3. SEROTONIN
4. 5-hydroxytryptamine
All answers are correct Answer variants: 4
QUESTION N 86. Complete this sentence: Epinephrine is formed from norepinephrine by ...
Question type: 5. Entering the answer manually using the keyboard
Question value (points): 1
Answer variants:
1. methylation
2. Methylation
All answers are correct Answer variants: 2
QUESTION N 87. What hormone are formed by the acethylation and methylation of serotonin?
Question type: 5. Entering the answer manually using the keyboard
Question value (points): 1
Answer variants:
1. melatonin
2. Melatonin
3. MELATONIN
All answers are correct Answer variants: 3
QUESTION N 88. What natural compound is required for transformation of methionine to
S-adenosylmethionine?
Question type: 5. Entering the answer manually using the keyboard
Question value (points): 1
Answer variants:
1. ATP
All answers are correct Answer variants: 1
QUESTION N 89. Complete this sentence: Creatinine is formed largely in muscles by the irreversible and
nonenzymic removal of water from ...
Question type: 5. Entering the answer manually using the keyboard
Question value (points): 1
Answer variants:
1. creatine phosphate
2. CP
3. Creatine phosphate
All answers are correct Answer variants: 3
QUESTION N 90. What compound (except oxygen) is required for the creatine methylation?
Question type: 5. Entering the answer manually using the keyboard
Question value (points): 1
Answer variants:
1. ATP
All answers are correct Answer variants: 1
QUESTION N 91. Choose the correct answer. Cellular fraction of blood in volume per cent is :
Question type: 1. Choosing the only correct answer
Question value (points): 1
Answer variants:
1. 40
2. 45
3. 50
4. 55
Correct answer: 2 Answer variants: 4
QUESTION N 92. Fill up gap in the following sentence: The activation of methionine occurs requiring ATP,
Mg(2+) and ... as well as methionine activating enzyme.
Question type: 5. Entering the answer manually using the keyboard
Question value (points): 1
Answer variants:
1. gluthathione
2. Gluthathione
3. GSH
All answers are correct Answer variants: 3
QUESTION N 93. Match the letters with the figures:
Question type: 4. Arranging the correspondences for the answers
Question value (points): 4
Answer variants:
1. Methionine
2. 5-hydroxytryptamine
3. Creatine synthesis
4. Excessive creatinuria
Answer variants: 4
Correspondence variants:
A. May occur in febrile and wasting diseases, diabetes mellitus, congenital muscle dystrophies
B. Is participated by glycine, arginine and methionine
C. Is involved in the detoxification of nicotinic acid for the formation of the excretory product trigonelline
D. A stimulant of the central nervous system and also promotes peristalsis
Correct answers: 1-C; 2-D; 3-B; 4-A Correspondence variants: 4
QUESTION N 94. Write name of enzyme catalyzes phenylalanine convertion to tyrosine
Question type: 5. Entering the answer manually using the keyboard
Question value (points): 1
Answer variants:
1. phenylalanine hydroxylase complex
2. phenylalanine hydroxylase
3. Phenylalanine hydroxylase
4. Phenylalanine hydroxylase complex
5. PHC
6. phc
All answers are correct Answer variants: 6
QUESTION N 95. Comlpete this sentence: Ascorbic acid is cofactor of enzymatic transformation
p-hydroxyphenylpyruvate to ... by p-hydroxyphenylpyruvate hydroxylase.
Question type: 5. Entering the answer manually using the keyboard
Question value (points): 1
Answer variants:
1. homogentisic acid
2. Homogentisic acid
3. homogentisic
All answers are correct Answer variants: 3
QUESTION N 96. Write name of cofactor of homogentisate oxidase.
Question type: 5. Entering the answer manually using the keyboard
Question value (points): 1
Answer variants:
1. iron
2. Iron
3. Fe
All answers are correct Answer variants: 3
QUESTION N 97. Fill gap in this sentence: The hydroxylation of dopamine produces norepinephrin in the
presence of ... and molecular oxygen.
Question type: 5. Entering the answer manually using the keyboard
Question value (points): 1
Answer variants:
1. ascorbic acid
2. Ascorbic acid
3. vitamin C
4. Vitamin C
5. vitamine C
All answers are correct Answer variants: 5
QUESTION N 98. Fill gap in the following sentence: Alkaptonuria is associated with excess excretion of
homogentisic acid in urine as result of ... deficiency
Question type: 5. Entering the answer manually using the keyboard
Question value (points): 1
Answer variants:
1. homogentisic acid oxidase
2. Homogentisic acid oxidase
All answers are correct Answer variants: 2
QUESTION N 99. Complete of this sentence: Colorization of urine is result of oxidation of the ... by air oxygen.
Question type: 5. Entering the answer manually using the keyboard
Question value (points): 1
Answer variants:
1. homogentisic acid
2. homogentisate
All answers are correct Answer variants: 2
QUESTION N 100. Complete this sentence: Leucine catabolism produces ... and acetyl CoA .
Question type: 5. Entering the answer manually using the keyboard
Question value (points): 1
Answer variants:
1. acetoacetate
All answers are correct Answer variants: 1
QUESTION N 101. Complete this sentence: Both valine and isoleucine produce ... during their catabolism
Question type: 5. Entering the answer manually using the keyboard
Question value (points): 1
Answer variants:
1. succinyl-CoA
2. succinyl CoA
All answers are correct Answer variants: 2
QUESTION N 102. The porphyrins are cyclic compounds formed through methylene bridges by the linkages of
pyrrol rings number:
Question type: 1. Choosing the only correct answer
Question value (points): 1
Answer variants:
1. 4
2. 3
3. 2
4. 1
Correct answer: 1 Answer variants: 4
QUESTION N 103. Indicate true or false of the following:
Question type: 4. Arranging the correspondences for the answers
Question value (points): 1
Answer variants:
1. The condensation of active succinate and glycine is catalyzed by the enzyme d-aminolevulinic acid synthase.
2. Two mols of d-aminolevulinic acid condense to form porphobilinogen catalyzed by the enzyme aminolevulinic acid
3. Methemoglobin can carry oxygen in blood.
4. Histidine contained in hemoglobin exerts its buffering action through its basic imidazole ring for which hemoglobin
plays an important role in regulating the acid-base balance of blood.
5. Iron in the ferrous state is incorporated into protoporphyrin to form heme.
Answer variants: 5
Correspondence variants:
A. False
B. True
Correct answers: 1-B; 2-B; 3-A; 4-B; 5-BCorrespondence variants: 2
QUESTION N 104. Indicate true or false of the following:
Question type: 4. Arranging the correspondences for the answers
Question value (points): 1
Answer variants:
1. Porphyrias are of two types congenital and acquired.
2. Erythropoietic porphyria causes highly increased excretion of uroporphyrin I and, to a lesser extent,
coproporphyrin
I in both urine and feces.
3. Erythropoietic coproporphyria shows large amounts of coproporphyrin II in the erythrocytes.
4. Acute intermittent porphyria causes periodic attacks of abdominal pain which is associated with fever and
leukocytosis.
Answer variants: 4
Correspondence variants:
A. False
B. True
Correct answers: 1-B; 2-B; 3-A; 4-B Correspondence variants: 2
QUESTION N 105. Fill up the blanks of the following:The condition by which the excretion of both
coproporphyrin and uroporphyrin in-creases is said to be ...
Question type: 5. Entering the answer manually using the keyboard
Question value (points): 1
Answer variants:
1. Porphyria
2. porphyria
All answers are correct Answer variants: 2
QUESTION N 106. Fill up the blanks of the following:Erythropoietic porphyria has got tendency to hemolysis
and defective ...
Question type: 5. Entering the answer manually using the keyboard
Question value (points): 1
Answer variants:
1. Erythropoieses
2. erythropoieses
All answers are correct Answer variants: 2
QUESTION N 107. Fill up the blanks of the following:Increased serum PBI and hypercholestemia occur in
acute ... porphyria.
Question type: 5. Entering the answer manually using the keyboard
Question value (points): 1
Answer variants:
1. Intermittent
2. intermittent
All answers are correct Answer variants: 2
QUESTION N 108. Fill up the blanks of the following: In erythropoietic porphyria, there is increased ... and
uroporphyrin in the circulating erythrocytes, plasma and the feces.
Question type: 5. Entering the answer manually using the keyboard
Question value (points): 1
Answer variants:
1. Protoporphyrin
2. protoporphyrin
All answers are correct Answer variants: 2
QUESTION N 109. Fill up the blanks of the following: Acute intermittent porphyria is due to a marked increase
of hepatic ...
Question type: 5. Entering the answer manually using the keyboard
Question value (points): 1
Answer variants:
1. ALA-synthase
2. ALA synthase
3. ALA - synthase
All answers are correct Answer variants: 3
QUESTION N 110. Fill up the blanks of the following:In porphyria variegata there is increased hepatic ...
Question type: 5. Entering the answer manually using the keyboard
Question value (points): 1
Answer variants:
1. ALA-synthase
2. ALA synthase
3. ALA - synthase
All answers are correct Answer variants: 3
QUESTION N 111. Fill up the blanks of the following: Hereditary coproporphyria causes increased urinary
output of ... and ALA during acute attacks.
Question type: 5. Entering the answer manually using the keyboard
Question value (points): 1
Answer variants:
1. Porphobilinogen
2. porphobilinogen
All answers are correct Answer variants: 2
QUESTION N 112. Fill up the blanks of the following: Acquired porphyria is caused by severe ... diseases and
ingestion of certain toxins.
Question type: 5. Entering the answer manually using the keyboard
Question value (points): 1
Answer variants:
1. Liver
2. liver
All answers are correct Answer variants: 2
QUESTION N 113. Fill up the blanks of the following: In acquired porphyria, there is increased excretion of ...
in urine.
Question type: 5. Entering the answer manually using the keyboard
Question value (points): 1
Answer variants:
1. Uroporphyrin
2. uroporphyrin
All answers are correct Answer variants: 2
QUESTION N 114. Fill up the blanks of the following: Porphyria cutanea tarda shows frequent rise in serum ...
Question type: 5. Entering the answer manually using the keyboard
Question value (points): 1
Answer variants:
1. Iron
2. iron
All answers are correct Answer variants: 2
QUESTION N 115. Match the letters with the figures:
Question type: 4. Arranging the correspondences for the answers
Question value (points): 2
Answer variants:
1. Catabolism of hemoglobin means
2. When hemoglobin is catabolized in the body
3. The porphyrin portion is broken down mainly
4. The green conjugated protein
Answer variants: 4
Correspondence variants:
A. Formed by the oxidation of hemoglobin by oxygen in presence of ascorbic acid is said to be choleglobin.
B. In the reticuloendothelial cells of the liver, spleen and bonemarrow.
C. Conversion of hemoglobin to bile pigments and metabolism of bile pigments.
D. The iron enters ferritin pool for reuse.
Correct answers: 1-C; 2-D; 3-B; 4-A Correspondence variants: 4
QUESTION N 116. Match the letters with the figures:
Question type: 4. Arranging the correspondences for the answers
Question value (points): 2
Answer variants:
1. Bilirubin , the chief pigment in human bile
2. Normal serum bilirubin level
3. L-stercobilinogen
4. L-stercobilin
Answer variants: 4
Correspondence variants:
A. Is the reduction product of bilirubin.
B. Is formed by the reduction of biliverdin by the enzyme bilirubin reductase requiring NAD+ or NADP+.
C. Is an orange yellow pigment which gives the normal colour of the feces and is strongly levorota-tory.
D. 0,1 to 1,5 mg per cent.
Correct answers: 1-B; 2-D; 3-A; 4-C Correspondence variants: 4
QUESTION N 117. Match the letters with the figures:
Question type: 4. Arranging the correspondences for the answers
Question value (points): 2
Answer variants:
1. Jaundice occurs
2. Crigler-Najjar syndrome. Type II
3. Gilberts disease
4. Crigler-Najjar syndrome. Type I
Answer variants: 4
Correspondence variants:
A. Is caused by the milder defect in the bilirubin conjugated system.
B. Is caused by the defect in the hepatic clearance of bilirubin.
C. Is owing to a primary metabolic defect in the conju-gation of bilirubin.
D. When the concentration of bile pigment in blood is more and diffuses into the tissues producing a yellow
Correct answers: 1-D; 2-A; 3-B; 4-C pigmentation. Correspondence variants: 4
QUESTION N 118. Match the letters with the figures:
Question type: 4. Arranging the correspondences for the answers
Question value (points): 2
Answer variants:
1. In hepatic jaundice
2. In hemolytic jaundice
3. In obstructive jaundice
4. Unconjugated hyperbilirubinemia
Answer variants: 4
Correspondence variants:
A. Results from liver dysfunction caused by chloroform, CCl4, hepatitis virus and cirrhosis.
B. Urobilinogen increased and bilirubin present in urine and urobilinogen decreased in feces.
C. Urobilinogen increased and bilirubin absent in urine and urobilinogen increased in feces.
D. Urobilinogen absent in urine and urobilinogen trace to absent in feces.
Correct answers: 1-B; 2-C; 3-D; 4-A Correspondence variants: 4
QUESTION N 119. Match the letters with the figures:
Question type: 4. Arranging the correspondences for the answers
Question value (points): 2
Answer variants:
1. Apoferritin
2. Ceruloplasmin
3. Ferritin
4. Transferrin
Answer variants: 4
Correspondence variants:
A. Glycoprotein containing carbohydrate
B. 24 identical protomers
C. Ferroxidase
D. is located in the mucosal cells
Correct answers: 1-D; 2-C; 3-B; 4-A Correspondence variants: 4
QUESTION N 120. Match the letters with the figures:
Question type: 4. Arranging the correspondences for the answers
Question value (points): 2
Answer variants:
1. Ceruloplasmin
2. Ferritin
3. Transferrin
Answer variants: 3
Correspondence variants:
A. Storage of iron
B. Transport of iron
C. Conversion of ferrous ion to ferric ion
Correct answers: 1-C; 2-A; 3-B Correspondence variants: 3
QUESTION N 121. Match the letters with the figures:
Question type: 4. Arranging the correspondences for the answers
Question value (points): 2
Answer variants:
1. Transferrin
2. Ferritin
3. Ceruloplasmin
Answer variants: 3
Correspondence variants:
A. Fe2+ into Fe3+
B. Fe3+
C. (FeOOH)8(FeOOPO3H2)
Correct answers: 1-B; 2-C; 3-A Correspondence variants: 3
QUESTION N 122. Indicate true or false of the following:
Question type: 4. Arranging the correspondences for the answers
Question value (points): 1
Answer variants:
1. Iron is not involved in the cellular respiration
2. A defect of hemoglobin synthesis in anemia is commonly found in copper deficiency
3. Transferrin is a specific iron binding protein.
Answer variants: 3
Correspondence variants:
A. True
B. False
Correct answers: 1-B; 2-B; 3-A Correspondence variants: 2
QUESTION N 123. Indicate true or false of the following:
Question type: 4. Arranging the correspondences for the answers
Question value (points): 1
Answer variants:
1. Ceruloplasmin is a copper binding plasma protein
2. Iron deficiency anemia is not prevalent among children and adolescent girls.
3. Transferrin is not present in intestines and also in liver.
Answer variants: 3
Correspondence variants:
A. True
B. False
Correct answers: 1-A; 2-B; 3-A Correspondence variants: 2
QUESTION N 124. The rate of synthesis of the transferrin receptor increases when iron levels are:
Question type: 1. Choosing the only correct answer
Question value (points): 1
Answer variants:
1. Increases
2. Decreases
3. Low
4. Higher
Correct answer: 3 Answer variants: 4
QUESTION N 125. Match the letters with the figures:
Question type: 4. Arranging the correspondences for the answers
Question value (points): 2
Answer variants:
1. Iron deficiency state
2. Iron deficiency anemia
3. Both
4. None of the above
Answer variants: 4
Correspondence variants:
A. Reduction of the Hb contents
B. Level of ferritin is decreased, Hb-is normal
C. Flush
D. Weakness
Correct answers: 1-B; 2-A; 3-D; 4-C Correspondence variants: 4
QUESTION N 126. Iron metabolism is disturbed mainly by the following causes:
Question type: 1. Choosing the only correct answer
Question value (points): 1
Answer variants:
1. Decreased formation of hemoglobin
2. Decrease in circulating hemoglobin
3. Abnormalities in the serum iron concentration
4. Abnormal deposition of iron-containing pigment in the tissues
5. All of the above
6. None of the above
Correct answer: 5 Answer variants: 6
QUESTION N 127. The causes of hemochromatosis may be:
Question type: 1. Choosing the only correct answer
Question value (points): 1
Answer variants:
1. Increased absorbtion of iron in the intestin
2. Decreased absorbtion of iron in the intestin
3. Intensive lactation
4. Friquent and massive blood transfusion
5. Decreased secretion of gastric juice
6. Increased secretion of gastric juice
7. All of the above
Correct answer: 1 Answer variants: 7
QUESTION N 128. The most frequent causes of iron deficiency anemia are (3):
Question type: 2. Choosing several possible correct answers
Question value (points): 1
Answer variants:
1. Systematic bleeding
2. Operation on gastro-intestinal tract
3. Pregnancy
4. Congeniatal deficiency of transferrin
Correct answers: 1; 2; 4Answer variants: 4
QUESTION N 129. Choose three types of bioactive molecules that are main targets of damage by ROS (3)?
Question type: 2. Choosing several possible correct answers
Question value (points): 3
Answer variants:
1. Amino acids
2. Proteins
3. Biogenic amines
4. Nucleic acids
5. Urea
6. Cations of alkalic metals
7. Lipids
Correct answers: 2; 4; 7Answer variants: 7
QUESTION N 130. Which one of the following ROS is especially dangerous for genetic information?
Question type: 1. Choosing the only correct answer
Question value (points): 1
Answer variants:
1. Lipid peroxyl radical
2. Hydrogen peroxide
3. Singlet oxygen
4. Hydroxyl radical
Correct answer: 4 Answer variants: 4
QUESTION N 131. Choose compounds directly defend cellular membranes against lipid peroxidation? (2)
Question type: 2. Choosing several possible correct answers
Question value (points): 2
Answer variants:
1. Superoxide dismutase
2. Tocopherols
3. Uric acid
4. Ascorbic acid
5. Glutathione peroxidase
6. Glutathione reductase
7. Catalase
8. Carotene
Correct answers: 2; 5 Answer variants: 8
QUESTION N 132. Which of following antioxidants are involved in intracellular ROS level control? (4)
Question type: 2. Choosing several possible correct answers
Question value (points): 2
Answer variants:
1. Superoxide dismutase
2. Tocopherols
3. Uric acid
4. Ascorbic acid
5. Glutathione peroxidase
6. Glutathione reductase
7. Catalase
8. Carotene
Correct answers: 1; 5; 6; 7 Answer variants: 8
QUESTION N 133. Choose diseases developing due to ROS hyperproduction or inadequate antioxidant
defence: (2)
Question type: 2. Choosing several possible correct answers
Question value (points): 2
Answer variants:
1. Cataracta
2. Familial hypercholesterolemia
3. Diabetic angiopathy
4. Galactosemia
Correct answers: 1; 3 Answer variants: 4
QUESTION N 134. Match the letters with the figures:
Question type: 4. Arranging the correspondences for the answers
Question value (points): 2
Answer variants:
1. Esterase
2. Microsomal monooxygenases
3. Cytochrom P450
4. Momoamine oxidase
Answer variants: 4
Correspondence variants:
A. need presence of NAD(P)H
B. degrades catecholamines
C. is a hydrolase
D. is involved in xenobiotic hydroxylation
Correct answers: 1-C; 2-A; 3-D; 4-B Correspondence variants: 4
QUESTION N 135. Why high intensity of microsomal hydroxylation can be deleterious? (2)
Question type: 2. Choosing several possible correct answers
Question value (points): 2
Answer variants:
1. Microsomal hydroxylation leads to deficiency of the flavin coenzymes
2. Tissue oxygen metabolism is disturbed by microsomal hydroxylation
3. Microsomal hydroxylation stimulates ROS generation
4. Microsomal hydroxylation can convert xenobiotic to more toxic compounds
5. Microsomal hydroxylation inhibits production of ATP
Correct answers: 3; 4 Answer variants: 5
QUESTION N 136. Conjugating enzymes serve for: (3)
Question type: 2. Choosing several possible correct answers
Question value (points): 3
Answer variants:
1. For decrease of toxicity of xenobiotics
2. For decrease of ATP outgoing
3. For elevation of oxygen consumption
4. For improvement of intracellular metabolism
5. For improvement of xenobiotic excretion
Correct answers: 1; 4; 5Answer variants: 5
QUESTION N 137. What drug can be transformed to cytotoxic product by microsomal monooxygenases?
Question type: 1. Choosing the only correct answer
Question value (points): 1
Answer variants:
1. Ibuprofen
2. Nitroglycerine
3. Acetaminophen
4. Ciprofloxacine
5. Penicilline
Correct answer: 3 Answer variants: 5
QUESTION N 138. Which features does glutathione have? (3)
Question type: 2. Choosing several possible correct answers
Question value (points): 2
Answer variants:
1. Glutathione consists of sulfur, carbon and oxygen
2. Glutathione is a protein
3. Glutathione enters into the composition of glutathione S-transferase
4. Glutathione is lipophilic compound
5. Glutathione contains a glutamic acid residue
6. Glutathione is easy oxidized compound
Correct answers: 1; 5; 6Answer variants: 6
QUESTION N 139. Which of following compounds are involved in detoxication of nitrogen-containing products
of putrefaction? (4)
Question type: 2. Choosing several possible correct answers
Question value (points): 2
Answer variants:
1. S-Adenosylmethionine
2. Adenosine 3-phosphate 5-phosphosulfate (PAPS)
3. Cysteine
4. Cholesterol
5. Taurine
6. Glutathione
Correct answers: 1; 2; 5; 6 Answer variants: 6
QUESTION N 140. Exclude non-conjugating agents from the following list: (2)
Question type: 2. Choosing several possible correct answers
Question value (points): 4
Answer variants:
1. UDP-glucuronic acid
2. Glutathione
3. Glycine
4. Alanine
5. S-Adenosylmethionine
6. S-Adenosylhomocysteine
7. Adenosine 3-phosphate 5-phosphosulfate (PAPS)
8. UDP-glucose
Correct answers: 4; 6 Answer variants: 8
QUESTION N 141. Which of following compounds are cancerogenous? (4)
Question type: 2. Choosing several possible correct answers
Question value (points): 2
Answer variants:
1. Glucose 6-phosphate
2. Organic nitrosamines
3. Selenium
4. Soluble salt of lead
5. Benzopyrenes
6. Alifatic chlorosubstituted hydrocarbons
7. Aromatic amines
Correct answers: 2; 5; 6; 7 Answer variants: 7
QUESTION N 142. How do tumor cells produce ATP chiefly?
Question type: 1. Choosing the only correct answer
Question value (points): 1
Answer variants:
1. Via aerobic degradation of glucose
2. Via glycolysis pathway
3. Via pehtose phosphate pathway
4. Via ?-oxidation of fatty acids
Correct answer: 2 Answer variants: 4
QUESTION N 143. Choose correct characteristics of tumor cells: (2)
Question type: 2. Choosing several possible correct answers
Question value (points): 1
Answer variants:
1. Decreased oxygen consumption
2. Elevated oxygen consumption
3. High level of differentiation
4. Immortality
5. Loosing of endoplasmic reticulum
Correct answers: 2; 4 Answer variants: 5
QUESTION N 144. Enumerate possible causes of cancer development: (4)
Question type: 2. Choosing several possible correct answers
Question value (points): 2
Answer variants:
1. Viral transformation
2. Disturbation of glucose metabolism regulation
3. Elevation of fatty acid input
4. Ionizing radiation exposure
5. Nitrite-containing diet
6. Smoking
7. Carbohydrate-rich diet
Correct answers: 1; 4; 5; 6 Answer variants: 7
QUESTION N 145. Which of following enzymes play important role in ethanol metabolism (3)
Question type: 2. Choosing several possible correct answers
Question value (points): 2
Answer variants:
1. Alanine aminotransferase
2. Aldehyde dehydrogenase
3. Succinate dehydrogenase
4. Thiolase
5. Alcohol dehydrogenase
6. Glutathione S-transferase
7. Microsomal monooxygenase
8. Carbonyl reductase
Correct answers: 2; 5; 7Answer variants: 8
QUESTION N 146. The N atoms at the positions 3 and 9 of purine base are derived from the amide nitrogen of:
Question type: 1. Choosing the only correct answer
Question value (points): 1
Answer variants:
1. Glutamate
2. Glutamine
3. Asparagine
4. Aspartate
Correct answer: 2 Answer variants: 4
QUESTION N 147. The carbon atoms at the positions 4 and 5 and the N atom at the positions 7 of purine base
are supplied from:
Question type: 1. Choosing the only correct answer
Question value (points): 1
Answer variants:
1. Valine
2. Alanine
3. Glycine
4. Serine
Correct answer: 3 Answer variants: 4
QUESTION N 148. The carbon atoms at the positions 4, 5 and 6 and the N atom at the positions 3 of pyrimidine
base are derived from:
Question type: 1. Choosing the only correct answer
Question value (points): 1
Answer variants:
1. Glutamic acid
2. Aspartic acid
3. Glycine
4. Serine
Correct answer: 2 Answer variants: 4
QUESTION N 149. The synthesis of pyrimidine starts with the formation of carbamoyl phosphate from ..., ATP
and CO2 being catalyzed by carbamoyl phosphate synthetase present in the cytosol of the cell.
Question type: 5. Entering the answer manually using the keyboard
Question value (points): 1
Answer variants:
1. Glutamine
2. glutamine
3. Gln
4. gln
All answers are correct Answer variants: 4
QUESTION N 150. Dihydroorotic acid on dehydrogenetion by dihydroorotate dehydrogenase utilizing NAD+ as
coenzyme is converted into ...
Question type: 5. Entering the answer manually using the keyboard
Question value (points): 1
Answer variants:
1. Orotic acid
2. orotic acid
3. orotate
4. Orotate
All answers are correct Answer variants: 4
QUESTION N 151. The end product of purine catabolism in other mammals except man:
Question type: 1. Choosing the only correct answer
Question value (points): 1
Answer variants:
1. Uric acid
2. Allantoin
3. Ammonia
4. Creatinine
Correct answer: 2 Answer variants: 4
QUESTION N 152. The net excretion of total uric acid in normal men in mg in 24 hours:
Question type: 1. Choosing the only correct answer
Question value (points): 1
Answer variants:
1. 100-300
2. 200-400
3. 300-500
4. 400-600
Correct answer: 4 Answer variants: 4
QUESTION N 153. Xanthine is oxidized to uric acid by...
Question type: 5. Entering the answer manually using the keyboard
Question value (points): 1
Answer variants:
1. Xanthine oxidase
2. xanthine oxidase
All answers are correct Answer variants: 2
QUESTION N 154. Xanthine oxidase is inhibited by ...for which uric acid can not be formed.
Question type: 5. Entering the answer manually using the keyboard
Question value (points): 1
Answer variants:
1. Allopurinol
2. allopurinol
All answers are correct Answer variants: 2
QUESTION N 155. The chief end product of purine catabolism in man ...
Question type: 5. Entering the answer manually using the keyboard
Question value (points): 1
Answer variants:
1. uric acid
2. Uric acid
All answers are correct Answer variants: 2
QUESTION N 156. In bacterial operons that are inducible
Question type: 1. Choosing the only correct answer
Question value (points): 1
Answer variants:
1. the inducer binds to the repressor and activates it
2. the inducer stimulates binding of RNA polymerase to the promoter
3. a regulatory gene produces an inactive repressor
4. structural genes that are adjacent on the DNA are coordinately expressed in response to the inducer
5. each of the structural genes produces a separate mRNA
Correct answer: 4 Answer variants: 5
QUESTION N 157. When cyclic adenosine monophosphate (cAMP) levels are relatively high in E.coli
Question type: 1. Choosing the only correct answer
Question value (points): 1
Answer variants:
1. lactose is not required for transcription of the lac operon
2. glucose levels in the medium are low
3. the repressor is bound to the lac operon if lactose is present
4. the enzymes for the metabolism of lactose are not induced
Correct answer: 2 Answer variants: 4
QUESTION N 158. Which of the following statements about regulation of protein synthesis in eukaryotes is
correct?
Question type: 1. Choosing the only correct answer
Question value (points): 1
Answer variants:
1. A gene that is methylated is transcribed more readily than one that is not methylated
2. Genes cannot undergo rearrangements that allow cells to produce new proteins
3. Red blood cells do not produce hemoglobin mRNA because they lack the appropriate gene
4. Recognition of alternative polyadenylation sites allows cells to produce proteins that have different N-terminal
regions
5. Steroid hormones activate genes that are specifically repressed by histones
Correct answer: 3 Answer variants: 5
QUESTION N 159. Select the most appropriate RNA of eukaryotes. Contains introns and a cap at the 5' end
Question type: 1. Choosing the only correct answer
Question value (points): 1
Answer variants:
1. mRNA
2. hnRNA
3. rRNA
4. tRNA
Correct answer: 2 Answer variants: 4
QUESTION N 160. Select the most appropriate RNA of eukaryotes. Contains no introns but has a poly (A) tail
Question type: 1. Choosing the only correct answer
Question value (points): 1
Answer variants:
1. mRNA
2. hnRNA
3. rRNA
4. tRNA
Correct answer: 1 Answer variants: 4
QUESTION N 161. Select the most appropriate RNA of eukaryotes. Does not travel from the nucleus to the
cytoplasm
Question type: 1. Choosing the only correct answer
Question value (points): 1
Answer variants:
1. mRNA
2. hnRNA
3. rRNA
4. tRNA
Correct answer: 2 Answer variants: 4
QUESTION N 162. Select the most appropriate RNA of eukaryotes. Is produced by RNA polymerase I
Question type: 1. Choosing the only correct answer
Question value (points): 1
Answer variants:
1. mRNA
2. hnRNA
3. rRNA
4. tRNA
Correct answer: 3 Answer variants: 4
QUESTION N 163. Choose one of the following statements, which best describes an operon:
Question type: 1. Choosing the only correct answer
Question value (points): 1
Answer variants:
1. An unregulated gene system
2. A coordinatedly regulated gene system
3. A gene that produces mRNA coding one protein
4. A constitutively expressed gene system
5. The region of DNA to which a repressor binds causing the inhibition of transcription
Correct answer: 2 Answer variants: 5
QUESTION N 164. Indicate the order of changes that take place on regulation of trp operon:
Question type: 3. Arranging the answers in the correct sequence
Question value (points): 1
Answer variants:
1. When the concentration of trp is very low, represser is inactive
2. Trp binds to the represser and it becomes active
3. Trp- represser complex binds to the operator
4. RNA polymerase cannot bind to the promoter region
5. Transcription of the structural genes does not occur
All answers are correct Answer variants: 5
QUESTION N 165. Compare the regulation of lac and trp operons. Match the figures with the letters:
Question type: 4. Arranging the correspondences for the answers
Question value (points): 1
Answer variants:
1. lac-operon
2. trp-operon
3. Both
4. None
Answer variants: 4
Correspondence variants:
A. Repressers control operons by inhibiting the binding of RNA polymerase
B. When the concentration of lactose is high, the repressor binds to the operator
C. When the concentration of trp is high, represser- trp complex binds to the operator
D. When lactose is absent, transcription of the structural genes does not occur
Correct answers: 1-D; 2-C; 3-A; 4-B Correspondence variants: 4
QUESTION N 166. All of the following statements about immunoglobulin genes are true EXCEPT
Question type: 1. Choosing the only correct answer
Question value (points): 1
Answer variants:
1. Ig genes are arranged in families, with each family consisting of a complex of genes resident on a different
chromosome
2. Memory B cells are formed by the rearrangement of the DNA sequences coding for the variable domains of the
light
and heavy chains
3. The DNA sequences coding for the variable regions of the Ig chains are located far away from the sequences
coding for the rest of the Ig molecule
4. Maturation of Ig's involves the translocation and rearrangement of Ig genes
5. The constant region of Ig heavy chains is coded for by genes that are unique for each class of Ig
Correct answer: 5 Answer variants: 5
QUESTION N 167. Choose the correct statements. Enhancer: (3)
Question type: 2. Choosing several possible correct answers
Question value (points): 1
Answer variants:
1. Is a segment of DNA
2. Is required for the transcription of gene at a basal level
3. Activates transcription of gene irrespective of their relative position in DNA
4. Has no polarity
5. Is nonspecific for tissue or species
Correct answers: 1; 3; 4Answer variants: 5
QUESTION N 168. Choose INCORRECT answer.The regulation of gene expression in eukaryotic cells may be
accomplished by:
Question type: 1. Choosing the only correct answer
Question value (points): 1
Answer variants:
1. The binding of trans-elements to the response -elements within the DNA sequence
2. Alternative splicing
3. Gene amplification
4. Condensation of chromatin
5. Activation of sets of genes (operon) by a single inducer
Correct answer: 5 Answer variants: 5
QUESTION N 169. Select the appropriate characteristics for different kinds of mutations:
Question type: 4. Arranging the correspondences for the answers
Question value (points): 1
Answer variants:
1. Missens-mutation
2. Nonsens-mutation
3. Insertion with frame-shift
5. Silence mutation
Answer variants: 4
Correspondence variants:
A. Synthesized protein has one amino acid alteration
B. Protein coded by the mutated gene is shorter than normal for some amino acids
C. Synthesized protein has accidental amino acid sequence in the part that codes after the point mutation
D. Synthesized protein has the same amino acid sequense
Correct answers: 1-A; 2-B; 3-C; 5-D Correspondence variants: 4
QUESTION N 170. Choose the correct answers. Causes of protein polymorphisms are:
Question type: 2. Choosing several possible correct answers
Question value (points): 1
Answer variants:
2. Gene amplification
3. Genetic variation
4. Recombinations
5. Increase in the activity of DNA polymerase alpha and beta
Correct answers: 3; 4 Answer variants: 4
QUESTION N 171. Regulation of enzymes by a phosphorylation mechanism:
Question type: 2. Choosing several possible correct answers
Question value (points): 1
Answer variants:
1. Always results in increased activity of the phosphorylated enzyme.
2. Always results in decreased activity of the phosphorylated enzyme.
3. Is only a cAMP-mediated process.
4. Involves formation of high energy bond between phosphate and enzyme
5. Involves a phosphoprotein phosphatase.
Correct answers: 3; 5 Answer variants: 5
QUESTION N 172. Choose the correct answer: Hereditary glucose-6-phosphatase deficiency can lead to:
Question type: 1. Choosing the only correct answer
Question value (points): 1
Answer variants:
1. glycogen synthesis after a meal
2. glycogen is degraded and glucose enters the blood during fasting
Correct answer: 1 Answer variants: 2
QUESTION N 173. The enzyme that interconverts UDP-galactose and UDP-glucose is called an epimerase.
This name is appropriate because glucose and galactose are epimers, which means that they are
Question type: 1. Choosing the only correct answer
Question value (points): 1
Answer variants:
1. mirror images of each other
2. ketoses rather than aldolases
3. hexoses of the L configuration
4. monosaccharides that differ only in the position of one hydroxyl group
5. disaccharides that contain a beta-1,4-glycosidic bond
Correct answer: 4 Answer variants: 5
QUESTION N 174. Which of the following statements concerning glycosaminoglycans is true?
Question type: 1. Choosing the only correct answer
Question value (points): 1
Answer variants:
1. They contain repeating disaccharides
2. They are usually positively charged
3. They contain short oligosaccharide chains
4. They rarely contain sulfate groups
5. They contain branches of N-acetylneuraminic acid
Correct answer: 1 Answer variants: 5
QUESTION N 175. Which of the following statements concerning glycoproteins is true?
Question type: 1. Choosing the only correct answer
Question value (points): 1
Answer variants:
1. They are usually positively charged
2. They never contain branched oligosaccharide chains
3. They contain oligosaccharides that are synthesized on dolichol phosphate and transferred to serine residues
4. They are degraded by lysosomal enzymes
5. They are all secreted into the blood
Correct answer: 4 Answer variants: 5
QUESTION N 176. The mucopolysaccharidoses are caused by deficiencies of enzymes involved in the
degradation of
Question type: 1. Choosing the only correct answer
Question value (points): 1
Answer variants:
1. fructose
2. galactose
3. glycosaminoglycans
4. glycoproteins
5. glycogen
Correct answer: 3 Answer variants: 5
QUESTION N 177. After digestion of a piece of a cake that contains flour, milk, and sucrose as its primary
ingredients, the major carbohydrate products entering the blood are
Question type: 1. Choosing the only correct answer
Question value (points): 1
Answer variants:
1. glucose
2. fructose and galactose
3. galactose and glucose
4. fructose and glucose
5. glucose, fructose, and galactose
Correct answer: 5 Answer variants: 5
QUESTION N 178. The degradation of glycogen normally produces
Question type: 1. Choosing the only correct answer
Question value (points): 1
Answer variants:
1. more glucose than glucose 1-phosphate
2. more glucose 1-phosphate than glucose
3. equal amounts of glucose and glucose 1-phosphate
4. neither glucose nor glucose 1-phosphate
5. only glucose 1-phosphate
Correct answer: 2 Answer variants: 5
QUESTION N 179. Which of the following statements about liver phosphorylase kinase is true?
Question type: 1. Choosing the only correct answer
Question value (points): 1
Answer variants:
1. It is present in an inactive form when epinephrine is elevated
2. It phosphorylates phosphorylase to an inactive form
3. It catalyzes a reaction that requires ATP
4. It is phosphorylated in response to elevated insulin
5. It is not affected by cAMP
Correct answer: 2 Answer variants: 5
QUESTION N 180. A patient had large deposits of liver glycogen, which, after an overnight fast, had shorter
than normal branches. This abnormality could be caused by a defective
Question type: 1. Choosing the only correct answer
Question value (points): 1
Answer variants:
1. glycogen phosphorylase
2. glucagon receptor
3. glycogenin
4. amylo-1,6-glucosidase (alpha-glucosidase)
5. amylo-4,6-transferase (4:6 transferase)
Correct answer: 4 Answer variants: 5
QUESTION N 181. Choose the correct answers. HbA and HbS:
Question type: 2. Choosing several possible correct answers
Question value (points): 1
Answer variants:
1. Are the products of the allel genes
2. Have similar spatial structure
3. Interact with the same ligand
4. Have many differences in amino acid composition
5. Are the result of missens mutation
Correct answers: 1; 3; 5Answer variants: 5
QUESTION N 182. Choose the one best answer.What the major function of the 92o temperature in PCR
reaction?
Question type: 1. Choosing the only correct answer
Question value (points): 1
Answer variants:
1. It is the optimal temperature for DNA polymerization
2. To increase the specificity of the binding of the primers
3. To denature the DNA strands
4. To drive the polymerase reaction to completion
5. To reanneal newly synthesized DNA strands
Correct answer: 3 Answer variants: 5
QUESTION N 183. Blood uric acid level also ... in hemolytic anemia and thalassemia.
Question type: 5. Entering the answer manually using the keyboard
Question value (points): 1
Answer variants:
1. increases
2. increase
3. elevates
4. elevate
All answers are correct Answer variants: 4
QUESTION N 184. Sodium urate crystals are deposited in the soft tissues and these urate deposits are
re-ferred to as ...
Question type: 5. Entering the answer manually using the keyboard
Question value (points): 1
Answer variants:
1. tophi
All answers are correct Answer variants: 1
QUESTION N 185. The main site of the catabolism of pyrimidines ...
Question type: 5. Entering the answer manually using the keyboard
Question value (points): 1
Answer variants:
1. Liver
2. liver
All answers are correct Answer variants: 2
QUESTION N 186. Match the formula of the dinucleotide A-U and compare the composition of its constituent
nucleotides:
Question type: 4. Arranging the correspondences for the answers
Question value (points): 1
Answer variants:
1. Purine base
2. Pyrimidine base
3. Ribose
4. Pyrophosphate
Answer variants: 4
Correspondence variants:
A. UMP
B. AMP
C. Both
D. None
Correct answers: 1-B; 2-A; 3-C; 4-D Correspondence variants: 4
QUESTION N 187. Which one of the following statements is true of the double helix DNA?
Question type: 1. Choosing the only correct answer
Question value (points): 1
Answer variants:
1. The planes of the bases lie parallel to the helix axis.
2. The chains have a backbone of linked glycosides.
3. The 3' -OH groups of each chain are at opposite ends of the molecule.
4. The duplex structure is stabilized only by hydrogen bonds between bases.
5. The two chains have an identical base sequence.
Correct answer: 3 Answer variants: 5
QUESTION N 188. Match the following linkages with the appropriate letters in figure
Question type: 4. Arranging the correspondences for the answers
Question value (points): 1
Answer variants:
1. A
2. B
3. C
4. D
Answer variants: 4
Correspondence variants:
A. N-glycoside
B. 3', 5'- phosphodiester
C. 5'-phosphoester
D. 3'- phosphoester
Correct answers: 1-C; 2-A; 3-D; 4-B Correspondence variants: 4
QUESTION N 189. Excessive degradation of AMP and GMP would result in increased urinary excretion of
Question type: 1. Choosing the only correct answer
Question value (points): 1
Answer variants:
1. creatinine
2. urea
3. uric acid
4. thiamine
5. thymine
Correct answer: 3 Answer variants: 5
QUESTION N 190. De novo pyrimidine synthesis requires
Question type: 1. Choosing the only correct answer
Question value (points): 1
Answer variants:
1. phosphoribosyl pyrophosphate (PRPP) for the initial step
2. tetrahydrofolate for the incorporation of carbons 2 and 8
3. both carbon and nitrogen of aspartate to form the ring
4. NH4+ as a substrate for carbamoyl phosphate synthethase II
5. glycine as the source of two nitrogens in the ring
Correct answer: 3 Answer variants: 5
QUESTION N 191. 5-Fluorouracil (5-FU) is an effective chemotherapeutic agent because it
Question type: 1. Choosing the only correct answer
Question value (points): 1
Answer variants:
1. FH2 -> FH4
2. dUMP -> dTMP
3. glutamine + PRPP -> phosphoribosylamine
4. methyl B12 -> B12
Correct answer: 2 Answer variants: 4
QUESTION N 192. Match each condition below with the component that would be elevated
Question type: 4. Arranging the correspondences for the answers
Question value (points): 1
Answer variants:
1. Gout
2. Myocardial infarction
3. Hepatitis
4. Kidney disease
Answer variants: 4
Correspondence variants:
A. Bilirubin
B. Uric acid
C. Creatine kinase
D. Blood urea nitrogen (BUN)
Correct answers: 1-B; 2-C; 3-A; 4-D Correspondence variants: 4
QUESTION N 193. Match each description below with the appropriate condition.
Question type: 4. Arranging the correspondences for the answers
Question value (points): 1
Answer variants:
1. Can be caused by a dietary tryptophan deficiency
2. Caused by a genetic defect in hypoxanthine guanine phosphoribosyl-transferase (HGPRT)
3. Caused by a defective transporter on intestinal epithelial cells for essential amino acids
4. Caused by a defective transporter on kidney tubule cells for a nonessential amino acid
5. Caused by decreased production of dopamine from dopa
Answer variants: 5
Correspondence variants:
A. Parkinson's disease
B. Cystinuria
C. Pellagra
D. Lesch-Nyhan syndrome
E. Hartnup's disease
Correct answers: 1-C; 2-D; 3-E; 4-B; 5-A Correspondence variants: 5
QUESTION N 194. All of the following statements regarding nucleosomes are true EXCEPT
Question type: 1. Choosing the only correct answer
Question value (points): 1
Answer variants:
1. The bead-like structures are made of core histones around which DNA is wrapped.
2. The bead-like structures are linked by a DNA chain.
3. Each bead-like structure contains two molecules of histones: HI, H2a, H2b, H3, H4.
4. Approximately 140 base pairs of DNA are wound around each nucleosome.
5. The primary structure of the histones is similar in all cell types and in all eucaryotes.
Correct answer: 3 Answer variants: 5
QUESTION N 195. All of the following statements about RNA are true EXCEPT
Question type: 1. Choosing the only correct answer
Question value (points): 1
Answer variants:
1. The sugar group of ribonucleotides has an OH group at the 2? position.
2. In RNA helices, A can base pair with U through two hydrogen bonds.
3. In RNA helices, G can base pair with C through three hydrogen bonds.
4. The mole fraction of A equals the mole fraction of U.
5. Ribonucleotides are linked by phosphodiester bonds between the 3'-OH on the sugar of one ribonucleotide
through
a phosphate to the 5'-OH on the sugar of another ribonucleo-tide.
Correct answer: 4 Answer variants: 5
QUESTION N 196. Match the figures with the letters.
Question type: 4. Arranging the correspondences for the answers
Question value (points): 1
Answer variants:
1. Histones take part in the arrangement of ..... in the cell.
2. The molecules of ..... contain modified nucleotides and an anticodon.
3. The 18S ... molds the 40S subunit of ribosome.
4. The "cap" at the 5'-end of ... plays a role in initiation of translation.
5. contain the CCA sequence whereby they bind with amino acids.
Answer variants: 5
Correspondence variants:
A. DNA
B. mRNA
C. tRNA
D. rRNA
Correct answers: 1-A; 2-C; 3-D; 4-B; 5-C Correspondence variants: 4
QUESTION N 197. Which of the following statements concerning Okazaki fragments is true?
Question type: 1. Choosing the only correct answer
Question value (points): 1
Answer variants:
1. They are produced by restricting nucleases.
2. They are synthesized on the leading strand during replication.
3. They are regions of DNA that do not code for the amino acids in a protein.
4. They are relatively short polydeoxyribonucleotides with a few ribonucleotide residues at the 5' end.
5. They are products of the action of DNA ligase.
Correct answer: 4 Answer variants: 5
QUESTION N 198. Match the figures with the letters:
Question type: 4. Arranging the correspondences for the answers
Question value (points): 1
Answer variants:
1. dNTPs incorporation into the polynucleotide chain.
2. Elimination of DNA errors.
3. Synthesis of new DNA molecules.
4. Synthesis of templates for translation.
Answer variants: 4
Correspondence variants:
A. Replication
B. Repair
C. Both
D. None
Correct answers: 1-C; 2-B; 3-A; 4-D Correspondence variants: 4
QUESTION N 199. In DNA, on a molar basis
Question type: 1. Choosing the only correct answer
Question value (points): 1
Answer variants:
1. adenine equals thymine
2. adenine equals uracil
3. guanine equals adenine
4. cytosine equals thymine
5. cytosine equals uracil
Correct answer: 1 Answer variants: 5
QUESTION N 200. Which of the following sequences is complementary to the DNA sequence 5'-AAGTCCGA-
3'?
Question type: 1. Choosing the only correct answer
Question value (points): 1
Answer variants:
1. 5'-AAGUCCGA-3'
2. 3'-TTCAGGCT-5'
3. 5'-TTCAGGCT-3'
4. 3'-TCGGACTT-5'
Correct answer: 2 Answer variants: 4
QUESTION N 201. Choose the correct answer. Plasma fraction of blood in volume per cent is
Question type: 1. Choosing the only correct answer
Question value (points): 1
Answer variants:
1. 40
2. 45
3. 50
4. 55
Correct answer: 4 Answer variants: 4
QUESTION N 202. DNA contains which one of the following components?
Question type: 1. Choosing the only correct answer
Question value (points): 1
Answer variants:
1. Nitrogenous bases joined by phosphodiester bonds
2. Negatively charged phosphate groups in the interior of the molecule
3. Base pairs stacked along the central axis of the molecule
4. Two strands that run in the same direction
5. The sugar ribose
Correct answer: 3 Answer variants: 5
QUESTION N 203. Which RNA contains 7-methylguanine at the 5' end?
Question type: 1. Choosing the only correct answer
Question value (points): 1
Answer variants:
1. 5S RNA
2. rRNA
3. hnRNA
4. tRNA
Correct answer: 3 Answer variants: 4
QUESTION N 204. Thymine is present in which type of RNA?
Question type: 1. Choosing the only correct answer
Question value (points): 1
Answer variants:
1. mRNA
2. rRNA
3. hnRNA
4. tRNA
Correct answer: 4 Answer variants: 4
QUESTION N 205. The action of DNA polymerases requires
Question type: 1. Choosing the only correct answer
Question value (points): 1
Answer variants:
1. a 5'-hydroxyl group
2. dUTP
3. NAD+ as a cofactor
4. a 3'-hydroxyl group
5. CTP
Correct answer: 4 Answer variants: 5
QUESTION N 206. Which of the following statements concerning replication of DNA is TRUE?
Question type: 1. Choosing the only correct answer
Question value (points): 1
Answer variants:
1. It progresses in both directions away from each point of origin on the chromosome
2. It requires a DNA template that is copied in its 5' to 3' direction
3. It occurs during the M phase of the cell cycle
4. It produces one newly synthesized double helix and one composed of the two parental strands
Correct answer: 1 Answer variants: 4
QUESTION N 207. When base-pairing occurs in loops of RNA, adenine is hydrogen-bonded to
Question type: 1. Choosing the only correct answer
Question value (points): 1
Answer variants:
1. guanine
2. thymine
3. cytosine
4. uracil
Correct answer: 4 Answer variants: 4
QUESTION N 208. Which of the following statements concerning Okazaki fragments is TRUE?
Question type: 1. Choosing the only correct answer
Question value (points): 1
Answer variants:
1. They are porduced by restriction enzymes
2. They are synthesized on the leading strand during replication
3. They are regions of DNA that do not code for the amino acids in a protein
4. They are relatively short polydeoxyribonucleotides with a few ribonucleotide residues at the 5' end
5. They are products of the action of RNase on hnRNA
Correct answer: 4 Answer variants: 5
QUESTION N 209. A bacterial mutant grows normally at 32C but at 42C accumulates short segments of newly
synthesized DNA. Which of the following enzymes is most likely to be defective in this mutant?
Question type: 1. Choosing the only correct answer
Question value (points): 1
Answer variants:
1. An endonuclease
2. DNA polymerase
3. An exonuclease
4. An unwinding enzyme (helicase)
5. DNA ligase
Correct answer: 5 Answer variants: 5
QUESTION N 210. Which of the following phrases describes nucleosomes?
Question type: 1. Choosing the only correct answer
Question value (points): 1
Answer variants:
1. Single ribosomes attached to mRNA
2. Complexes of DNA and newly-transcribed RNA
3. Subunits of chromatin
4. Structures that contain DNA in the core with histones wrapped around the surface
5. Complexes of protein and the 45S rRNA precursors found in the nucleolus
Correct answer: 3 Answer variants: 5
QUESTION N 211. Match the following statements and properties of the genetic code:
Question type: 4. Arranging the correspondences for the answers
Question value (points): 1
Answer variants:
1. Degeneracy
2. Unambiguous
3. Universality
Answer variants: 3
Correspondence variants:
A. A given codon designates only one amino acid
B. The genetic code is the same for all examined species of plants, animals and people
C. More than 1 codon can specify the same amino acid.
Correct answers: 1-C; 2-A; 3-B Correspondence variants: 3
QUESTION N 212. Choose the correct answer. Degeneracy of the genetic code means that:
Question type: 1. Choosing the only correct answer
Question value (points): 1
Answer variants:
1. A given base triplet can code for more than one amino acid
2. There is no punctuation in the code sequences
3. The third base in a codon is not important in coding
4. Each amino acid can be coded for by more than one base triplet
5. Codons are not ambiguous
Correct answer: 4 Answer variants: 5
QUESTION N 213. The following statements about RNA polymerases are true EXCEPT
Question type: 2. Choosing several possible correct answers
Question value (points): 1
Answer variants:
1. Initiate the synthesis of new RNA chains.
2. Copy DNA template in the 3' to 5'direction.
3. Perform the splicing of the precursors of mRNA, rRNA and tRNA
4. Associate with promoters of genes.
5. All human RNAs are synthesized by a single RNA polymerase.
Correct answers: 3; 5 Answer variants: 5
QUESTION N 214. Choose the MOST COMPLETE answer: during the processing:
Question type: 1. Choosing the only correct answer
Question value (points): 1
Answer variants:
1. The "cap" is bonded with the 5'-end of mRNA precursors.
2. RNA precursors turn into mature molecules that are involved in protein synthesis.
3. Intrones are excised.
4. Some nitrogen bases undergo methylation.
5. The poly (A) fragment is added to the 3'-end of mRNA.
Correct answer: 2 Answer variants: 5
QUESTION N 215. Choose the correct answer. The diffusible constituent of plasma:
Question type: 1. Choosing the only correct answer
Question value (points): 1
Answer variants:
1. Urea
2. Vitamins
3. Hormones
4. All of the above
Correct answer: 4 Answer variants: 4
QUESTION N 216. Choose the correct answer. The catabolic products of diffusible constituent of plasma:
Question type: 1. Choosing the only correct answer
Question value (points): 1
Answer variants:
1. Uric acid
2. Creatinine
3. Both of the above
4. None of the above
Correct answer: 3 Answer variants: 4
QUESTION N 217. Which of the following statements (3) about inhibitors of protein synthesis is correct
Question type: 2. Choosing several possible correct answers
Question value (points): 1
Answer variants:
1. Erythromycin inhibits DNA synthesis causing chromosomal breaks.
2. Diphtheria toxin inactivates the eukaryotic elongation factor eEF-2.
3. a-Amanitin inhibits eukaryotic RNA polymerases.
4. Doxorubicin binds to the 30S subunit and distorts its structure.
5. Bleomycin inhibits DNA synthesis causing fragmentation of molecule.
Correct answers: 2; 3; 5Answer variants: 5
QUESTION N 218. Eukaryotic genes that produce mRNA
Question type: 1. Choosing the only correct answer
Question value (points): 1
Answer variants:
1. contain a TATA box downstream from the start site of transcription
2. can contain a CAAT box in the 5' flanking region
3. are transcribed by RNA polymerase III
4. contain long stretches of thymine nucleotides that produce the poly(A) tail of mRNA
5. do not contain introns
Correct answer: 2 Answer variants: 5
QUESTION N 219. If a fragment of DNA containing the sequence 5'-AGCCAATT-3' serves as the template for
transcription, the RNA that is produced will have the sequence
Question type: 1. Choosing the only correct answer
Question value (points): 1
Answer variants:
1. 5'-AGCCAAUU-3'
2. 5'-UCGGUUAA-3'
3. 5'-UUAACCGA-3'
4. 5'-AAUUGGCU-3'
Correct answer: 4 Answer variants: 4
QUESTION N 220. A person ate mushrooms picked in a wooded area. Shortly thereafter, he was rushed to the
hospital, where he died. He had no previous medical problems. The cause of his death was most likely the
RNA polymerase inhibitor
Question type: 1. Choosing the only correct answer
Question value (points): 1
Answer variants:
1. rifampicin
2. alpha-amanitin
3. streptolydigin
4. actinomycin
Correct answer: 2 Answer variants: 4
QUESTION N 221. An aminoacyl-tRNA exhibits which one of the following characteristics?
Question type: 1. Choosing the only correct answer
Question value (points): 1
Answer variants:
1. It is produced by a synthethase that is specific for the amino acid, but not the tRNA
2. It is composed of an amino acid esterified tj the 5' end of the tRNA
3. It requires GTP for its synthesis from an amino acid and a tRNA
4. It contains an anticodon that is complementary to the codon for the amino acid
Correct answer: 4 Answer variants: 4
QUESTION N 222. Which of the following statements about methionine is TRUE?
Question type: 1. Choosing the only correct answer
Question value (points): 1
Answer variants:
1. It is the amino acid used for initiation of the synthesis of all proteins
2. It is generally found at the N-terminus of proteins isolated from cells
3. It requires a codon other than AUG to be added to growing polypeptide chains
4. It is formylated when it is bound to tRNA in eukaryotic cells
Correct answer: 1 Answer variants: 4
QUESTION N 223. Which of the following statements about bacteria is correct?
Question type: 1. Choosing the only correct answer
Question value (points): 1
Answer variants:
1. They contain 80S ribosomes
2. They initiate protein synthesis with methionyl-tRNA
3. They are insensitive to chloramphenicol
4. They synthesize proteins on mRNA that is in the process of being transcribed
Correct answer: 4 Answer variants: 4
QUESTION N 224. Which of the following is required for initiation of protein synthesis in the cytosol of
eukaryotic cells?
Question type: 1. Choosing the only correct answer
Question value (points): 1
Answer variants:
1. A 40S ribosomal subunit
2. IF-2
3. Methionyl-tRNA(Met,f)
4. UTP
5. EF-2
Correct answer: 1 Answer variants: 5
QUESTION N 225. Which of the following is required for the elongation reactions of protein synthesis in
eukaryotes?
Question type: 1. Choosing the only correct answer
Question value (points): 1
Answer variants:
1. Peptidyl transferase
2. CTP
3. Formylmethionyl-tRNA
4. Elongation factor G (EF-G)
5. hnRNA
Correct answer: 1 Answer variants: 5
QUESTION N 226. The mechanism for termination of protein synthesis in eukaryotes requires
Question type: 1. Choosing the only correct answer
Question value (points): 1
Answer variants:
1. a peptidyl-tRNA that cannot bind at the P site
2. the codon UGA, UAG, or AUG in the A site
3. nuclease cleavage of mRNA
4. release factors
Correct answer: 4 Answer variants: 4
QUESTION N 227. Proteins that are secreted form the cells
Question type: 1. Choosing the only correct answer
Question value (points): 1
Answer variants:
1. contain methionine as the N-terminal amino acid
2. are produced from translation products that have a signal sequence at the C-terminal end
3. are synthesized on ribosomes that bind to proteins on the endoplasmic reticulum
4. contain a hydrophobic sequence that is embedded in the membrane os secretory vesicles
5. contain carbohydrate residues that bind to receptors on the interior of lysosomal membranes
Correct answer: 3 Answer variants: 5
QUESTION N 228. Tetracycline, streptomycin, and erythromycin are effective antibiotics because they inhibit
Question type: 1. Choosing the only correct answer
Question value (points): 1
Answer variants:
1. RNA synthesis in prokaryotes
2. RNA synthesis in eukaryotes
3. protein synthesis in prokaryotes
4. protein synthesis on cytosolic ribosomes of eukaryotes
5. protein synthesis on mitochondrial ribosomes of eukaryotes
Correct answer: 3 Answer variants: 5
QUESTION N 229. When this in vitro protein-synthesizing system was treated with the new antibiotic, the only
product was methionyl-phenylalanyl-tRNA. What process in protein synthesis is inhibited by the antibiotic?
Question type: 1. Choosing the only correct answer
Question value (points): 1
Answer variants:
1. Binding of an aminoacyl-tRNA to the A site of the ribosome
2. Initiation
3. Translocation
4. Peptidyl transferase catalyzed formation of a peptide bond
Correct answer: 3 Answer variants: 4
QUESTION N 230. Match the figures with the letters. Glycolysis supplies erythrocyte ________for
________________
Question type: 4. Arranging the correspondences for the answers
Question value (points): 1
Answer variants:
1. 2,3-diphosphoglycerate
2. ATP
3. NADH
Answer variants: 3
Correspondence variants:
A. Regeneration Hb from metHb
B. Work of ion pump
C. Regulation of affinity of hemoglobin for oxygen
Correct answers: 1-C; 2-B; 3-A Correspondence variants: 3
QUESTION N 231. Match the number and the letters. The nature of antigenic determinants on the surface of
the RBC:
Question type: 4. Arranging the correspondences for the answers
Question value (points): 1
Answer variants:
1. Type A
2. Type B
3. Type AB
4. Type O
Answer variants: 4
Correspondence variants:
A. Galactose at the nonreducing end
B. N-acetylgalactosamine
C. None of the above
D. All of the above
Correct answers: 1-B; 2-A; 3-D; 4-C Correspondence variants: 4
QUESTION N 232. Choose the correct couple. Coenzyme of glutation reductase is ___________,which is
formed in_________
Question type: 1. Choosing the only correct answer
Question value (points): 1
Answer variants:
1. NADH;Glycolysis
2. NADPH; Pentose phosphate pathway
3. NAD; Methemoglobin reductase reaction
4. NADPH; Methemoglobin reductase reaction
5. NADH; Pentose phosphate pathway
Correct answer: 2 Answer variants: 5
QUESTION N 233. Choose the correct statement. D-glucose 6-phosphate dehydrogenase protects
erythrocyte
from hemolysis because it:
Question type: 1. Choosing the only correct answer
Question value (points): 1
Answer variants:
1. Produces ATP, that is necessary for ion pump work
2. Provides coenzyme for glutathione peroxidase
3. Provides coenzyme for glutathione reductase
4. Provides coenzyme for methemoglobin reductase
Correct answer: 3 Answer variants: 4
QUESTION N 234. Indicate True or False of the followings :
Question type: 4. Arranging the correspondences for the answers
Question value (points): 1
Answer variants:
1. 80 per cent of the red blood cell copper is presented as superoxide dismutase.
2. Erythrocytes contain enzymes of TCA cycle
3. Erythrocytes contain enzyme catalase
4. Erythrocytes contain enzyme D-glucose6-phosphate dehydrogenase
5. Erythrocytes persist about 120 days
6. Erythrocytes contain enzyme methemoglobin reductase
7. Erythrocytes contain enzymes of beta-oxidation of fatty acid
Answer variants: 7
Correspondence variants:
A. true
B. false
Correct answers: 1-A; 2-B; 3-A; 4-A; 5-A; 6-A; 7-B Correspondence variants: 2
QUESTION N 235. Fill up the blanks of the following. Carbonic anhydrase catalyzes the formation of H2CO3
from CO2 and __________
Question type: 5. Entering the answer manually using the keyboard
Question value (points): 1
Answer variants:
1. Water
2. water
3. H2O
4. h2o
All answers are correct Answer variants: 4
QUESTION N 236. Fill up the blanks of the following. The carbonic acid is then buffered by the intracellular
buffers (phosphate and hemoglobin) combining with ______________
Question type: 5. Entering the answer manually using the keyboard
Question value (points): 1
Answer variants:
1. Potassium
2. potassium
All answers are correct Answer variants: 2
QUESTION N 237. Fill up the blanks of the following. When the CO2 tension is reduced, __________leaves the
cell and enters the plasma.
Question type: 5. Entering the answer manually using the keyboard
Question value (points): 1
Answer variants:
1. Chloride
2. chloride
3. Cl-
4. cl-
5. Cl
All answers are correct Answer variants: 5
QUESTION N 238. Fill up the blanks of the following. Fetal hemoglobin takes up ______ more readily at low
oxygen tension and releases CO2 more readily than adult hemoglobin(A).
Question type: 5. Entering the answer manually using the keyboard
Question value (points): 1
Answer variants:
1. O2
2. oxygen
3. Oxygen
All answers are correct Answer variants: 3
QUESTION N 239. Fill up the blanks of the following. Hemoglobin F is gradually replaced by _________ during
the first 6 months of ex-trauterine life.
Question type: 5. Entering the answer manually using the keyboard
Question value (points): 1
Answer variants:
1. HbA
2. Hemoglobin A
3. hemoglobin A
All answers are correct Answer variants: 3
QUESTION N 240. Fill up the blanks of the following. Hb F is more resistant to denaturation by Alkali and is
more susceptible to conversion to methemoglobin by ________
Question type: 5. Entering the answer manually using the keyboard
Question value (points): 1
Answer variants:
1. nitrites
2. Nitrites
All answers are correct Answer variants: 2
QUESTION N 241. Fill up the blanks of the following. Some of the abnormal hemoglobins are easily
differentiated by their electrophoretic mobilities and have given rise to the concept of _______.
Question type: 5. Entering the answer manually using the keyboard
Question value (points): 1
Answer variants:
1. Molecular diseases
2. molecular diseases
All answers are correct Answer variants: 2
QUESTION N 242. Fill up the blanks of the following. Methemoglobin is present in normal erythrocytes about
_____ per cent of the total hemoglobin.
Question type: 5. Entering the answer manually using the keyboard
Question value (points): 1
Answer variants:
1. 0,4
2. 0.4
All answers are correct Answer variants: 2
QUESTION N 243. Match the letter and the number:
Question type: 4. Arranging the correspondences for the answers
Question value (points): 1
Answer variants:
1. Hemolytic anemia
2. Hypochromic microcytic anemia
3. Megaloblastic anemia
Answer variants: 3
Correspondence variants:
A. Cobalamin deficiency
B. D-glucose 6-phosphate dehydrogenase deficiency
C. Iron deficiency
D. Sickle cell anemia
E. Folate deficiency
F. Pyridoxine deficiency
Correct answers: 1-B, D; 2-C, F; 3-A, E Correspondence variants: 6
QUESTION N 244. Match the letter and the number:
Question type: 4. Arranging the correspondences for the answers
Question value (points): 1
Answer variants:
1. Sickle cell anemia
2. Methemoglobinemia
3. Thalassemia
Answer variants: 3
Correspondence variants:
A. Replacement his in a pocket of heme
B. Large deletions in the beta-globin gene
C. Replacement glu in 6th position of beta-globin
Correct answers: 1-C; 2-A; 3-B Correspondence variants: 3
QUESTION N 245. Choose the statements (4) that correctly characterize tRNAs:
Question type: 2. Choosing several possible correct answers
Question value (points): 1
Answer variants:
1. No template is required for their synthesis.
2. Have a template (mRNA) recognition site anticodon.
3. Have an attachment site for a specific amino acid.
4. Carry amino acids to the ribosomal side of protein synthesis
5. The carboxyl group of amino acids are esterified to the 3'- or 2'-OH group of the 3 termi-nal ribose ofRNA.
Correct answers: 2; 3; 4; 5 Answer variants: 5
QUESTION N 246. Choose the correct statements (2). In protein biosynthesis:
Question type: 2. Choosing several possible correct answers
Question value (points): 1
Answer variants:
1. Each amino acid recognizes its own codon by a direct interaction with the mRNA tem-plate.
2. The formation of peptide bonds by ribosomal messenger RNA complex continues until a stop codon on mRNA is
reached.
3. Peptide bond formation ceases when the ribosome reaches the 5' end of the mRNA.
4. A given codon and its anticodon must have identical base sequences in order for them to be proper base.
5. Each amino acid is added in its proper place to a growing peptide chain through the "adaptor" function of tRNA.
Correct answers: 2; 5 Answer variants: 5
QUESTION N 247. All of following statements about biochemical consequences of anemia are true EXCEPT :
Question type: 1. Choosing the only correct answer
Question value (points): 1
Answer variants:
1. Decrease of synthesis P by anaerobic way
2. Inhibition of oxidation processes by high concentration ND+
3. Production of active forms of oxygen is strengthed and cells are ruined
4. Increase of synthesis P by aerobic way
Correct answer: 4 Answer variants: 4
QUESTION N 248. The development of methemoglobinemia may be due to the following factors:
Question type: 2. Choosing several possible correct answers
Question value (points): 1
Answer variants:
1. Acidic poisoning
2. Lower partial pressure of oxygen
3. Hereditary defect of methemoglobin reductase
4. CO poisoning
5. All of the above
6. None of the above
Correct answers: 1; 3; 4Answer variants: 6
QUESTION N 249. Choose the correct answer. The human plasma proteins are a mixture of
Question type: 1. Choosing the only correct answer
Question value (points): 1
Answer variants:
1. Simple proteins
2. Glycoproteins
3. Lipoproteins
4. All of the above
Correct answer: 4 Answer variants: 4
QUESTION N 250. Choose the correct answer. The plasma proteins are separated by
Question type: 1. Choosing the only correct answer
Question value (points): 1
Answer variants:
1. Salt precipitation
2. Electrophoresis
3. Immunoelectrophoresis
4. All of the above
Correct answer: 4 Answer variants: 4
QUESTION N 251. Choose the correct answer: Retinol is transported to the blood as retinol attached to
Question type: 1. Choosing the only correct answer
Question value (points): 1
Answer variants:
1. alpha 1-globulin
2. alpha 2-globulin
3. beta-globulin
4. gamma-globulin
Correct answer: 1 Answer variants: 4
QUESTION N 252. Indicate True or False of the followings :
Question type: 4. Arranging the correspondences for the answers
Question value (points): 1
Answer variants:
1. alpha 1-globulins are several complex proteins containing carbohydrates and lipids
2. gamma-globulins are formed in the liver
3. In normal individuals, the plasma proteins vary from 60 to 85 g/l
4. Albumin has a molecular weight of about 69 000 and is synthesized in the liver.
5. Albumin plays an important role in the exchange of water between tissue and blood.
6. Bilirubin is associated with gamma-globulins.
7. Albumin is largely involved in the nutritive functions of the plasma proteins owing to its high concentration.
8. Gamma-globulins are immunoglobulins having antibody activity.
Answer variants: 8
Correspondence variants:
A. true
B. false
Correct answers: 1-A; 2-B; 3-A; 4-A; 5-A; 6-B; 7-A; 8-A Correspondence variants: 2
QUESTION N 253. Choose the correct answer: The serum albumin concentration decreases in
Question type: 1. Choosing the only correct answer
Question value (points): 1
Answer variants:
1. Severe protein deficiency
2. Liver diseases
3. Nephritis
4. All of the above
Correct answer: 4 Answer variants: 4
QUESTION N 254. Match the letter and the number:Type of disproteinemia
Question type: 4. Arranging the correspondences for the answers
Question value (points): 1
Answer variants:
1. Decrease of albumin and increase of gamma-globulin fractions
2. Decrease of albumin, alpha 2-globulin, increase of beta-globulin and gamma-globulin
3. Increase of albumin and moderate decrease of all globuline fractions
Answer variants: 3
Correspondence variants:
A. Dehydratation
B. Hepatitis
C. Inflamatory process
Correct answers: 1-C; 2-B; 3-A Correspondence variants: 3
QUESTION N 255. Match the figures with the letters.
Question type: 4. Arranging the correspondences for the answers
Question value (points): 1
Answer variants:
1. Secretary enzyme
2. Excretary enzyme
3. Indicated enzyme
Answer variants: 3
Correspondence variants:
A. Choline esterase
B. Aspartate transaminase
C. Gamma-glutamyl transferase
D. Alanine transaminase
E. Alkaline phosphatase
F. Factor XIII
Correct answers: 1-A, F; 2-C, E; 3-B, D Correspondence variants: 6
QUESTION N 256. Choose the correct answer: Isoenzymes are:
Question type: 1. Choosing the only correct answer
Question value (points): 1
Answer variants:
1. Phisically distinct forms of the same enzyme
2. Catalyse the same reaction
3. Have different mobility on electrophoresis
4. All of the above
Correct answer: 4 Answer variants: 4
QUESTION N 257. Choose the correct answer (3): In the case of liver diseases especially with liver cells
damage, the enzyme of diagnostic interest is:
Question type: 2. Choosing several possible correct answers
Question value (points): 1
Answer variants:
1. Aspartate transaminase
2. Alanine transaminase
3. Creatine kinase
4. Gamma-glutamyl transferase
5. Alkaline phosphatase
Correct answers: 2; 4; 5Answer variants: 5
QUESTION N 258. Choose the correct answer: In the case of myocardial infarction, the enzyme which rises
more slowly is:
Question type: 1. Choosing the only correct answer
Question value (points): 1
Answer variants:
1. Creatine kinase
2. Aspartate transaminase
3. Lactate dehydrogenase
4. All of the above
5. None of the above
Correct answer: 3 Answer variants: 5
QUESTION N 259. Choose the correct answer: In the case of myocardial infarction, the first enzyme to
increase in activity is:
Question type: 1. Choosing the only correct answer
Question value (points): 1
Answer variants:
1. Creatine kinase
2. Aspartate transaminase
3. Lactate dehydrogenase
4. All of the above
5. None of the above
Correct answer: 1 Answer variants: 5
QUESTION N 260. Choose the correct answer: In the case of acute viral hepatits, there is a rapid rise in the
level of:
Question type: 1. Choosing the only correct answer
Question value (points): 1
Answer variants:
1. Alanine transaminase (GOT)
2. Aspartate transaminase (GPT)
3. Serum bilirubin
4. Alkaline phosphatase
5. Acid phosphatase
Correct answer: 1 Answer variants: 5
QUESTION N 261. Match the figures with the letters.
Question type: 4. Arranging the correspondences for the answers
Question value (points): 1
Answer variants:
1. Aspartate transaminase (GPT)
2. Gamma-glutamyl transferase
3. Alkaline phosphatase is elevated
4. Serum CK level is increased
5. Transaminases level is increased
6. LDH is liberated to the blood stream
Answer variants: 6
Correspondence variants:
A. Has cytosol and mitochondrial forms
B. In acute viral hepatitis
C. Is induced by ethanol
D. During myocardial injury
E. In viral and alcoholic hepatitis
F. In muscle trauma, polymyositis
Correct answers: 1-A; 2-C; 3-B, E; 4-D; 5-B; 6-F Correspondence variants: 6
QUESTION N 262. Choose the correct answer: Vitamin K regulates the synthesis of blood clotting factors
Question type: 1. Choosing the only correct answer
Question value (points): 1
Answer variants:
1. VII
2. IX
3. X
4. All of the above
Correct answer: 4 Answer variants: 4
QUESTION N 263. Fill up the blanks of the following. Plasmin normally exists in plasma in the inactive
form___________.
Question type: 5. Entering the answer manually using the keyboard
Question value (points): 1
Answer variants:
1. plasminogen
2. Plasminogen
All answers are correct Answer variants: 2
QUESTION N 264. Fill up the blanks of the following. Antithrombin III has a major antitrombin activity and has
some ________ activity.
Question type: 5. Entering the answer manually using the keyboard
Question value (points): 1
Answer variants:
1. endogenus
2. Endogenus
All answers are correct Answer variants: 2
QUESTION N 265. Fill up the blanks of the following. The common deficiency of factor ________ produces a
disease known as hemophilia A.
Question type: 5. Entering the answer manually using the keyboard
Question value (points): 1
Answer variants:
1. VIII
2. 8
3. eight
All answers are correct Answer variants: 3
QUESTION N 266. Fill up the blanks of the following. Thrombin converts factor XIII to XIIIa which is a
________.
Question type: 5. Entering the answer manually using the keyboard
Question value (points): 1
Answer variants:
1. transglutaminase
2. Transglutaminase
All answers are correct Answer variants: 2
QUESTION N 267. Fill up the blanks of the following. Prothrombin is a single chain ________with a molecular
weight of 72000.
Question type: 5. Entering the answer manually using the keyboard
Question value (points): 1
Answer variants:
1. glycoprotein
2. Glycoprotein
All answers are correct Answer variants: 2
QUESTION N 268. Fill up the blanks of the following. The initial fibrin clot is _________ and held together only
by fibrin monomers.
Question type: 5. Entering the answer manually using the keyboard
Question value (points): 1
Answer variants:
1. weak
2. Weak
All answers are correct Answer variants: 2
QUESTION N 269. Fill up the blanks of the following. Fibrinogen is a soluble plasma ________.
Question type: 5. Entering the answer manually using the keyboard
Question value (points): 1
Answer variants:
1. Glycoprotein
2. glycoprotein
All answers are correct Answer variants: 2
QUESTION N 270. Fill up the blanks of the following. The white thrombus is composed of platelets and fibrin
and is poor in _________.
Question type: 5. Entering the answer manually using the keyboard
Question value (points): 1
Answer variants:
1. Erythrocytes
2. erythrocytes
All answers are correct Answer variants: 2
QUESTION N 271. Fill up the blanks of the following. Stop of bleeding is said to be _________.
Question type: 5. Entering the answer manually using the keyboard
Question value (points): 1
Answer variants:
1. hemostasis
2. Hemostasis
All answers are correct Answer variants: 2
QUESTION N 272. Fill up the blanks of the following. Blood platelets contain a contractile protein which is
involved in the process of clot _______.
Question type: 5. Entering the answer manually using the keyboard
Question value (points): 1
Answer variants:
1. Retraction
2. retraction
All answers are correct Answer variants: 2
QUESTION N 273. Which of the following statements about adult hemoglobin (HbA) is TRUE?
Question type: 1. Choosing the only correct answer
Question value (points): 1
Answer variants:
1. HbA is composed of two betta and two gumma subunits
2. Two subunits combine to form HbA
3. Each subunit of HbA binds 1 mole of O2
4. The betta chain of HbA is more hydrophobic than the betta chain of sickle cell hemoglobin (HbS)
Correct answer: 3 Answer variants: 4
QUESTION N 274. In red blood cells, a pyruvate kinase deficiency would be expected to increase
Question type: 1. Choosing the only correct answer
Question value (points): 1
Answer variants:
1. the lifespan of the cells
2. the rate of fatty acid oxidation
3. ATP production
4. the NADH/NAD+ ratio
5. the activity of hexokinase
Correct answer: 4 Answer variants: 5
QUESTION N 275. Which of the following changes occurs in hemolytic anemia?
Question type: 1. Choosing the only correct answer
Question value (points): 1
Answer variants:
1. A decrease in the rate of formation of bilirubin diglucuronides in the liver
2. A decrease in the rate of secretion of bilirubin diglucuronides into the gallbladder
3. An increase in the amount of bile pigments converted to stercobilin in the intestine
4. An increase in the amount of hemoglobin in the blood
Correct answer: 3 Answer variants: 4
QUESTION N 276. When excessive amounts of iron are present in the diet, the excess iron is stored as
Question type: 1. Choosing the only correct answer
Question value (points): 1
Answer variants:
1. hemoglobin
2. transferrin
3. hemosiderin
4. ferritin
Correct answer: 3 Answer variants: 4
QUESTION N 277. For each untreated condition below, select the blood or urine value that best distinguishes
that condition from the others. All values are measured after an overnight fast and are compared with those
of a normal individual.
Question type: 4. Arranging the correspondences for the answers
Question value (points): 1
Answer variants:
1. Type 1 (insulin-dependent) diabetes melitus (DM)
2. Myocardial infarction
3. Hepatitis
4. Renal failure
Answer variants: 4
Correspondence variants:
A. Increased MB fraction of serum creatine kinase (CK)
B. Increased blood ketone bodies
C. Decreased creatinine in the urine
D. Decreased blood urea nitrogen (BUN)
Correct answers: 1-B; 2-A; 3-D; 4-C Correspondence variants: 4
QUESTION N 278. Match the corresponding couples:
Question type: 4. Arranging the correspondences for the answers
Question value (points): 1
Answer variants:
1. Insulin
2. Glucagon
3. Both
4. None
Answer variants: 4
Correspondence variants:
A. It is synthesized as a prehormone
B. It is activated by partial cleavage
C. It is secretion is stimulated by guts hormones
D. Its secretion is inhibited by a hormone release in response to stress, trauma, vigorous exercise
Correct answers: 3-A, B; 2-A, B; 1-A, B, C, D; 4- Correspondence variants: 4
QUESTION N 279. Choose the correct statements about the effects of insulin
Question type: 1. Choosing the only correct answer
Question value (points): 1
Answer variants:
1. Induction or repression of specific genes
2. Stimulation of general protein synthesis
3. Stimulation of glucose transport
4. Phosphorylation of proteins
5. All of the above
Correct answer: 5 Answer variants: 5
QUESTION N 280. Low insulin/glucagon ratio results in
Question type: 1. Choosing the only correct answer
Question value (points): 1
Answer variants:
1. Phosphorylation of pyruvate dehydrogenase
2. Inactivation of pyruvate dehydrogenase
3. Induced synthesis of phosphoenolpyruvate carboxykinase
4. Induced synthesis of glucokinase
5. Decreased blood glucose level
Correct answer: 3 Answer variants: 5
QUESTION N 281. Choose the correct amino acid sequences into collagen:
Question type: 1. Choosing the only correct answer
Question value (points): 1
Answer variants:
1. Gly-Ala-Val-
2. Lys-Arg-Pro-
3. Gly-Hydroxypro- Pro-
4. Hudroxypro-Glu-Asp-
Correct answer: 3 Answer variants: 4
QUESTION N 282. Specify the correct order of stages of collagen biosynthesis and formation of collagenic
fibres
Question type: 3. Arranging the answers in the correct sequence
Question value (points): 1
Answer variants:
1. Synthesis of procollagen polypeptide chain
2. Hydroxylation of proline and lysine
3. Addition of galactosyl-glucose to hydroxylysine residues
4. Formation of procollagen
5. The C- and N- terminal peptide is removed by proteases
6. Assemble into collagen-fibrils
7. Formation of collagenic fibres
All answers are correct Answer variants: 7
QUESTION N 283. Choose the correct answers: High durability of collagenic fibres is caused:
Question type: 2. Choosing several possible correct answers
Question value (points): 1
Answer variants:
1. By lots of strong covalent bonds between tropocollagen molecules
2. Formation of a threefold helix from polypeptide chains
3. Interaction of proteoglycans with collagenic fibers
4. Hydrophobic bonds between tropocollagen molecules
5. Shift of tropocollagen molecules in 1/4 relation to each other
Correct answers: 1; 2; 5Answer variants: 5
QUESTION N 284. In collagen of which type spiral parts of polypeptide are interrupted by short non spiral
segments?
Question type: 5. Entering the answer manually using the keyboard
Question value (points): 1
Answer variants:
1. IV
2. 4
All answers are correct Answer variants: 2
QUESTION N 285. Choose the reasons (4) of infringement of biosynthesis of collagen and formation of
collagenic fibres :
Question type: 2. Choosing several possible correct answers
Question value (points): 1
Answer variants:
1. Mutations in collagen genes
2. Defects of enzymes participating in posttranslational modifications
3. Hypovitaminoses (vit C, PP, B6)
4. Low contents of coenzymes (Fe2+, Cu 2+)
5. Hyperparathireoidism
Correct answers: 1; 2; 3; 4 Answer variants: 5
QUESTION N 286. Math the letter and the number:
Question type: 4. Arranging the correspondences for the answers
Question value (points): 1
Answer variants:
1. Collagen II type
2. Collagen IY type
3. Both
4. None of the above
Answer variants: 4
Correspondence variants:
A. Fibreforming
B. Associated with fibres
C. Main component of basal membrane
D. Hydroxyproline is in it composition
Correct answers: 1-A; 2-C; 3-D; 4-B Correspondence variants: 4
QUESTION N 287. Choose the correct statements: biosynthesis of collagen is different from biosynthesis of
other proteins by:
Question type: 1. Choosing the only correct answer
Question value (points): 1
Answer variants:
1. Molecule of collagen has the longest polypeptide chain
2. The formation of polypeptide chain is result of multistage splising of mRNA
3. Gen coding pro-alpha-chain engages great area of DNA
4. There are many different enzymes and cofactors in posttranslation reactions
5. All statements are correct
Correct answer: 5 Answer variants: 5
QUESTION N 288. Collagen cannot form alpha-helix or beta-layer because in its composition there are many
amino acids:
Question type: 1. Choosing the only correct answer
Question value (points): 1
Answer variants:
1. Glycine
2. Alanine
3. Valine
4. Proline
Correct answer: 4 Answer variants: 4
QUESTION N 289. Name the tissues with high contents of elastin:
Question type: 1. Choosing the only correct answer
Question value (points): 1
Answer variants:
1. Bones
2. Cervical ligament
3. Aortas wall
4. Articularis
5. Achilles tendon
Correct answer: 3 Answer variants: 5
QUESTION N 290. One of the enzymes is activated by glucagon. It is
Question type: 1. Choosing the only correct answer
Question value (points): 1
Answer variants:
1. Phosphodiesterase
2. Pancreatic lipase
3. Glycogen synthase
4. Phosphorylase kinase
5. Lipoprotein lipase
Correct answer: 4 Answer variants: 5
QUESTION N 291. Which of the following liver enzymes becomes less active when a diabetic person in
ketoacidosis is treated with insulin?
Question type: 1. Choosing the only correct answer
Question value (points): 1
Answer variants:
1. Fructose 1,6-bisphosphatase
2. Pyruvate kinase
3. Pyruvate dehydrogenase
4. Phosphofructokinase 1 (PFK1)
5. All the above
Correct answer: 1 Answer variants: 5
QUESTION N 292. Classify the main steps of thyroid hormone synthesis in correct order:
Question type: 3. Arranging the answers in the correct sequence
Question value (points): 1
Answer variants:
1. The transport of iodide from the blood into the thyroid acinar
2. The oxidation of iodide
3. The iodination of tyrosyl residues on the protein
4. The coupling of residues of monoiodo- and diiodotyrosine
5. Proteolytic cleavage of thyroglobulin
All answers are correct Answer variants: 5
QUESTION N 293. All of the following statements about thyroid hormones are correct, EXCEPT:
Question type: 1. Choosing the only correct answer
Question value (points): 1
Answer variants:
1. They are synthesized from tyrosine
2. TSH stimulates the thyroid hormone synthesis
3. In hypothyroidism the blood level of TSH is decreased
4. A deficiency of thyroid hormones causes cold intolerance
5. Thyroid hormones inhibit the TSH synthesis
Correct answer: 3 Answer variants: 5
QUESTION N 294. A dietary deficiency of iodine would:
Question type: 1. Choosing the only correct answer
Question value (points): 1
Answer variants:
1. Directly affect the synthesis of thyroglobulin on ribosomes
2. Result in increased secretion of TSH
3. Result in decreased production of TRH
4. Result in increased heat production
5. All of the above
Correct answer: 3 Answer variants: 5
QUESTION N 295. All of the following statements about corticoid hormones are correct, EXCEPT:
Question type: 1. Choosing the only correct answer
Question value (points): 1
Answer variants:
1. They are synthesized from cholesterol
2. They are all glucocorticoids
3. ACTH stimulates their synthesis
4. Cortisol inhibits both CRH and ACTH production
5. Adrenal cortex also produces androgens
Correct answer: 2 Answer variants: 5
QUESTION N 296. Match the characteristic with the appropriate steroid hormone:
Question type: 4. Arranging the correspondences for the answers
Question value (points): 1
Answer variants:
1. Cortisol
2. Aldosterone
3. Both
4. None
Answer variants: 4
Correspondence variants:
A. Action mediated by a second messenger
B. Receptors that have a DNA binding domain
C. Associated with induction of phosphoenolpyruvate carboxykinase
D. Secreted in response to angiotensin
Correct answers: 2-D; 3-B; 1-C; 4-A Correspondence variants: 4
QUESTION N 297. Which one of the following statements about the effect of ADH is incorrect. ADH:
Question type: 1. Choosing the only correct answer
Question value (points): 1
Answer variants:
1. Is stimulated by increased osmolality of plasma
2. Increases the reabsorption of water from the distal renal tubules
3. Stimulates proteinkinase A
4. Causes increased expression of the gene for aquaporin 2
5. Is stimulated by hemodilution
Correct answer: 5 Answer variants: 5
QUESTION N 298. Choose the appropriate statement about aldosterone. Aldosterone:
Question type: 1. Choosing the only correct answer
Question value (points): 1
Answer variants:
1. Is synthesized in the zona glomerulosa of the adrenal cortex
2. Has cholesterol as the precursor
3. Requires four cytochrome P450 enzymes for its biosynthesis
4. Production is stimulated by serum Na+ and Cl- levels
5. All the above
Correct answer: 5 Answer variants: 5
QUESTION N 299. Choose the correct couples:
Question type: 4. Arranging the correspondences for the answers
Question value (points): 1
Answer variants:
1. Only aldosterone
2. Only angiotensin II
3. Both
4. None
Answer variants: 4
Correspondence variants:
A. Causes a potent vasoconstrictor action
C. Causes sodium and water reabsorption
D. Inhibits renin secretion
E. Promotes the secretion of K+, H+ and NH4+
Correct answers: 1-C; 2-D; 3-A; 4-E Correspondence variants: 4
QUESTION N 300. Choose the correct statements(4). ANP:
Question type: 2. Choosing several possible correct answers
Question value (points): 1
Answer variants:
1. Binds to the plasma membrane receptors of the target tissues
2. Causes the activation of phospholipase C
3. Causes the activation of guanylate cyclase
4. Increases sodium and water excretion
5. Inhibits renin, aldosterone and ADH production and secretion
Correct answers: 1; 3; 4; 5 Answer variants: 5
QUESTION N 301. Match the figures with the letters:
Question type: 4. Arranging the correspondences for the answers
Question value (points): 1
Answer variants:
1. Diabetes insipidus
2. Addison's disease
3. Both
4. None
Answer variants: 4
Correspondence variants:
A. Deficiency of hormones controls water and sodium balance
B. Destruction of adrenal cortex
C. Excretion of large volume of the dilute urine
D. Hypertension and metabolic alkalosis
Correct answers: 1-C; 2-B; 3-A; 4-D Correspondence variants: 4
QUESTION N 302. Choose the correct couples:
Question type: 4. Arranging the correspondences for the answers
Question value (points): 1
Answer variants:
1. Glucagon
2. TSH
3. Thyroxine
4. Aldosterone
Answer variants: 4
Correspondence variants:
A. Polypeptide
B. Steroid
C. Derived from tyrosine
D. Glycoprotein
Correct answers: 1-A; 2-D; 3-C; 4-B Correspondence variants: 4
QUESTION N 303. Addison's disease is followed by:
Question type: 1. Choosing the only correct answer
Question value (points): 1
Answer variants:
1. Deficiency of adrenocortical steroids
2. Loss of sodium ions and water into the urine
3. Hypokalemia
4. Reduction in blood pressure
5. All the above
Correct answer: 5 Answer variants: 5
QUESTION N 304. What reactions (2) occur in the synthesis of calcitriol from 7- dehydrocholesterol:
Question type: 2. Choosing several possible correct answers
Question value (points): 1
Answer variants:
1. The steroid ring structure remains intact
2. Cholesterol is an precursor
3. Ultraviolet light is required
4. Three hydroxylations occur
5. All reactions occur in liver
Correct answers: 2; 3 Answer variants: 5
QUESTION N 305. Choose the correct couples.
Question type: 4. Arranging the correspondences for the answers
Question value (points): 1
Answer variants:
1. Kidney
2. Skin
4. Intestine
Answer variants: 3
Correspondence variants:
A. Conversion of 7-dehydrocholesterol to vitamin D in nonenzymatic photolysis reaction
B. Production 1,25 (OH)2D3 in monooxygenase reaction that requires NADPH and O2
C. Induction of calcium-binding proteins synthesis
Correct answers: 1-B; 2-A; 4-C Correspondence variants: 3
QUESTION N 306. Compare the effects and mechanism of action of CT and calcitriol and match the correct
couples:
Question type: 4. Arranging the correspondences for the answers
Question value (points): 1
Answer variants:
1. Only calcitonine
2. Only calcitriol
3. Both
Answer variants: 3
Correspondence variants:
A. cAMP is the intracellular messenger of action
B. The primary targets of action are genes
C. Changes ionized calcium level in the blood
Correct answers: 1-A; 2-B; 3-C Correspondence variants: 3
QUESTION N 307. Choose the correct answer. Rickets may be caused by:
Question type: 1. Choosing the only correct answer
Question value (points): 1
Answer variants:
1. Deficiency of vitamin D in fuel
2. Defect in the conversion of vitamin D3 to calcitriol
3. Malabsorption of vitamin D in the intestine
4. Chronic renal failure
5. All the above
Correct answer: 5 Answer variants: 5
QUESTION N 308. All of the following statements about symptoms of hyperparathyroidism are correct,
EXCEPT:
Question type: 1. Choosing the only correct answer
Question value (points): 1
Answer variants:
1. The concentration of calcium in serum is elevated
2. The concentration of phosphate in serum is elevated
3. Generalized demineralization of the skeleton
4. Skeletal abnormalities
5. Renal colic caused by kidney stones
Correct answer: 2 Answer variants: 5
QUESTION N 309. In the synthesis of 1,25-DHC from 7-dehydrocholesterol
Question type: 1. Choosing the only correct answer
Question value (points): 1
Answer variants:
1. the steroid ring structure remains intact
2. cholesterol in an intermediate
3. ultraviolet light is required
4. three hydroxylations occur
Correct answer: 3 Answer variants: 4
QUESTION N 310. A dietary deficiency of iodine would
Question type: 1. Choosing the only correct answer
Question value (points): 1
Answer variants:
1. directly affect the synthesis of thyroglobulin on ribosomes
2. result in increased secretion of thyroid stimulating hormone (TSH)
3. result in decreased production of thyrotropin releasing hormone (TRH)
4. result in increased heat production
Correct answer: 2 Answer variants: 4
QUESTION N 311. A female patient with thinning scalp hair, excessive facial hair, and high ACTH and low
cortisol levels in the blood would most likely have
Question type: 1. Choosing the only correct answer
Question value (points): 1
Answer variants:
1. a tumor of the anterior pituitary that produces abnormally large amounts of proopiomelanocortin (POMC)
2. a tumor of the adrenal medulla that secretes abnormally large amounts of its normal hormone product
3. a genetic deficiency of an enzyme in the pathway for cortisol synthesis
4. a mutation in the gene for corticotropin releasing hormone that decreases production of this polypeptide
Correct answer: 3 Answer variants: 4
QUESTION N 313. Action mediated by second messenger
Question type: 1. Choosing the only correct answer
Question value (points): 1
Answer variants:
1. Cortisol
2. Aldosterone
3. Both cortisol and aldosterone
4. Neither cortisol nor aldosterone
Correct answer: 4 Answer variants: 4
QUESTION N 314. Synthesized from cholesterol by cells of the adrenal cortex
Question type: 1. Choosing the only correct answer
Question value (points): 1
Answer variants:
1. Cortisol
2. Aldosterone
3. Both cortisol and aldosterone
4. Neither cortisol nor aldosterone
Correct answer: 3 Answer variants: 4
QUESTION N 315. Receptors have a DNA binding domain
Question type: 1. Choosing the only correct answer
Question value (points): 1
Answer variants:
1. Cortisol
2. Aldosterone
3. Both cortisol and aldosterone
4. Neither cortisol nor aldosterone
Correct answer: 3 Answer variants: 4
QUESTION N 316. Associated with induction of phosphoenolpyruvate carboxykinase (PEPCK)
Question type: 1. Choosing the only correct answer
Question value (points): 1
Answer variants:
1. Cortisol
2. Aldosterone
3. Both cortisol and aldosterone
4. Neither cortisol nor aldosterone
Correct answer: 1 Answer variants: 4
QUESTION N 317. Secreted in response to angiotensin II
Question type: 1. Choosing the only correct answer
Question value (points): 1
Answer variants:
1. Cortisol
2. Aldosterone
3. Both cortisol and aldosterone
4. Neither cortisol nor aldosterone
Correct answer: 2 Answer variants: 4
QUESTION N 318. Thin limbs, central obesity, fat cheeks, a ruddy complexion, and an elevated blood glucose
level. Choose the most likely cause from the list above
Question type: 1. Choosing the only correct answer
Question value (points): 1
Answer variants:
1. Elevated blood levels of aldosterone and renin resulting from an atherosclerotic plaque in a renal artery
2. Hyperprolactinemia due to a pituitary tumor
3. Acromegaly due to a growth hormone (GH)-producing tumor that developed in adulthood
4. Cushing's syndrome due to an adrenal tumor
Correct answer: 3 Answer variants: 4
QUESTION N 319. Thin limbs, central obesity, fat cheeks, a ruddy complexion, and an elevated blood glucose
level
Question type: 1. Choosing the only correct answer
Question value (points): 1
Answer variants:
1. Elevated blood levels of aldosterone and renin resulting from an atherosclerotic plaque in a renal artery
2. Hyperprolactinemia due to a pituitary tumor
3. Acromegaly due to a growth hormone (GH)-producing tumor that developed in adulthood
4. Cushing's syndrome due to an adrenal tumor
Correct answer: 4 Answer variants: 4
QUESTION N 320. Hypertension and heart disease
Question type: 1. Choosing the only correct answer
Question value (points): 1
Answer variants:
1. Elevated blood levels of aldosterone and renin resulting from an atherosclerotic plaque in a renal artery
2. Hyperprolactinemia due to a pituitary tumor
3. Acromegaly due to a growth hormone (GH)-producing tumor that developed in adulthood
4. Cushing's syndrome due to an adrenal tumor
Correct answer: 1 Answer variants: 4
QUESTION N 321. Galactorrhea, amenorrhea, and blurred vision
Question type: 1. Choosing the only correct answer
Question value (points): 1
Answer variants:
1. Elevated blood levels of aldosterone and renin resulting from an atherosclerotic plaque in a renal artery
2. Hyperprolactinemia due to a pituitary tumor
3. Acromegaly due to a growth hormone (GH)-producing tumor that developed in adulthood
4. Cushing's syndrome due to an adrenal tumor
Correct answer: 2 Answer variants: 4
QUESTION N 322. Which of the following is true of receptor subtypes?
Question type: 1. Choosing the only correct answer
Question value (points): 1
Answer variants:
1. Subtypes of a specific receptor bind to different hormones
2. Subtypes of a specific receptor all elicit the same effect
3. Subtypes of a specific receptor may elicit different effects
4. No receptor subtypes have been identified
5. Receptor subtypes for hydrophilic hormone do not exist
Correct answer: 3 Answer variants: 5
QUESTION N 323. The role of Gs protein in the activation of adenylate cyclase is best described by which of
the following statements?
Question type: 1. Choosing the only correct answer
Question value (points): 1
Answer variants:
1. Gs protein forms a complex with hormone, and the hormone-Gs protein complex ac-tivates adenylate cyclase
2. Activation of receptor by hormone relieves the inhibition of adenylate cyclase by Gs protein
3. The Gs protein activates adenylate cyclase in a reaction that is driven by the hydroly-sis of guanosine
triphosphate
(GTP) to guanosine diphosphate (GDP)
4. The G-alpha subunit of Gs protein exchanges GDP for GTP, dissociates from the G-beta,gamma subunits, and
activates adenylate cyclase
Correct answer: 4 Answer variants: 4
QUESTION N 324. Which one of the following statements about cyclic adenosine monophosphate (cAMP) is
true?
Question type: 1. Choosing the only correct answer
Question value (points): 1
Answer variants:
1. High levels of cAMP that result from activation of G proteins by hormones are nor-mally prolonged
2. cAMP is formed by phospholipase C-beta and adenylate cyclase
3. Levels of cAMP quickly decline because it is hydrolyzed by cyclic nucleotide phos-phodiesterase (PDEase)
4. cAMP phosphorylates proteins in the cell
Correct answer: 3 Answer variants: 4
QUESTION N 325. The mechanisms through which the products of the erbB oncogene, cholera toxin, and the
phorbol esters exert deleterious effects on an organism illustrate which critically important characteristic of
hormone-receptor systems?
Question type: 1. Choosing the only correct answer
Question value (points): 1
Answer variants:
1. The appropriate inactivation of the hormonal signal is necessary for normal cell function
2. Hormone-receptor complexes must form and be internalized for proper cell signaling
3. Second messengers must be generated for cells to respond to hormones
4. Gene expression must ultimately be altered for cells to respond to hormones
5. Covalent modification is not involved in normal cell signaling
Correct answer: 1 Answer variants: 5
QUESTION N 326. Which one of the following inhibits adenylate cyclase?
Question type: 1. Choosing the only correct answer
Question value (points): 1
Answer variants:
1. Phosphodiesterase
2. Gq
3. Cyclic adenosine monophosphate (cAMP)
4. Gi
5. Cholera toxin
Correct answer: 4 Answer variants: 5
QUESTION N 327. Phospholipase C is best described by which one of the following actions?
Question type: 1. Choosing the only correct answer
Question value (points): 1
Answer variants:
1. It exists as a membrane phospholipid
2. It diffuses into the cytosol and causes the release of calcium ions from intracellular stores
3. It hydrolyzes phosphatidylinositol 4,5-bisphosphate (PIP2) to inositol 1,4,5-triphosphate (IP3) and diacylglycerol
(DAG), which are both second messengers
4. It directly activates protein kinase C
5. It dephosphorylates IP3
Correct answer: 3 Answer variants: 5
QUESTION N 328. Which one of the following statements best describes the mechanism of action of insulin on
target cells?
Question type: 1. Choosing the only correct answer
Question value (points): 1
Answer variants:
1. Insulin binds to cytoplasmic receptor molecules and is transferred as a hormone-receptor complex to the nucleus,
where it acts to modulate gene expression
2. Insulin binds to a receptor molecule on the outer surface of the plasma membrane, and the hormone-receptor
complex activates adenylate cyclase through the Gs protein
3. Insulin binds to a transmembrane receptor at the outer surface of the plasma mem-brane, which activates the
tyrosine kinase that is the cytosolic domain of the receptor
4. Insulin enters the cell and causes the release of calcium ions from intracellular stores
Correct answer: 3 Answer variants: 4
QUESTION N 329. Which one of the following dimerizes upon phosphorylation by tyrosine kinase and then
translocates to the nucleus and directly activates the expression of particular genes?
Question type: 1. Choosing the only correct answer
Question value (points): 1
Answer variants:
1. erbB
2. STATs
3. Ras
4. Steroid hormone receptor
5. Guanylate cyclase
Correct answer: 2 Answer variants: 5
QUESTION N 330. Hydrophilic hormones are best described by which one of the following statements? They:
Question type: 1. Choosing the only correct answer
Question value (points): 1
Answer variants:
1. Include the thyroid and steroid hormones
2. Bind to cell-surface receptors, which transmit a signal to the interior of the cell
3. Enter the cell, bind to intracellular receptors, and in a complex with the receptor, al-ter gene expression
4. Bind irreversibly to their receptors
5. Bind to G proteins in the cell membrane
Correct answer: 2 Answer variants: 5
QUESTION N 331. Which one of the following hormones would have the longest duration of action?
Question type: 1. Choosing the only correct answer
Question value (points): 1
Answer variants:
1. Thyroxine
2. Insulin
3. Glucagon
4. Epinephrine
5. Epidermal growth factor (EGF)
Correct answer: 1 Answer variants: 5
QUESTION N 332. Which of the following is true of epinephrine?
Question type: 1. Choosing the only correct answer
Question value (points): 1
Answer variants:
1. It acts only through the phosphatidylinositol biphosphate system
2. It is synthesized from tyrosine
3. It causes the level of cAMP in liver cells to decrease
4. It functions like a steroid hormone
Correct answer: 2 Answer variants: 4
QUESTION N 333. Which of the following acts to increase the release of Ca2+ from the endoplasmic
reticulum?
Question type: 1. Choosing the only correct answer
Question value (points): 1
Answer variants:
1. Diacylglycerol (DAG)
2. Inositol triphosphate (IP3)
3. Parathyroid hormone (PTH)
4. 1,25-Dihydroxycholecalciferol (1,25-DHC)
Correct answer: 2 Answer variants: 4
QUESTION N 334. Choose the correct answers: Elastin is characterized by such positions:
Question type: 2. Choosing several possible correct answers
Question value (points): 1
Answer variants:
1. It is the fibrous protein
2. It is capable to a return stretching
3. It settles down on walls of large vessels, providing their elastic properties
4. It prevails in structure of large tendons
5. It provides mesh structure of elastin
Correct answers: 1; 2; 3; 5 Answer variants: 5
QUESTION N 335. Math the letter and the number:
Question type: 4. Arranging the correspondences for the answers
Question value (points): 1
Answer variants:
1. Collagen
2. Elastin
3. Both
4. None of the above
Answer variants: 4
Correspondence variants:
A. Structural protein of extracellular matrix
B. With high contents of hydrophobic amino acids
C. Main component of extracellular matrix, bones
D. With high contents of Asp, Glu residues
Correct answers: 1-C; 2-B; 3-A; 4-D Correspondence variants: 4
QUESTION N 336. Fill up the blanks of the followings :On aging the skin becomes wrinkled, dry due to of the
correlation of ______fibers decreases in connective tissue. This leads to the considerable decrease of
connection water and the decrease of tissue an skin turgor.
Question type: 5. Entering the answer manually using the keyboard
Question value (points): 1
Answer variants:
1. proteoglycan
2. Proteoglycan
All answers are correct Answer variants: 2
QUESTION N 337. What properties of collagen metabolism lead to injures healing?
Question type: 1. Choosing the only correct answer
Question value (points): 1
Answer variants:
1. Increase of collagenase synthesis
2. Collagen ability induces aggregation of thrombocytes
3. Depression of prolypheration of collagen synthesizing cells
4. None of the above
5. All of the above
Correct answer: 2 Answer variants: 5
QUESTION N 338. Choose the correct couples The causes of collagen structure changes:
Question type: 4. Arranging the correspondences for the answers
Question value (points): 1
Answer variants:
1. Decrease of lysyloxidase activity
2. Change of primary structure
3. Absence of lysyloxidase
4. Decrease of proline hydroxylase activity
Answer variants: 4
Correspondence variants:
A. Mutations in DNA of fibroblasts
B. Deficiency of Cu 2+
C. Deficiency of vitamin A
D. Deficiency of vitamin C
Correct answers: 1-B; 2-A; 3-C; 4-D Correspondence variants: 4
QUESTION N 339. Indicate True or False of the following: Collagenase produced by some pathogenic
bacteria is a factor of their penetration into the human body because it destructs the structure of
extracellular matrix.
Question type: 5. Entering the answer manually using the keyboard
Question value (points): 1
Answer variants:
1. true
2. True
All answers are correct Answer variants: 2
QUESTION N 340. Excretion of oxyproline in urine with age is decreased because:
Question type: 1. Choosing the only correct answer
Question value (points): 1
Answer variants:
1. The number of transversal collagen bonds with age is increased
2. Its sollubility desreases
3. Its accessibility of collagenase action decreases
4. All of the above
Correct answer: 4 Answer variants: 4
QUESTION N 341. Choose the correct answers (3). Proteoglycan complexes are amortizators in matrix
because of:
Question type: 2. Choosing several possible correct answers
Question value (points): 1
Answer variants:
1. The same charges of sulfated proteoglycan chains cause their repel.
2. Proteoglycans are hydratated.
3. Water connected by glycosaminoglycans forms the gel limiting diffusion and permeability.
4. On increasing external pressure molecules are approximated, squeezing out water from intermolecular intervals,
when the pressure is limited they come back to their initial position.
5. Glycosaminoglycans as polyvalent anions are capable to connect plenty of ions of sodium.
Correct answers: 2; 3; 4Answer variants: 5
QUESTION N 342. Choose the correct statements, which describe composition and structure of fibronectin:
Question type: 2. Choosing several possible correct answers
Question value (points): 1
Answer variants:
1. It is lipoprotein.
2. It is glycoprotein.
3. It has a domain structure.
4. It is metalloprotein.
5. It has some centres of linkage (polyvalent protein).
Correct answers: 2; 3; 5Answer variants: 5
QUESTION N 343. Indicate True or False of the statement: One of the factors, leading to metastasize of
malignant tumor is the decrease of fibronectin amount on cells surface because of fibronectin is a substance,
that maintanes intercellular interaction.
Question type: 5. Entering the answer manually using the keyboard
Question value (points): 1
Answer variants:
1. true
2. True
All answers are correct Answer variants: 2
QUESTION N 344. In scurvy, defective collagen is produced because hydroxylation of procollagen does not
occur at the normal rate. Hydroxylation of lysine residues on procollagen
Question type: 1. Choosing the only correct answer
Question value (points): 1
Answer variants:
1. is not required for polymerization of collagen
2. requires vitamin C. as does hydroxilation of proline residues
3. occurs before incorporation of lysine into the polypeptide chain
4. occurs after the mature collagen molecule is secreted from the cell
Correct answer: 2 Answer variants: 4
QUESTION N 345. Which one of the following does not change length during muscle contraction?
Question type: 1. Choosing the only correct answer
Question value (points): 1
Answer variants:
1. The A band
2. The I band
3. The H zone
4. The sarcomere
5. The myofibril
Correct answer: 1 Answer variants: 5
QUESTION N 346. The primary component(s) of thin filaments include which one of the following?
Question type: 1. Choosing the only correct answer
Question value (points): 1
Answer variants:
1. Actin
2. Myosin
3. Tropomyosin and myosin
4. Actin, tropomyosin and troponin
5. Myosin and M protein
Correct answer: 4 Answer variants: 5
QUESTION N 347. Choose correct definitions from the list below:
Question type: 4. Arranging the correspondences for the answers
Question value (points): 1
Answer variants:
1. Myosin
2. F-Actin
3. Actomyosin
4. Tropomyosin
5. Troponin
Answer variants: 5
Correspondence variants:
A. Forms thick filaments
B. Consist of 3 subunits
C. Consists of two heavy and 4 light chains
D. Is a part of thin filaments
E. Is formed in process of muscle contraction
Correct answers: 1-A, C; 2-D; 3-E; 4-D; 5-B Correspondence variants: 5
QUESTION N 348. Put the events in the correct order. Muscle contraction is stimulated:
Question type: 3. Arranging the answers in the correct sequence
Question value (points): 1
Answer variants:
1. Actin becomes accessible and S-1 head of myosin binds it
2. The actin-myosin-ADP-Pi complex is formed
3. ADP is released
4. Myosin reverts to low energy state
5. Molecule of ATP binds to S-1 head
6. Actin is released
7. The S-1 head hydrolyses the ATP
All answers are correct Answer variants: 7
QUESTION N 349. Choose correct inhibitor of F actin-myosin interaction. Striated muscle:
Question type: 1. Choosing the only correct answer
Question value (points): 1
Answer variants:
1. Phosphorilated myosin p-light chain
2. TpA
3. TpI
4. Unhosphorilated myosin p-light chain
Correct answer: 3 Answer variants: 4
QUESTION N 350. Choose correct inhibitor of F actin-myosin interaction. Smooth muscle:
Question type: 1. Choosing the only correct answer
Question value (points): 1
Answer variants:
1. Phosphorilated myosin p-light chain
2. TpA
3. TpI
4. Unhosphorilated myosin p-light chain
Correct answer: 4 Answer variants: 4
QUESTION N 351. Inside the sarcoplasmic reticulum Ca2+ is bound to ...
Question type: 5. Entering the answer manually using the keyboard
Question value (points): 1
Answer variants:
1. calsequestrin
2. Calsequestrin
All answers are correct Answer variants: 2
QUESTION N 352. Choose four correct definitions from the right column for each type of fibers: Type I fibers
Question type: 2. Choosing several possible correct answers
Question value (points): 1
Answer variants:
1. Increases in athletes training for sprints
2. Increases in athletes training for marathons
3. Short duration of contraction
4. Prolonged duration of contraction
5. Low energy utilization
6. High energy utilization
7. Low level of ATPase activity
8. High level of ATPase activity
Correct answers: 2; 4; 5; 7 Answer variants: 8
QUESTION N 353. Choose four correct definitions from the right column for each type of fibers: Type II fibers
Question type: 2. Choosing several possible correct answers
Question value (points): 1
Answer variants:
1. Increases in athletes training for sprints
2. Increases in athletes training for marathons
3. Short duration of contraction
4. Prolonged duration of contraction
5. Low energy utilization
6. High energy utilization
7. Low level of ATPase activity
8. High level of ATPase activity
Correct answers: 1; 3; 6; 8 Answer variants: 8
QUESTION N 354. The number of iron atoms present in myoglobin is:
Question type: 1. Choosing the only correct answer
Question value (points): 1
Answer variants:
1. 1
2. 2
3. 3
4. 4
5. None of the above
Correct answer: 1 Answer variants: 5
QUESTION N 355. The number of peptide chain present in myoglobin is:
Question type: 1. Choosing the only correct answer
Question value (points): 1
Answer variants:
1. 1
2. 2
3. 3
4. 4
5. None of the above
Correct answer: 1 Answer variants: 5
QUESTION N 356. The number of amino acids present in peptide chain of myoglobin is:
Question type: 1. Choosing the only correct answer
Question value (points): 1
Answer variants:
1. 141
2. 146
3. 153
4. 176
5. 216
Correct answer: 3 Answer variants: 5
QUESTION N 357. Isoenzymes of CPK are:
Question type: 1. Choosing the only correct answer
Question value (points): 1
Answer variants:
1. 1
2. 2
3. 3
4. 4
5. 5
Correct answer: 3 Answer variants: 5
QUESTION N 358. The source of energy in contracting muscles is:
Question type: 2. Choosing several possible correct answers
Question value (points): 1
Answer variants:
1. Adenosine triphosphate
2. Creatine phosphate
3. Creatinine phosphate
4. Actomyosine
5. Actin
Correct answers: 1; 2 Answer variants: 5
QUESTION N 359. Muscle glycogen cannot give rise to blood glucose because it lacks the enzyme:
Question type: 1. Choosing the only correct answer
Question value (points): 1
Answer variants:
1. Phospholipase
2. Branching enzyme
3. Glucose-6-phosphatase
4. Glucosinase
5. Debranching enzyme
Correct answer: 3 Answer variants: 5
QUESTION N 360. Ketone bodies cannot be utilized by:
Question type: 1. Choosing the only correct answer
Question value (points): 1
Answer variants:
1. Liver
2. Muscles
3. Kidney
4. Brain
5. None of the above
Correct answer: 1 Answer variants: 5
QUESTION N 363. Cerebrocides are:
Question type: 2. Choosing several possible correct answers
Question value (points): 1
Answer variants:
1. Sphingosine, fatty acid, galactose
2. Sphingosine, fatty acid, phosphoric acid, galactose
3. Sphingosine, fatty acid, phosphoric acid, glucose
4. Sphingosine, fatty acid, glucose
5. Sphingosine, fatty acid, lactose
Correct answers: 1; 4 Answer variants: 5
QUESTION N 364. Sphingomyelins are:
Question type: 2. Choosing several possible correct answers
Question value (points): 1
Answer variants:
1. Sphingosine, fatty acid, phosphoric acid, choline
2. Glycerol, fatty acid, phosphoric acid, choline
3. Glycerol, fatty acid, phosphoric acid, srine
4. Sphingosine, fatty acid, phosphoric acid, glycerol
5. Sphingosine, glycerol, phosphoric acid
Correct answers: 1 Answer variants: 5
QUESTION N 365. Fill the gap in the next sentence. Glutamic acid is the precursor of ...(mediator)
Question type: 5. Entering the answer manually using the keyboard
Question value (points): 1
Answer variants:
1. GABA
All answers are correct Answer variants: 1
QUESTION N 366. Fill the gap in the next sentence. The major neurotransmitting systems are
acetylcholinergic and ...
Question type: 5. Entering the answer manually using the keyboard
Question value (points): 1
Answer variants:
1. noradrenalinergic
2. Noradrenalinergic
All answers are correct Answer variants: 2
QUESTION N 367. Choose neurotransmitters from the list below:
Question type: 2. Choosing several possible correct answers
Question value (points): 1
Answer variants:
1. Adrenaline
2. Dopamine
3. Progesterone
4. Serotonin
5. Histamine
6. Cortisol
7. GABA
8. Ca2+
9. NO
Correct answers: 1; 4; 7; 9 Answer variants: 9
QUESTION N 368. The precursor of the serotonin:
Question type: 1. Choosing the only correct answer
Question value (points): 1
Answer variants:
1. Arginine
2. Tyrosine
3. Tryptophan
4. Glycine
5. Glutamate
Correct answer: 3 Answer variants: 5
QUESTION N 369. The precursor of the nitric oxide:
Question type: 1. Choosing the only correct answer
Question value (points): 1
Answer variants:
1. Arginine
2. Tyrosine
3. Tryptophan
4. Glycine
5. Glutamate
Correct answer: 1 Answer variants: 5
QUESTION N 370. Mark correct answer.Alpha-amylase cleavages:
Question type: 1. Choosing the only correct answer
Question value (points): 1
Answer variants:
1. alpha 1 -> 1 glicosidic bonds
2. alpha1 -> 4 glicosidic bonds at end saccharine residues
3. alpha 1 -> 6 glicosidic bonds
4. alpha 1 -> 4 glicosidic bonds
5. beta 1 -> 4 glicosidic bonds
6. All of the above
Correct answer: 4 Answer variants: 6
QUESTION N 371. Complete sentences: mucin is a
Question type: 1. Choosing the only correct answer
Question value (points): 1
Answer variants:
1. Carbohydrate
2. Simple protein
3. Mucoprotein
4. Nucleoprotein
5. Steroid
Correct answer: 3 Answer variants: 5
QUESTION N 372. Mark correct sentence
Question type: 1. Choosing the only correct answer
Question value (points): 1
Answer variants:
1. Pepsin is exoprotease
2. Pepsin is endoprotease
3. Pepsin hydrolyses casein
4. Pepsin hydrolyses elastine
5. Pepsinogen is activated by the autocatalytic removal of 44 N-terminal amino acid at low pH
6. Pepsinogen is activated by the enteropeptidase (enterokinase)
Correct answer: 3 Answer variants: 6
QUESTION N 373. Mark correct answer. Pancreatic secret content:
Question type: 2. Choosing several possible correct answers
Question value (points): 1
Answer variants:
1. alpha-amilase
2. Lipases
3. Trypsin
4. Trypsinogen
5. HCO3-
6. HCl
7. Mucin
Correct answers: 1; 2; 4; 5 Answer variants: 7
QUESTION N 374. What nutrients are actively absorbed by specialized transport mechanisms that are found in
the distal ileum?
Question type: 2. Choosing several possible correct answers
Question value (points): 1
Answer variants:
1. Iron
2. Water
3. Bile salt
4. Vitamin B12
5. Vitamin D
6. Amino acids
7. Fat
8. Glucose
Correct answers: 3; 4 Answer variants: 8
QUESTION N 375. Which one of the following statements is correct?
Question type: 1. Choosing the only correct answer
Question value (points): 1
Answer variants:
1. The Recommended Dietary Allowance is the minimal nutrient requirement for indi-viduals.
2. Protein requirements per kilogram of body weight are constant throughout life.
3. Alcohol, like fat, provides 9 kcal/g.
4. Thermic effect of food is the heat produced by the body during the digestion and ab-sorption of food.
5. The basal metabolic rate typically accounts for 10% to 20% of the energy expendi-ture in a sedentary individual.
Correct answer: 4 Answer variants: 5
QUESTION N 376. Which one of the following is a purely ketogenic essential amino acid?
Question type: 1. Choosing the only correct answer
Question value (points): 1
Answer variants:
1. Leucine
2. Cysteine
3. Tyrosine
4. Histidine
5. Proline
Correct answer: 1 Answer variants: 5
QUESTION N 377. Kwashiorkor disease is :
Question type: 1. Choosing the only correct answer
Question value (points): 1
Answer variants:
1. Caused by malnutrition
2. Due to poor protein intake
3. Common among infants of South Africa
4. Results in edema, diarrhoea
5. All of the above.
Correct answer: 5 Answer variants: 5
QUESTION N 378. All of the following are observed in high-fiber diets EXCEPT:
Question type: 1. Choosing the only correct answer
Question value (points): 1
Answer variants:
1. reduced incidence of constipation
2. decrease in blood cholesterol
3. increased frequency of hyperglycemia
4. decreased Zn2+ absorption
5. increased bowel motility
6. increases colonocyte proliferation
Correct answer: 3 Answer variants: 6
QUESTION N 379. Which one of the following dietary components most strongly influences the risk of
coronary artery disease?
Question type: 1. Choosing the only correct answer
Question value (points): 1
Answer variants:
1. Saturated fat
2. Polyunsaturated fat
3. Monounsaturated fat
4. Trans fatty acids
Correct answer: 1 Answer variants: 4
QUESTION N 380. Given the information that a 70-kg man is consuming a daily average of 275 g of
carbohydrate, 75 g of protein, and 65 g of lipid, one can draw which conclusion?
Question type: 2. Choosing several possible correct answers
Question value (points): 1
Answer variants:
1. Total energy intake per day is approximately 3000 kcal.
2. About 20% of the calories are derived from lipid.
3. The diet does not contain a sufficient amount of dietary fiber.
4. The proportions of carbohydrate, protein, and lipid in the diet conform to the recom-mendations of academic
groups
and government agencies.
5. The individual is in nitrogen balance.
Correct answers: 1; 2; 4; 5 Answer variants: 5
QUESTION N 381. All of the following are widely accepted dietary recommendations EXCEPT:
Question type: 1. Choosing the only correct answer
Question value (points): 1
Answer variants:
1. Limit consumption of polyunsaturated fats to 10% or less of total calories.
2. Decrease consumption of saturated fats.
3. Increase consumption of polyunsaturated fats.
4. Increase fiber consumption to 20 to 30 g per day.
5. Decrease consumption of total fats to less than 50% of total calories.
Correct answer: 1 Answer variants: 5
QUESTION N 382. Which one of the following contains the highest percent of monounsaturated fat?
Question type: 2. Choosing several possible correct answers
Question value (points): 1
Answer variants:
1. Corn oil
2. Soybean oil
3. Olive oil
4. Palm oil
5. Coconut oil
Correct answers: 3 Answer variants: 5
QUESTION N 383. The postprandial state is characterized by (2):
Question type: 2. Choosing several possible correct answers
Question value (points): 1
Answer variants:
1. high blood levels of glucose and low levels of insulin
2. high blood levels of glucose and high levels of insulin
3. high blood levels of free fatty acids and low levels of glucagon
4. high blood levels of free fatty acids and high levels of glucagon
5. high blood levels of insulin and glucagon
Correct answers: 2; 3 Answer variants: 5
QUESTION N 384. Vitamins are :
Question type: 1. Choosing the only correct answer
Question value (points): 1
Answer variants:
1. Organic compounds
2. Required in small amounts
3. Essential for normal growth, maintenance and reproduction
4. Not degraded for energy purposes
5. All of the above
Correct answer: 5 Answer variants: 5
QUESTION N 385. The fat soluble vitamin is :
Question type: 2. Choosing several possible correct answers
Question value (points): 1
Answer variants:
1. Vitamin D
2. Vitamin E
3. Vitamin
4. Vitamin A
5. All of the above
Correct answers: 1; 2; 4Answer variants: 5
QUESTION N 386. The provitamin form of Vitamin A is :
Question type: 1. Choosing the only correct answer
Question value (points): 1
Answer variants:
1. Retinol
2. Retinal
3. Carotenes
4. Xanthine
5. None of the above
Correct answer: 3 Answer variants: 5
QUESTION N 387. Which form of carotene give rise to two molecules of vitamin A:
Question type: 1. Choosing the only correct answer
Question value (points): 1
Answer variants:
1. alpha-carotene
2. beta-carotene
3. All of the above
4. None of the above
Correct answer: 2 Answer variants: 4
QUESTION N 388. Of the water soluble vitamins,.........cannot be synthesized by plants
Question type: 1. Choosing the only correct answer
Question value (points): 1
Answer variants:
1. Cobalamin
2. Biotin
3. Folic acid
4. Pantothenic acid
5. Ascorbic acid
Correct answer: 1 Answer variants: 5
QUESTION N 389. Normal human liver can store several years supply of:
Question type: 1. Choosing the only correct answer
Question value (points): 1
Answer variants:
1. Cobalamin
2. Biotin
3. Folic acid
4. Pantothenic acid
5. Ascorbic acid
Correct answer: 1 Answer variants: 5
QUESTION N 390. All the water soluble vitamins except.....serve as coenzymes or cofactors in enzymatic
reactions :
Question type: 1. Choosing the only correct answer
Question value (points): 1
Answer variants:
1. Thiamine
2. Pyridoxine
3. Nicotinic acid
4. Folic acid
5. Ascorbic acid
Correct answer: 5 Answer variants: 5
QUESTION N 391. Which of the following if taken excessively can accumulate in body and cause toxicity :
Question type: 1. Choosing the only correct answer
Question value (points): 1
Answer variants:
1. Vitamin B2
2. Vitamin B6
3. Vitamin
4. Vitamin D
5. Vitamin B12
Correct answer: 4 Answer variants: 5
QUESTION N 392. Which of the following statements regarding riboflavin is incorrect :
Question type: 2. Choosing several possible correct answers
Question value (points): 1
Answer variants:
1. It consists of a heterocyclic structure to which is attached Ribitol
2. Riboflavin is a colored and fluorescent pigment
3. It is synthesized by almost all plants
4. Free riboflavin crosses the placentas
5. Riboflavin is excreted in urine
Correct answers: 1; 4 Answer variants: 5
QUESTION N 393. New-borns infants with hyperbilirubinemia treated with phototherapy may develop
deficiency of :
Question type: 1. Choosing the only correct answer
Question value (points): 1
Answer variants:
1. Ascorbic acid
2. Riboflavin
3. Vitamin A
4. Vitamin D
5. None of the above
Correct answer: 2 Answer variants: 5
QUESTION N 394. Nicotine of tobacco is structurally related to :
Question type: 1. Choosing the only correct answer
Question value (points): 1
Answer variants:
1. Niacin
2. Pyridoxine
3. Riboflavin
4. Cobalamin
5. Folic acid
Correct answer: 1 Answer variants: 5
QUESTION N 395. The daily recommended intake of Vitamin D is :
Question type: 1. Choosing the only correct answer
Question value (points): 1
Answer variants:
1. 200 IU
2. 400 IU
3. 600 IU
4. 1000 IU
5. 25000 IU
Correct answer: 2 Answer variants: 5
QUESTION N 396. The daily recommended intake of Vitamin A is :
Question type: 1. Choosing the only correct answer
Question value (points): 1
Answer variants:
1. 500 IU
2. 2500 IU
3. 5000 IU
4. 10000 IU
5. 25000 IU
Correct answer: 3 Answer variants: 5
QUESTION N 397. Hypervitaminosis A may result if the daily intake exceeds:
Question type: 1. Choosing the only correct answer
Question value (points): 1
Answer variants:
1. 500 IU
2. 2500 IU
3. 5030 IU
4. 10000 IU
5. 50000 IU
Correct answer: 5 Answer variants: 5
QUESTION N 398. Antirachitic vitamin is :
Question type: 1. Choosing the only correct answer
Question value (points): 1
Answer variants:
1. Vitamin A
2. Vitamin D
3. Vitamin E
4. Vitamin
5. None of the above
Correct answer: 2 Answer variants: 5
QUESTION N 399. Find the calcitriol among compounds listed below :
Question type: 1. Choosing the only correct answer
Question value (points): 1
Answer variants:
1. 25 cholecalciferol
2. 1,25 dihydroxycholecalciferol
3. 24,25 dihydroxycholecalciferol
5. None of the above
Correct answer: 2 Answer variants: 4
QUESTION N 400. The formation of 25 cholecalciferol, active form of Vitamin D takes place in:
Question type: 1. Choosing the only correct answer
Question value (points): 1
Answer variants:
1. Liver
2. Kidney
3. Intestines
4. Pancreas
5. None of the above
Correct answer: 1 Answer variants: 5
QUESTION N 401. The formation of 1,25 dihydroxycholecalciferol takes place in :
Question type: 1. Choosing the only correct answer
Question value (points): 1
Answer variants:
1. Liver
2. Kidney
3. Intestines
4. Pancreas
5. None of the above
Correct answer: 2 Answer variants: 5
QUESTION N 402. The antisterility Vitamin is :
Question type: 1. Choosing the only correct answer
Question value (points): 1
Answer variants:
1. Vitamin A
2. Vitamin D
3. Vitamin E
4. Vitamin
5. None of the above.
Correct answer: 3 Answer variants: 5
QUESTION N 403. Biological antioxidant is a function of :
Question type: 1. Choosing the only correct answer
Question value (points): 1
Answer variants:
1. Vitamin A
2. Vitamin D
3. Vitamin E
4. Vitamin
5. None of the above
Correct answer: 3 Answer variants: 5
QUESTION N 404. Which of the following vitamin is present in enzyme phosphorylase :
Question type: 1. Choosing the only correct answer
Question value (points): 1
Answer variants:
1. Thiamine
2. Pyridoxine
3. Niacin
4. Pantothenic acid
5. Riboflavin
Correct answer: 2 Answer variants: 5
QUESTION N 405. Coenzyme A contains:
Question type: 1. Choosing the only correct answer
Question value (points): 1
Answer variants:
1. Pantothenic acid
2. Niacin
3. Riboflavin
4. Thiamine
5. Pyridoxine
Correct answer: 1 Answer variants: 5
QUESTION N 406. L-Amino acids needs for absorption :
Question type: 1. Choosing the only correct answer
Question value (points): 1
Answer variants:
1. Thiamine
2. Pyridoxal
3. Riboflavin
4. Vitamin B12
5. Folic acid
Correct answer: 2 Answer variants: 5
QUESTION N 407. Transamination reactions require the cofactor :
Question type: 1. Choosing the only correct answer
Question value (points): 1
Answer variants:
1. Transaminase
2. Pyridoxal phosphate
3. Thiamine pyrophosphate
4. Biotin
5. Folic acid
Correct answer: 2 Answer variants: 5
QUESTION N 408. Which of the followings vitamins is the electron carrier of mitochondrial respiratory chain :
Question type: 1. Choosing the only correct answer
Question value (points): 1
Answer variants:
1. Niacinamide
2. Riboflavin
3. Ubiquinone
4. Ascorbic acid
5. All of the above
Correct answer: 3 Answer variants: 5
QUESTION N 409. Which vitamin protects against the destruction of vitamin A by oxidation :
Question type: 1. Choosing the only correct answer
Question value (points): 1
Answer variants:
1. Vitamin
2. Vitamin E
3. Vitamin
4. Vitamin B12
5. Vitamin D
Correct answer: 2 Answer variants: 5
QUESTION N 410. The one carbon moiety is :
Question type: 1. Choosing the only correct answer
Question value (points): 1
Answer variants:
1. CHs (methyl)
2. CH2OH (hydroxy methyl)
3. CHO(formyl)
4. CH=NH (formimino group)
5. All of the above
Correct answer: 5 Answer variants: 5
QUESTION N 411. Match the following vitamins with their deficiency disease :
Question type: 4. Arranging the correspondences for the answers
Question value (points): 1
Answer variants:
1. Retinol
2. Ergocalciferol
3. alpha-Tocopherol
4. Phylloquinone
Answer variants: 4
Correspondence variants:
A. Hemorrhagic disease
B. Sterility in rats
C. Osteomalacia
D. Xerophthalmia
Correct answers: 1-D; 2-C; 3-B; 4-A Correspondence variants: 4
QUESTION N 412. Match the following vitamins with the deficiency disease :
Question type: 4. Arranging the correspondences for the answers
Question value (points): 1
Answer variants:
1. L-Ascorbic acid
2. Vitamin B1
3. Vitamin B2
4. Vitamin B6
5. Vitamin B12
6. Niacin
7. Biotin
8. Pantothenic acid
9. Folic acid
Answer variants: 9
Correspondence variants:
A. Cheilosis, photophobia
B. Anemia, achromotrichia
C. Dermatitis, alopecia
D. Cheilosis, glossitis
E. Macrocytic and megaloblastic anemias
F. Scurvy
G. Beriberi
H. Pernicious anemia
I. Pellagra, diarrhea
Correct answers: 7-C; 9-E; 1-F; 6-I; 8-B; 2-G; 5-H; 3-A; 4-D Correspondence variants: 9
QUESTION N 413. Find correspondence "Vitamin" - "Reaction":
Question type: 4. Arranging the correspondences for the answers
Question value (points): 1
Answer variants:
1. Thiamine
2. Riboflavin
3. Pyridoxine
4. Biotin
5. Folic acid
6. Cyanocobalamin
7. Lipoic acid
Answer variants: 7
Correspondence variants:
A. Methyl, methylene group transfer
B. Acetyl group transfer
C. Carbon dioxide fixation
D. Hydrogen group transfer
E. Alkyl group transfer
F. Aldehyde group transfer
G. Amino group transfer
Correct answers: 1-F; 2-D; 3-G; 4-C; 5-A; 6-E; 7-B Correspondence variants: 7
QUESTION N 414. Match the following excretory produces with the corresponding vitamins :
Question type: 4. Arranging the correspondences for the answers
Question value (points): 1
Answer variants:
1. Formimino glutamic acid
2. Pyruvic acid
3. Xanthurenic acid
4. Homogentisic acid
5. Methylmalonic acid
Answer variants: 5
Correspondence variants:
A. Vitamin B12
B. Folic acid
C. Thiamine
D. Ascorbic acid
E. Vitamin B6
Correct answers: 1-B; 4-D; 5-A; 2-C; 3-E Correspondence variants: 5
QUESTION N 415. For each vitamin listed below, select the metabolic process with which it is most likely to
be associated.
Question type: 4. Arranging the correspondences for the answers
Question value (points): 1
Answer variants:
1. Degradation of amino acids
2. Synthesis of DNA
3. Calcium metabolism
4. Electron transport
5. Vision
6. Blood clotting
7. Collagen formation
Answer variants: 7
Correspondence variants:
A. Pyridoxine (vitamin B6)
B. Folic acid
C. Cholecalciferol (vitamin D3)
D. Niacin
E. Ascorbic acid (vitamin C)
F. Vitamin A
G. Vitamin K
Correct answers: 1-A; 2-B; 3-C; 4-D; 5-F; 6-G; 7-E Correspondence variants: 7
QUESTION N 416. Which one of the following must be modified to serve as a cofactor for enzymes?
Question type: 1. Choosing the only correct answer
Question value (points): 1
Answer variants:
1. Copper
2. Biotin
3. Zinc
4. Pantothenic acid
5. Ascorbic acid
Correct answer: 4 Answer variants: 5
QUESTION N 417. Which of the following amino acids is an imino acid?
Question type: 1. Choosing the only correct answer
Question value (points): 1
Answer variants:
1. Leucine
2. Lysine
3. gamma-Carboxyglutamate
4. Glycine
5. Proline
Correct answer: 5 Answer variants: 5
QUESTION N 418. What of following processes (2) belong to the mitocondrial respiration?
Question type: 2. Choosing several possible correct answers
Question value (points): 1
Answer variants:
1. Decomposition of organic molecules into their simpler components.
2. The utilization of O2 to derive ATP from oxidizing fuels to CO2 and H2O.
3. The complete oxidation of the acetyl group to CO2 in the TCA cycle.
4. ATP generation by oxidative phosphorylation.
5. The electron transport chain running.
Correct answers: 2; 4 Answer variants: 5
QUESTION N 419. What of follow processes need in ATP as an energy source (4)?
Question type: 2. Choosing several possible correct answers
Question value (points): 1
Answer variants:
1. Mechanical work
2. Transport work
3. Biosynthetic work
4. Activated intermediates formation
5. Glycogen phosphorolysis
Correct answers: 1; 2; 3; 4 Answer variants: 5
QUESTION N 420. Choose correct answer. The electron transport chain carrier CoQ are located:
Question type: 1. Choosing the only correct answer
Question value (points): 1
Answer variants:
1. In the inner mitochondrial membrane.
2. In the mitochondrial matrix.
3. In the intermembrane space.
4. On the inner surface of the outer mitochondrial membrane
5. On the outer surface of the outer mitochondrial membrane.
Correct answer: 1 Answer variants: 5
QUESTION N 421. Many substrates use a common catabolic pathway for the transfer of electrons to oxygen
because:
Question type: 1. Choosing the only correct answer
Question value (points): 1
Answer variants:
1. The substrates are oxidized in the mitochondria.
2. Many of the substrates are oxidized by enzymes linked with NAD+ and FAD.
3. All substrates are oxidized by the same enzymes.
4. The electrons from all substrates can be transferred to ATP.
5. Protons from all substrates are used to production water.
Correct answer: 2 Answer variants: 5
QUESTION N 422. All the following electron carriers are components of the mitochondrial ETC, EXCEPT:
Question type: 1. Choosing the only correct answer
Question value (points): 1
Answer variants:
1. NAD+
2. NADP+
3. FMN
4. FAD
5. Coenzyme Q
Correct answer: 2 Answer variants: 5
QUESTION N 423. Match the figures with the letters
Question type: 4. Arranging the correspondences for the answers
Question value (points): 1
Answer variants:
1. Only NADH-dehydrogenase
2. Only CoQ
3. Both
4. None
Answer variants: 4
Correspondence variants:
A. Accepts and donates two hydrogen atoms.
B. Contains FMN as coenzyme
C. Accepts electrons donated by NADH2
D. Transfers electrons to O2
Correct answers: 1-B; 2-A; 3-C; 4-D Correspondence variants: 4
QUESTION N 424. Match the correct couple: Redox potential - Carriers
Question type: 4. Arranging the correspondences for the answers
Question value (points): 1
Answer variants:
1. 0,06
2. 0,816
3. -0,32
4. 0,29
Answer variants: 4
Correspondence variants:
A. Cyt.a
B. Coenzyme QH2/Q
C. NADH/NAD+
D. H2O/ O2
Correct answers: 1-B; 2-D; 3-C; 4-A Correspondence variants: 4
QUESTION N 425. Which of the following statements (3) about the chemiosmotic theory are true:
Question type: 2. Choosing several possible correct answers
Question value (points): 1
Answer variants:
1. The function of mitochondrial electron transport is to translocate protons across the inner membrane into the
mitochondrial matrix.
2. The free energy released by ETC is stored in an electrochemical gradient.
3. F1-ATP-ase catalyzes the in vivo synthesis of ATP.
4. The major carriers are organized into three complexes which have a vectorial arrangement within the membrane.
5. Protons can cross the membrane from intermembrane space to matrix only by passin through the ATP-ase.
Correct answers: 2; 3; 5Answer variants: 5
QUESTION N 426. How many moles of ATP can be formed per a pair of electrons transferred from NADH2 to
oxygen?
Question type: 1. Choosing the only correct answer
Question value (points): 1
Answer variants:
1. 0
2. 1
3. 2
4. 3
5. 4
Correct answer: 4 Answer variants: 5
QUESTION N 427. At which sites in the mitochondrial ETC is more than 34 kkal/mole of energy released?
Question type: 2. Choosing several possible correct answers
Question value (points): 1
Answer variants:
1. NADH2 -> CoQ
2. Cyt.(b c1) complex -> Cyt.c
3. Succinate -> Q
4. Cyt c -> Cytochrome oxidase
5. Cyt.a3 -> oxygen
Correct answers: 1; 2; 5Answer variants: 5
QUESTION N 428. Match correct sentences (3)
Question type: 2. Choosing several possible correct answers
Question value (points): 1
Answer variants:
1. The ADP phosphorylation rate is correlated with H+ influx into matrix.
2. NADH2 donates electrons to the electron transport chain
3. The use of protons from the cytosolic side decreases the proton gradient.
4. Oxygen is reduced to H2O.
5. ADP is phosphorylated to ATP.
Correct answers: 2; 4; 5Answer variants: 5
QUESTION N 429. Which of the following actions does the uncoupling of oxidative phosphorylation describe?
Question type: 1. Choosing the only correct answer
Question value (points): 1
Answer variants:
1. The phosphorylation of ADP to ATP accelerates.
2. The phosphorylation of ADP continues but oxygen uptake stops.
3. The phosphorylation of ADP stops but oxygen uptake continues.
4. Oxygen uptake stops.
5. None of the above.
Correct answer: 3 Answer variants: 5
QUESTION N 430. If both oligomycin, the inhibitor of ATP-ase, and 2,4-dinitrophenol are added to a
mitochondrial preparation in the presence of substrate and ATP, then:
Question type: 1. Choosing the only correct answer
Question value (points): 1
Answer variants:
1. Both oxygen uptake and phosphorylation of ADP would cease.
2. Oxygen uptake would be reduced, but phosphorylation of ADP would continue.
3. Oxygen uptake would be high, but phosphorylation would cease.
4. There would be no change in oxygen uptake, nor of P/O ratio.
5. None of the above.
Correct answer: 3 Answer variants: 5
QUESTION N 431. Rotenone, which is used as a fish poison and as an insecticide, blocks mitochondrial
electron transport by:
Question type: 1. Choosing the only correct answer
Question value (points): 1
Answer variants:
1. Inhibiting the interaction between oxygen and the terminal electron carrier.
2. Inhibiting the reduction of Cyt.C.
3. Inhibiting the transfer of electrons through the NADH-dehydrogenase.
4. Formation of an inactive complex with Cyt.C.
5. Inhibiting electron transfer at QH2-dehydrogenase.
Correct answer: 3 Answer variants: 5
QUESTION N 432. Carbon monoxide inhibits mitochondrial electron transport by:
Question type: 1. Choosing the only correct answer
Question value (points): 1
Answer variants:
1. Binding to hemoglobin in the erythrocytes and so blocking the transport of oxygen to tissues.
2. Binding to the oxygen-binding site of cytochrome oxidase.
3. Blocking electron transport at the level of cytochrome b-c1 complex.
4. Combining with coenzyme Q and preventing its interaction with the complex II.
5. Inhibiting the electron transport by complex I
Correct answer: 2 Answer variants: 5
QUESTION N 433. Electrons derived from many substrates may use the electron transport chain because
Question type: 1. Choosing the only correct answer
Question value (points): 1
Answer variants:
1. each substrate is oxidized in the mitochondria
2. the substrates are oxidized by enzymes linked to oxidized nicotinamide or flavin adenine dinucleotide
3. each substrate is oxidized by the same enzyme
4. electrons from each substrate are transferred to adenosine triphosphate
5. protons from each substrate are used to form water
Correct answer: 2 Answer variants: 5
QUESTION N 434. The mitochondrial electron transport chain carriers are located
Question type: 1. Choosing the only correct answer
Question value (points): 1
Answer variants:
1. in the inner mitochondrial membrane
2. in the mitochondrial matrix
3. in the intermembrane space
4. on the inner surface of the outer mitochondrial membrane
5. on the outer surface of the outer mitochondrial membrane
Correct answer: 1 Answer variants: 5
QUESTION N 435. Which one of the following disorders is characterized by a reduced capacity to produce
superoxide?
Question type: 1. Choosing the only correct answer
Question value (points): 1
Answer variants:
1. Leigh syndrome
2. Alpers syndrome
3. Pearson syndrome
4. Chronic granulomatous disease
5. Benign infantile myopathy
Correct answer: 4 Answer variants: 5
QUESTION N 436. If oxidative phosphorylation is uncoupled, which one of the following actions takes place?
Question type: 1. Choosing the only correct answer
Question value (points): 1
Answer variants:
1. Phosphorylation of adenosine diphosphate (ADP) accelerates
2. Phosphorylation of ADP continues but oxygen uptake stops
3. Phosphorylation of ADP stops but oxygen uptake continues
4. Oxygen uptake stops
5. Both phosphorylation of ADP and oxygen uptake stop
Correct answer: 3 Answer variants: 5
QUESTION N 437. The chemiosmotic theory proposes that adenosine triphosphate is formed because of
which
one of the following reasons?
Question type: 1. Choosing the only correct answer
Question value (points): 1
Answer variants:
1. A change in the permeability of the unner mitochondrial membrane toward adenosine diphosphate (ADP)
2. The formation of high-energy bonds in mitochondrial proteins
3. ADP is pumped out of the matrix into the intermembrane space
4. A proton gradient is formed across the inner membrane
5. Protons are pumped into the mitochondrial matrix
Correct answer: 4 Answer variants: 5
QUESTION N 438. A patient is diagnosed with Kearns-Sayre syndrome. Some clinical manifestations of this
disorder may respond to therapeutic doses of
Question type: 1. Choosing the only correct answer
Question value (points): 1
Answer variants:
1. 2,4-dinitrophenol
2. coenzyme Q (ubiquinone)
3. oligomycin
4. flavin adenine dinucleotide (FAD)
5. adenosine diphosphate (ADP)
Correct answer: 2 Answer variants: 5
QUESTION N 439. Match the following descriptions with the correct component of the electron transfer chain
or oxidative phosphorylation system. It is the point of entry for electrons from succinate dehydrogenase.
Question type: 1. Choosing the only correct answer
Question value (points): 1
Answer variants:
1. Complex I
2. Complex II
3. Complex III
4. Complex IV
5. Adenosine triphosphate (ATP) synthase
6. Cytochrome
Correct answer: 2 Answer variants: 6
QUESTION N 440. Match the following descriptions with the correct component of the electron transfer chain
or oxidative phosphorylation system. It is inhibited by rotenone.
Question type: 1. Choosing the only correct answer
Question value (points): 1
Answer variants:
1. Complex I
2. Complex II
3. Complex III
4. Complex IV
5. Adenosine triphosphate (ATP) synthase
6. Cytochrome
Correct answer: 1 Answer variants: 6
QUESTION N 441. Match the following descriptions with the correct component of the electron transfer chain
or oxidative phosphorylation system. It is inhibited by cyanide.
Question type: 1. Choosing the only correct answer
Question value (points): 1
Answer variants:
1. Complex I
2. Complex II
3. Complex III
4. Complex IV
5. Adenosine triphosphate (ATP) synthase
6. Cytochrome
Correct answer: 4 Answer variants: 6
QUESTION N 442. Match the following descriptions with the correct component of the electron transfer chain
or oxidative phosphorylation system. It is inhibited by antimycin A.
Question type: 1. Choosing the only correct answer
Question value (points): 1
Answer variants:
1. Complex I
2. Complex II
3. Complex III
4. Complex IV
5. Adenosine triphosphate (ATP) synthase
6. Cytochrome
Correct answer: 3 Answer variants: 6
QUESTION N 443. Match the following descriptions with the correct component of the electron transfer chain
or oxidative phosphorylation system. It is the point of entry for most of the electrons generated by the action
of the citric acid cycle.
Question type: 1. Choosing the only correct answer
Question value (points): 1
Answer variants:
1. Complex I
2. Complex II
3. Complex III
4. Complex IV
5. Adenosine triphosphate (ATP) synthase
6. Cytochrome
Correct answer: 1 Answer variants: 6
QUESTION N 444. Match the following descriptions with the correct component of the electron transfer chain
or oxidative phosphorylation system. It is inhibited by oligomycin.
Question type: 1. Choosing the only correct answer
Question value (points): 1
Answer variants:
1. Complex I
2. Complex II
3. Complex III
4. Complex IV
5. Adenosine triphosphate (ATP) synthase
6. Cytochrome
Correct answer: 5 Answer variants: 6
QUESTION N 445. Match the following descriptions with the correct component of the electron transfer chain
or oxidative phosphorylation system. It is the only soluble component of the respiratory chain.
Question type: 1. Choosing the only correct answer
Question value (points): 1
Answer variants:
1. Complex I
2. Complex II
3. Complex III
4. Complex IV
5. Adenosine triphosphate (ATP) synthase
6. Cytochrome
Correct answer: 6 Answer variants: 6
QUESTION N 448. Put the listed steps of the PDH complex action in true order:
Question type: 3. Arranging the answers in the correct sequence
Question value (points): 1
Answer variants:
1. TPP forms a covalent bond with the alpha-carbon atom of pyruvate.
2. CO2 is released.
3. The acetyl group is transferred from lipoic acid to CoA.
4. The electrons are transferred from Lip (SH)2 to lipoamide dehydrogenase.
5. The electrons are transferred to NAD+
All answers are correct Answer variants: 5
QUESTION N 449. Each of the following statements concerning PDH complex is true, EXCEPT:
Question type: 1. Choosing the only correct answer
Question value (points): 1
Answer variants:
1. It is an example of multienzyme complex.
2. PDH complex produces oxaloacetate from pyruvate.
3. It is inhibited when NADH and acetyl CoA levels are increased.
4. Has two regulatory subunits.
5. It is converted to an active form by phosphorylation.
Correct answer: 2 Answer variants: 5
QUESTION N 450. Find WRONG answer. Pyruvate dehydrogenase activity is regulated by :
Question type: 1. Choosing the only correct answer
Question value (points): 1
Answer variants:
1. Covalent modification
2. Acceptor control
3. Product inhibition
4. NADH
5. Citric acid
Correct answer: 5 Answer variants: 5
QUESTION N 451. Choose the most correct answer. The principle function of the TCA cycle is to:
Question type: 1. Choosing the only correct answer
Question value (points): 1
Answer variants:
1. Generate CO2
2. Transfer electrons from acetyl-CoA to NAD and FAD
3. Generate heat from oxidation of the acetyl
4. Oxidize the acetyl portion of acetyl-CoA to oxaloacetate
5. Dispose the excess of the pyruvate and fatty acids
Correct answer: 2 Answer variants: 5
QUESTION N 452. Choose the correct answer. The reactions of TCA cycle oxidizing succinate to oxaloacetate:
Question type: 1. Choosing the only correct answer
Question value (points): 1
Answer variants:
1. Require CoA
2. Include an isomerization reaction
3. Produce one high- energy phosphate bond
4. Require both NAD+ and FAD+
5. Produce one mole GTP from GDP and Pi
Correct answer: 4 Answer variants: 5
QUESTION N 453. Choose the correct answer. The enzyme catalyzing an anaplerotic reaction for the TCA
cycle is:
Question type: 1. Choosing the only correct answer
Question value (points): 1
Answer variants:
1. Succinate dehydrogenase
2. Citrate lyase
3. Citrate synthetase
4. Pyruvate dehydrogenase
5. Pyruvate carboxylase
Correct answer: 5 Answer variants: 5
QUESTION N 455. Which one of the following conditions decreases the oxidation of acetyl- CoA by the citric
acid cycle?
Question type: 1. Choosing the only correct answer
Question value (points): 1
Answer variants:
1. A low ATP/ADP ratio
2. Low NADH due to rapid oxidation to NAD+ through ETC
3. A low NAD+ / NADH ratio
4. High concentration of AMP
5. Low GTP/GDP ratio.
Correct answer: 3 Answer variants: 5
QUESTION N 456. Find components of the PDH complex(3):
Question type: 2. Choosing several possible correct answers
Question value (points): 1
Answer variants:
1. Biotin
2. Thiamine pyrophosphate
3. Pyridoxal phosphate
4. NAD+
5. FAD+
Correct answers: 2; 4; 5Answer variants: 5
QUESTION N 457. The convertion of pyruvate to acetyl CoA and CO2 (4):
Question type: 2. Choosing several possible correct answers
Question value (points): 1
Answer variants:
1. Is catalyzed by multimolecular aggregate.
2. Involves the participation of lipoic acid.
3. Occurs in the cytosol
4. Depends on CoA.
5. Donates electrons to ETC.
Correct answers: 1; 2; 4; 5 Answer variants: 5
QUESTION N 458. Choose the correct answers (2). The isocitrate dehydrogenase is
Question type: 2. Choosing several possible correct answers
Question value (points): 1
Answer variants:
1. Inhibited by increased levels of ATP
2. Inhibited by decreased levels of NADH
3. Stimulated by a high-energy charge
4. Stimulated by increased levels of ADP
5. None of the above
Correct answers: 1; 4 Answer variants: 5
QUESTION N 459. Which one of the following enzymes is tightly associated with the inner mitochondrial
membrane?
Question type: 1. Choosing the only correct answer
Question value (points): 1
Answer variants:
1. Citrate synthase
2. alpha-Ketoglutarate dehydrogenase
3. Succinate dehydragenase
4. Fumarase
5. Malate dehydrogenase
Correct answer: 3 Answer variants: 5
QUESTION N 460. A patient is deficient in activities of both pyruvate dehydrogenase (PDH) and
alpha-ketoglutarate dehydrogenase. What is the most likely explanation for this finding?
Question type: 1. Choosing the only correct answer
Question value (points): 1
Answer variants:
1. The patient possesses a genetic defect in the E1 subunit of PDH
2. The patient possesses a genetic defect in the E2 subunit of PDH
3. The patient possesses a genetic defect in the E3 subunit of PDH
4. The decarboxylase component of each enzyme is defective
5. The dihydrolipoyl transacetylase component of each enzyme is defective
Correct answer: 3 Answer variants: 5
QUESTION N 461. Which one of the following compounds is formed directly in one or more reaction(s) of the
citric acid cycle?
Question type: 1. Choosing the only correct answer
Question value (points): 1
Answer variants:
1. Reduced nicotinamide adenine dinucleotide (NADH)
2. Triphosphates (GTP)
3. Both compounds
4. Neither
Correct answer: 3 Answer variants: 4
QUESTION N 462. Which one of the following compounds is required for acetyl coenzyme A (acetyl CoA) to
enter the citric acid cycle?
Question type: 1. Choosing the only correct answer
Question value (points): 1
Answer variants:
1. Isocitrate
2. Malate
3. Oxaloacetate
4. Pyruvate
5. Succinate
Correct answer: 3 Answer variants: 5
QUESTION N 463. A patient suffers from a genetic defect in the E2 subunit of the pyruvate dehydrogenase
(PDH) complex, which decreases its activity. Which one of the following is a likely symptom of this disorder?
Question type: 1. Choosing the only correct answer
Question value (points): 1
Answer variants:
1. Serum pH greater than 7.4
2. Above normal levels of serum lactate
3. Below normal levels of serum alanine
4. Below normal levels of serum pyruvate
Correct answer: 2 Answer variants: 4
QUESTION N 464. A patient suffers from a genetic defect in the E2 subunit of the pyruvate dehydrogenase
(PDH) complex, which decreases its activity. Which one of the following treatments is most likely to be
beneficial in this case?
Question type: 1. Choosing the only correct answer
Question value (points): 1
Answer variants:
1. Restriction of dietary protein
2. Restriction of dietary fats
3. Restriction of dietary carbohydrates
Correct answer: 3 Answer variants: 3
QUESTION N 465. A patient suffers from a genetic defect in the E2 subunit of the pyruvate dehydrogenase
(PDH) complex, which decreases its activity. Some patients with this genetic defect may respond to
therapeutic doses of
Question type: 1. Choosing the only correct answer
Question value (points): 1
Answer variants:
1. thiamine
2. lipolic acid
3. riboflavin
4. niacin
5. pantothenic acid
Correct answer: 2 Answer variants: 5
QUESTION N 466. The reaction that it catalyzes results in the formation of high-energy phosphate compound
Question type: 1. Choosing the only correct answer
Question value (points): 1
Answer variants:
1. Malate dehydrogenase
2. Succinyl CoA synthetase
3. Citrate synthase
4. Pyruvate dehydrogenase
5. Pyruvate carboxylase
Correct answer: 2 Answer variants: 5
QUESTION N 467. Its activity is regulated by covalent modification
Question type: 1. Choosing the only correct answer
Question value (points): 1
Answer variants:
1. Malate dehydrogenase
2. Succinyl CoA synthetase
3. Citrate synthase
4. Pyruvate dehydrogenase
5. Pyruvate carboxylase
Correct answer: 4 Answer variants: 5
QUESTION N 469. Find enzymes reducing nicotinamide adenine dinucleotide (2)
Question type: 2. Choosing several possible correct answers
Question value (points): 1
Answer variants:
1. Malate dehydrogenase
2. Succinyl CoA synthetase
3. Citrate synthase
4. Pyruvate dehydrogenase
5. Pyruvate carboxylase
Correct answers: 1; 4 Answer variants: 5
QUESTION N 470. It uses acetyl CoA as a substrate
Question type: 1. Choosing the only correct answer
Question value (points): 1
Answer variants:
1. Malate dehydrogenase
2. Succinyl CoA synthase
3. Citrate synthase
4. Pyruvate dehydrogenase
5. Pyruvate carboxylase
Correct answer: 3 Answer variants: 5
QUESTION N 471. Match the number and the letter:
Question type: 4. Arranging the correspondences for the answers
Question value (points): 1
Answer variants:
1. Salivary alphaamilase
2. Pancreatic alphaamilase
3. Both
4. None
Answer variants: 4
Correspondence variants:
A. Cleaves alpha1,6-bonds.
B. Exhibits the most activity at pH 8.0.
C. The major products of hydrolysis are disaccharides
D. Is hydrolase.
Correct answers: 1-B; 2-C; 3-D; 4-A Correspondence variants: 4
QUESTION N 472. Choose the enzymes that hydrolyze bonds between the units
Question type: 4. Arranging the correspondences for the answers
Question value (points): 1
Answer variants:
1. Maltase
2. Isomaltase
3. Lactase
4. Sucrase
Answer variants: 4
Correspondence variants:
A. Glucose (alpha1-4) glucose
B. Galactose (beta1-4) glucose
C. Glucose (alpha1-6) glucose
D. Glucose (alpha 1-2) fructose
Correct answers: 1-A; 2-C; 3-B; 4-D Correspondence variants: 4
QUESTION N 473. Choose the correct statements (4) about sucrase-isomaltase complex
Question type: 2. Choosing several possible correct answers
Question value (points): 1
Answer variants:
1. Is synthesized by intestinal brush-border epithelium
2. Hydrolyzes alpha-1,4bonds
3. Catalyses glucose formation
4. Catalyses fructose formation
5. Optimal pH 5,0
Correct answers: 1; 2; 3; 4 Answer variants: 5
QUESTION N 474. Match the number and the letter:
Question type: 4. Arranging the correspondences for the answers
Question value (points): 1
Answer variants:
1. Glucose
2. Galactose
3. Both
4. None
Answer variants: 4
Correspondence variants:
A. The product of sucrose digestion
B. Absorption occurs by simple diffusion
C. The product of lactose digestion
D. Moves across the membrane by using concentration gradient in Na
Correct answers: 1-A; 2-C; 3-D; 4-B Correspondence variants: 4
QUESTION N 475. Match the number and the letter:
Question type: 4. Arranging the correspondences for the answers
Question value (points): 1
Answer variants:
1. Fructose
2. Galactose
3. Both
4. None
Answer variants: 4
Correspondence variants:
A. The monomer of starch
B. Moves across membrane by transporter proteins
C. Moves across membrane by active transport
D. The product of digestion of sucrose
Correct answers: 1-D; 2-C; 3-B; 4-A Correspondence variants: 4
QUESTION N 476. Match the correct couples:
Question type: 4. Arranging the correspondences for the answers
Question value (points): 1
Answer variants:
1. Glucose transport into muscle and adipose cells
2. Glucose transport into brain and liver cells
3. Both
4. None
Answer variants: 4
Correspondence variants:
A. Is independent on insulin
B. Requires transport proteins
C. Requires Na+,K+ ATPase pump
D. The mechanism involves the recruitment of glucose transporters from intracellular vesicles into the plasma
membrane.
Correct answers: 1-D; 2-A; 3-B; 4-C Correspondence variants: 4
QUESTION N 477. Glycogen is stored in the cells as
Question type: 1. Choosing the only correct answer
Question value (points): 1
Answer variants:
1. A component of endoplasmic reticulum membranes
2. Granules, which also contain the enzymes that catalyze their formation and degradation.
3. A component of the Golgi apparatus, where it is formed.
4. Free glycogen in solution in the cvtosol.
Correct answer: 2 Answer variants: 4
QUESTION N 478. The greatest amount of body glycogen can be found in which of the following human
tissues?
Question type: 1. Choosing the only correct answer
Question value (points): 1
Answer variants:
1. Liver
2. Kidney
3. Skeletal muscle
4. Cardiac muscle
5. Adipose tissue
Correct answer: 3 Answer variants: 5
QUESTION N 479. Which of the following enzymes (2) are used both in gluconeogenesis and glycolysis?
Question type: 2. Choosing several possible correct answers
Question value (points): 1
Answer variants:
1. Glucokinase
2. Hexokinase
3. Phosphofructokinase 1
4. Aldolase
5. Phosphoglucomutase
Correct answers: 4; 5 Answer variants: 5
QUESTION N 480. The conversion of blood glucose to glycogen (4)
Question type: 2. Choosing several possible correct answers
Question value (points): 1
Answer variants:
1. Involves the net consumption of 2 high energy phosphate bonds.
2. Requires UDP-glucose as a precursor.
3. Requires the formation of both alpha-1,4 bonds and alpha-1,6 bonds.
4. Requires glycogen primer for initiation of glycogen synthesis.
5. Occurs in cytosol.
Correct answers: 2; 3; 4; 5 Answer variants: 5
QUESTION N 481. Glycogen synthase is characterized by all of the following statements EXCEPT
Question type: 1. Choosing the only correct answer
Question value (points): 1
Answer variants:
1. Uridine diphosphate glucose is a substrate.
2. Requires a primer strand of glycogen.
3. Catalyses the regulatory step.
4. Attaches the glucosyl residues in 1,6-bonds.
5. Can not initiate synthesis.
Correct answer: 4 Answer variants: 5
QUESTION N 482. Match the correct couples:
Question type: 4. Arranging the correspondences for the answers
Question value (points): 1
Answer variants:
1. Debranching enzyme.
2. Glycogen phosphorylase
3. Both.
4. None.
Answer variants: 4
Correspondence variants:
A. Cleaves alpha-1,4-glycosidic bonds
B. Cleaves alpha-l,6-glycosidic bonds
C. Results in free glucose.
D. Results in glucose-1-phosphate.
Correct answers: 1-B; 2-A; 3-D; 4-C Correspondence variants: 4
QUESTION N 483. All of the following statements about glycogen degradation are true EXCEPT?
Question type: 1. Choosing the only correct answer
Question value (points): 1
Answer variants:
1. Glycogen phosphorylase releases G-l-P.
2. Liver glycogen is completely degraded in 12 hours.
3. The debranching enzyme has two catalytic activities.
4. Glycogen phosphorylase can not act on the closest glycosidic bonds to a branchpoint.
5. Liver glycogen is a source of glucose during fasting state.
Correct answer: 4 Answer variants: 5
QUESTION N 484. Free glucose is formed from
Question type: 1. Choosing the only correct answer
Question value (points): 1
Answer variants:
1. Glucose residues in 1,4 linkage to the main chain.
2. Glucose residues in 1,6 linkage to the main chain.
3. Glucose-1-phosphate.
4. Glucose 6-phosphate.
5. Breakdown of uridine diphosphate glucose
Correct answer: 4 Answer variants: 5
QUESTION N 485. Which of the following statements (4) regarding the regulation of glycogen metabolism in
liver are correct?
Question type: 2. Choosing several possible correct answers
Question value (points): 1
Answer variants:
1. Liver glycogen is synthesized when blood glucose level is elevated.
2. The fall of the insulin/glucagon ratio results in activation of the glycogen synthesis.
3. Liver glycogen is degraded to free glucose.
4. Protein kinase A and protein kinase C inactivate glycogen synthase.
5. Glucogen phosphorylase becomes active due to phosphorylation.
Correct answers: 1; 3; 4; 5 Answer variants: 5
QUESTION N 486. Glycogen synthesis in liver is stimulated by
Question type: 1. Choosing the only correct answer
Question value (points): 1
Answer variants:
2. Phosphorylation of glycogen synthase.
3. Dephosphorylation of phosphorylase kinase
4. Epinephrine.
5. Phosphodiesterase activation
Correct answer: 5 Answer variants: 4
QUESTION N 487. When a normal individual in the basal metabolic state ingests a high-carbohydrate meal,
there is (3)
Question type: 2. Choosing several possible correct answers
Question value (points): 1
Answer variants:
1. Enhanced glycogen synthase activity in liver.
2. An increased ratio of phosphorylase a to phosphorylase b in liver.
3. An increased insulin/glucagon ratio.
4. Phosphorylation of glycogen synthase in liver.
5. Dephosphorylation of glycogen synthase.
Correct answers: 1; 3; 5Answer variants: 5
QUESTION N 488. Which of the following phosphorylated enzymes are active (2)?
Question type: 2. Choosing several possible correct answers
Question value (points): 1
Answer variants:
1. Phosphorylase kinase.
2. Glycogen phosphorylase
3. Glycogen synthase .
4. Pyruvate kinase.
5. Pyruvate dehydrogenase.
Correct answers: 1; 2 Answer variants: 5
QUESTION N 489. Which of the following statements regarding the regulation of glycogenolysis is correct?
Question type: 1. Choosing the only correct answer
Question value (points): 1
Answer variants:
1. cAMP enhances glycogenolysis by adenylation of glycogen phosphorylase.
2. Glycogen phosphorylase is activated by phosphorylation.
3. Phosphorylase kinase is inactivated by phosphorylation by cAMP.
4. Muscle phosphorylase kinase is inactivated by Ca2+
5. Glycogen synthase is phosphorylated at three sites.
Correct answer: 2 Answer variants: 5
QUESTION N 490. An adolescent patient with deficiency of muscle phosphorylase was examined while
exercising her forearm by squeezing a rubber ball. Compared to a normal person performing the same
exercise, this patient:
Question type: 1. Choosing the only correct answer
Question value (points): 1
Answer variants:
1. Could exercise for a longer period of time without fatigue.
2. Had increased glucose level in blood drawn from her forearm.
3. Had decreased lactate level in blood drawn from her forearm.
4. Had lower level of glycogen on biopsies of her forearm muscle.
Correct answer: 4 Answer variants: 4
QUESTION N 491. Match the correct couples
Question type: 4. Arranging the correspondences for the answers
Question value (points): 1
Answer variants:
1. Epinephrine
2. Glucagon
3. Both
4. None
Answer variants: 4
Correspondence variants:
A. Acts in liver and muscle
B. Acts in liver only
C. Stimulates glycogen synthesis
D. Transmits a signal through G protein
Correct answers: 1-A; 2-B; 3-D; 4-C Correspondence variants: 4
QUESTION N 492. An adolescent patient with a deficiency of muscle phosphorylase was examined while
exercising her forearm by squeezing a rubber ball. Compared with a normal person performing the same
exercise, this patient
Question type: 1. Choosing the only correct answer
Question value (points): 1
Answer variants:
1. could exercise for a longer period of time without fatigue
2. had increased glucose levels in blood drawn from her forearm
3. had decreased lactate levels in blood drawn from her forearm
4. had lower levels of glycogen in biopsies of her forearm muscle
Correct answer: 3 Answer variants: 4
QUESTION N 493. In which compartment of the cell does glycolysis occur?
Question type: 1. Choosing the only correct answer
Question value (points): 1
Answer variants:
1. Mitochondrium
2. Nucleus
3. Soluble cytoplasm (cytosol)
4. Rough endoplasmic reticulum
5. Smooth endoplasmic reticulum
Correct answer: 3 Answer variants: 5
QUESTION N 494. What type of bond is formed between phosphate and carbon 1 of 1,3-bisphosphoglycerate?
Question type: 1. Choosing the only correct answer
Question value (points): 1
Answer variants:
1. Anchydride
2. Ester
3. Phosphodiester
4. Amide
5. Ether
Correct answer: 1 Answer variants: 5
QUESTION N 495. Which of the following statements about glycolysis is true?
Question type: 1. Choosing the only correct answer
Question value (points): 1
Answer variants:
1. Glucokinase catalyzes the conversion of glucose to glucose 6-phosphate in the liver
2. Phosphofructokinase 1 catalyzes the conversion of fructose 1,6-bisphosphate to dihydroxyacetone phosphate
3. When one molecule of glucose is converted to pyruvate via glycolysis, one molecule of NAD+ is reduced
4. When one molecule of glucose is converted to pyruvate via glycolysis, one carbon is lost as CO2
5. Hexokinase catalyzes the conversion of fructose 6-phosphate to fructose 1,6-bisphosphate
Correct answer: 1 Answer variants: 5
QUESTION N 496. A positive allosteric activator of phosphofructokinase 1 in the liver is
Question type: 1. Choosing the only correct answer
Question value (points): 1
Answer variants:
1. ADP
2. acetyl CoA
3. fructose 2,6-bisphosphate
4. ATP
5. citrate
Correct answer: 3 Answer variants: 5
QUESTION N 497. Which of the following is a regulatory mechanism of glycolysis?
Question type: 1. Choosing the only correct answer
Question value (points): 1
Answer variants:
1. Inhibition of phosphofructokinase 1 by AMP
2. Inhibition of hexokinase by its product
3. Activation of pyruvate kinase when glucagon levels are elevated
4. Inhibition of aldolase by fructose 1,6-bisphosphate
5. Inhibition of glucokinase by fructose 2,6-bisphosphate
Correct answer: 2 Answer variants: 5
QUESTION N 498. An alcoholic went on a weekend binge. The metabolism of ethanol produces NADH, mainly
in the liver. As a result of high NADH levels, pyruvate is converted to
Question type: 1. Choosing the only correct answer
Question value (points): 1
Answer variants:
1. oxaloacetate
2. acetyl CoA
3. phosphoenolpyruvate
4. lactate
Correct answer: 4 Answer variants: 4
QUESTION N 499. Which of the following glycolytic enzymes is used in gluconeogenesis?
Question type: 1. Choosing the only correct answer
Question value (points): 1
Answer variants:
1. Glucokinase
2. Phosphofructokinase 1
3. Pyruvate kinase
4. Aldolase B
Correct answer: 4 Answer variants: 4
QUESTION N 500. In the conversion of pyruvate to glucose during gluconeogenesis,
Question type: 1. Choosing the only correct answer
Question value (points): 1
Answer variants:
1. biotin is required
2. CO2, added in one reaction, appears in the final product
3. energy is utilized only in the form of GTP
4. all of the reactions occur in the cytosol
Correct answer: 1 Answer variants: 4
QUESTION N 501. In gluconeogenesis, both alanine and lactate are converted in a single step to
Question type: 1. Choosing the only correct answer
Question value (points): 1
Answer variants:
1. oxaloacetate
2. acetyl CoA
3. phosphoenolpyruvate
4. pyruvate
5. aspartate
Correct answer: 4 Answer variants: 5
QUESTION N 502. A common intermediate in the conversion of glycerol and lactate to glucose is
Question type: 1. Choosing the only correct answer
Question value (points): 1
Answer variants:
1. pyruvate
2. oxaloacetate
3. malate
4. glucose 6-phosphate
5. phosphoenolpyruvate
Correct answer: 4 Answer variants: 5
QUESTION N 503. In which of the following compounds do carbons derived from pyruvate leave the
mitochondria for the synthesis of glucose during fasting?
Question type: 1. Choosing the only correct answer
Question value (points): 1
Answer variants:
1. malate
2. acetyl CoA
3. oxaloacetate
4. lactate
5. glutamine
Correct answer: 1 Answer variants: 5
QUESTION N 504. The pentose phosphate pathway generates
Question type: 1. Choosing the only correct answer
Question value (points): 1
Answer variants:
1. NADH, which may be used for fatty acid synthesis
2. ribose 5-phosphate, which may be used for the biosynthesis of ATP
3. pyruvate and fructose 1,6-bisphosphate by the transaldolase and transketolase reactions
4. xylulose 5-phosphate by one of the oxidative reactions
5. glucose from ribose 5-phosphate and CO2
Correct answer: 2 Answer variants: 5
QUESTION N 505. Choose the correct answer: The prostaglandins are synthesized from:
Question type: 1. Choosing the only correct answer
Question value (points): 1
Answer variants:
1. Arachidonic acid
2. Oleic acid
3. Linolic acid
4. Linolenic acid
Correct answer: 1 Answer variants: 4
QUESTION N 506. Choose the correct answer: Arachidonic acid contains the number of double bonds:
Question type: 1. Choosing the only correct answer
Question value (points): 1
Answer variants:
1. 2
2. 3
3. 4
4. 5
Correct answer: 3 Answer variants: 4
QUESTION N 507. All of following statements about features of structure of fatty acids in the human are true
EXCEPT:
Question type: 1. Choosing the only correct answer
Question value (points): 1
Answer variants:
1. Fatty acids are hydrocarbon derivatives
2. Their chain is unbranched
3. The double bonds of polyunsatured fatty acids are almost never conjugated, but are separated by a methylene
group
4. The double bonds are in cis configuration
5. The double bonds are in trans configuration
6. The nonpolar hydrocarbon chain accounts for the poor solubility of fatty acids in water
7. Fatty acids contain even number of carbon atoms
Correct answer: 5 Answer variants: 7
QUESTION N 508. Match the letter and the number:
Question type: 4. Arranging the correspondences for the answers
Question value (points): 1
Answer variants:
1. Phosphoacylglycerols
2. Triacylglycerols
Answer variants: 2
Correspondence variants:
A. Neutral lipids
B. Storage lipids
C. Polar lipids
D. Membrane lipids
Correct answers: 1-C, D; 2-A, B Correspondence variants: 4
QUESTION N 509. Lecithin contains nitrogenous base named
Question type: 1. Choosing the only correct answer
Question value (points): 1
Answer variants:
1. Ethanolamine
2. Choline
3. Inositol
4. All of the above
Correct answer: 2 Answer variants: 4
QUESTION N 510. Lecithins contain an unsaturated fatty acid at position
Question type: 1. Choosing the only correct answer
Question value (points): 1
Answer variants:
1. 1-
2. 1- and 2-
3. 2-
4. None of the above
Correct answer: 3 Answer variants: 4
QUESTION N 511. Choose the correct answer: The essential fatty acids retard
Question type: 1. Choosing the only correct answer
Question value (points): 1
Answer variants:
1. Atherosclerosis
2. Diabetes Mellitus
3. Nephritis
4. Bronchitis
Correct answer: 1 Answer variants: 4
QUESTION N 512. The polyunsatured fatty acids which are not synthesized in the body but are taken from
natural sources are called ...
Question type: 5. Entering the answer manually using the keyboard
Question value (points): 1
Answer variants:
1. essential
2. Essential
All answers are correct Answer variants: 2
QUESTION N 513. The polyunsaturated fatty acids can ... serum cholesterol level.
Question type: 5. Entering the answer manually using the keyboard
Question value (points): 1
Answer variants:
1. decrease
2. Decrease
All answers are correct Answer variants: 2
QUESTION N 514. Choose the correct answer: Lipase present in the stomach cannot hydrolyze fats owing to:
Question type: 1. Choosing the only correct answer
Question value (points): 1
Answer variants:
1. Alkalinity
2. Acidity
3. High acidity
4. Neutrality
Correct answer: 3 Answer variants: 4
QUESTION N 515. Before the action of lipase the fat is emulsified by:
Question type: 1. Choosing the only correct answer
Question value (points): 1
Answer variants:
1. Lipoproteins
2. Phospholipids
3. Bile salts
4. Ergosterols
Correct answer: 3 Answer variants: 4
QUESTION N 516. Choose the correct answer: Phospholipase A1 attacks the ester bond of phospholipids in
position
Question type: 1. Choosing the only correct answer
Question value (points): 1
Answer variants:
1. 1
2. 2
3. 3
4. All of the above
Correct answer: 1 Answer variants: 4
QUESTION N 517. Choose the correct answer: Phospholipase C release, 1,2-diacylglycerol and phosphoryl
base attack the ester bond in position
Question type: 1. Choosing the only correct answer
Question value (points): 1
Answer variants:
1. 1
2. 2
3. 3
4. 4
Correct answer: 3 Answer variants: 4
QUESTION N 518. The correct answer: Bile acids are synthesized from cholesterol in the:
Question type: 1. Choosing the only correct answer
Question value (points): 1
Answer variants:
1. Duodenum
2. Intestine
3. Gall-bladder
4. Liver
Correct answer: 4 Answer variants: 4
QUESTION N 519. Choose the correct answer: In the human bile, sodium glycocholate concentration is
greater than sodium taurocholate in:
Question type: 1. Choosing the only correct answer
Question value (points): 1
Answer variants:
1. Two times
2. Three times
3. Four times
4. Five times
Correct answer: 1 Answer variants: 4
QUESTION N 520. Choose the correct answer: Bile salts activate
Question type: 1. Choosing the only correct answer
Question value (points): 1
Answer variants:
1. Pancreatic lipase
2. Cholesterol esterase
3. Both
4. None
Correct answer: 3 Answer variants: 4
QUESTION N 521. All of following statements about biological role of bile salts in the human are true
EXCEPT:
Question type: 1. Choosing the only correct answer
Question value (points): 1
Answer variants:
1. Bile salts emulsify fats increasing area surface.
2. Bile salts activate pancreatic lipase.
3. They form micelles and help in their absorption in the intestines.
4. Bile salts are the bile pigments.
5. Bile salts stimulate intestinal peristalsis.
Correct answer: 4 Answer variants: 5
QUESTION N 522. Choose the correct answer: The great majority of absorbed fat appears in form of:
Question type: 1. Choosing the only correct answer
Question value (points): 1
Answer variants:
1. HDL
2. Chylomicrons
3. VLDL
4. LDL
Correct answer: 2 Answer variants: 4
QUESTION N 523. Choose the correct answer: Chylomicrons and VLDL both are released from the intestine or
hepatic cell by reverse:
Question type: 1. Choosing the only correct answer
Question value (points): 1
Answer variants:
1. Pinocytosis
2. Diffusion
3. Osmosis
4. Passive diffusion
Correct answer: 2 Answer variants: 4
QUESTION N 524. Choose the correct answer: HDL is synthesized and secreted from:
Question type: 1. Choosing the only correct answer
Question value (points): 1
Answer variants:
1. Pancreas
2. Liver
3. Kidney
4. Muscle
Correct answer: 2 Answer variants: 4
QUESTION N 525. Match the letter and the number:
Question type: 4. Arranging the correspondences for the answers
Question value (points): 1
Answer variants:
1. VLDL
2. LDL
3. HDL
Answer variants: 3
Correspondence variants:
A. beta-lipoproteins
B. pre-beta-lipoproteins
C. alpha-lipoproteins
Correct answers: 1-B; 2-A; 3-C Correspondence variants: 3
QUESTION N 526. Origin of fatty acids as substrates for beta-oxidation:
Question type: 1. Choosing the only correct answer
Question value (points): 1
Answer variants:
1. Fatty acids are derived from the diet
2. Fatty acids are synthesized in the liver from glucose
3. Fatty acids are products of mobilization of adipose triacylglycerols
4. All of the above
Correct answer: 4 Answer variants: 4
QUESTION N 527. Match the letter with the number: Fatty acids may be activated in:
Question type: 4. Arranging the correspondences for the answers
Question value (points): 1
Answer variants:
1. Shot chain fatty acids
2. Medium chain fatty acids
3. Long chain fatty acids
Answer variants: 3
Correspondence variants:
A. Endoplasmatic reticulum
B. Cytosol or mitochondria
C. Mitochondrial matrix
Correct answers: 1-B; 2-C; 3-A Correspondence variants: 3
QUESTION N 528. The enzymes of beta-oxidation are located in:
Question type: 1. Choosing the only correct answer
Question value (points): 1
Answer variants:
1. Cytosol
2. Mitochondrial inner membrane
3. Mitochondrial outer membrane
4. Mitochondrial matrix
Correct answer: 4 Answer variants: 4
QUESTION N 529. Beta-oxidation occurs in all tissues EXCEPT (2):
Question type: 2. Choosing several possible correct answers
Question value (points): 1
Answer variants:
1. Liver
2. Brain
3. Kidneys
4. Erythrocyte
5. Muscle
Correct answers: 2; 4 Answer variants: 5
QUESTION N 530. All of the following statements about ketone bodies are true EXCEPT:
Question type: 1. Choosing the only correct answer
Question value (points): 1
Answer variants:
1. Ketone bodies are formed in liver but utilized in the extrahepatic tissue.
2. Ketone bodies are formed and utilized in liver.
3. Acetoacetate is reduced to beta-hydroxybutyrate.
4. Acetoacetate is decarboxylated to acetone.
5. Two reactions occur in extrahepatic tissue for the activation of acetoacetate to acetoacetyl CoA.
Correct answer: 2 Answer variants: 5
QUESTION N 531. Which of the following compound are precursors for ketone bodies?
Question type: 1. Choosing the only correct answer
Question value (points): 1
Answer variants:
1. Amino acids (Ala, Asp, Glu, His)
2. Glucose
3. Fatty acids
4. All of above
Correct answer: 4 Answer variants: 4
QUESTION N 532. Match the letter and the number:
Question type: 4. Arranging the correspondences for the answers
Question value (points): 1
Answer variants:
1. Synthesis of ketone bodies
2. Oxidation of ketone bodies
3. Both
4. None of the above
Answer variants: 4
Correspondence variants:
A. D-beta-hydroxybutyrate dehydrogenase
B. HMG-CoA synthethase
C. Enoyl CoA hydratase
D. Succinyl CoA: acetoacetate CoA transferase
Correct answers: 1-B; 2-D; 3-A; 4-C Correspondence variants: 4
QUESTION N 533. Match the letter and the number:
Question type: 4. Arranging the correspondences for the answers
Question value (points): 1
Answer variants:
1. TCA cycle
2. Synthesis of fatty acids
3. Synthesis of cholesterol
Answer variants: 3
Correspondence variants:
A. Deficiency of NADPH
B. Deficiency of oxaloacetate
Correct answers: 1-A; 2-B; 3-A Correspondence variants: 2
QUESTION N 534. Biological role of cholesterol:
Question type: 2. Choosing several possible correct answers
Question value (points): 1
Answer variants:
1. It is structural component of membrane
2. It is an energy source for cells
3. It is a precursor for bile salts
4. It is a precursor for steroid hormones
5. It is assists in formation of vitamin D3 in skin
6. All of the above
Correct answers: 1; 3; 4; 5 Answer variants: 6
QUESTION N 535. Specify the correct order of stages of cholesterol synthesis.
Question type: 3. Arranging the answers in the correct sequence
Question value (points): 1
Answer variants:
1. Formation of mevalonate
2. Condensation of six isoprene units to give C30 terpene (squalene)
3. Cyclization of squalene to lanosterol
All answers are correct Answer variants: 3
QUESTION N 536. Match the letter and the number:
Question type: 4. Arranging the correspondences for the answers
Question value (points): 1
Answer variants:
1. Condensation of six isoprene units to give C30 terpene (squalene)
2. Cyclization of squalene to lanosterol
3. Formation of mevalonate
Answer variants: 3
Correspondence variants:
A. HMG-CoA reductase
B. Mevalonate kinase
C. Cyclase
Correct answers: 1-B; 2-C; 3-A Correspondence variants: 3
QUESTION N 537. The enzymes of cholesterol synthesis are located in:
Question type: 1. Choosing the only correct answer
Question value (points): 1
Answer variants:
1. The cytosol
2. Mitochondrial inner membrane
3. Mitochondrial outer memebrane
4. Mitochondrial matrix
Correct answer: 1 Answer variants: 4
QUESTION N 538. The highest contents of cholesterol is found in:
Question type: 1. Choosing the only correct answer
Question value (points): 1
Answer variants:
1. VLDL
2. LDL
3. HDL
4. Chylomicrons
Correct answer: 2 Answer variants: 4
QUESTION N 539. Match the letter and the number:
Question type: 4. Arranging the correspondences for the answers
Question value (points): 1
Answer variants:
1. VLDL is converted to IDL
2. Recognize apoE, apoB-100
3. Esterify cholesterol
4. Recognize oxidized LDL
Answer variants: 4
Correspondence variants:
A. Lipoprotein lipase
B. Lecithin: cholesterol acyltransferase
C. "Scavenger" receptor
D. LDL receptor
Correct answers: 1-A; 2-D; 3-B; 4-C Correspondence variants: 4
QUESTION N 540. Cholesterol synthesis "for export" occurs in tissues (2):
Question type: 2. Choosing several possible correct answers
Question value (points): 1
Answer variants:
1. Muscle
2. Intestine
3. Nervous tissue
4. Liver
5. Adipose tissue
Correct answers: 2; 4 Answer variants: 5
QUESTION N 541. Compare the formation, transport and metabolism of exogenes and endogenes cholesterol:
Question type: 4. Arranging the correspondences for the answers
Question value (points): 1
Answer variants:
1. Exogenes cholesterol
2. Endogenes cholesterol
Answer variants: 2
Correspondence variants:
A. Joins in lipoproteins forming in intestinal epithelial cells
B. Joins in VLDL forming in liver
C. Is secreted into the lymph in composition of chylomicrons
D. Is secreted into the blood directly
Correct answers: 1-A, C; 2-B, D Correspondence variants: 4
QUESTION N 542. Specify the correct order of stages of cholesterol transport from intestine to liver.
Question type: 3. Arranging the answers in the correct sequence
Question value (points): 1
Answer variants:
1. Hydrolysis of dietary cholesterol esters
2. Formation of mixed micelles
3. Absorption
4. Formation of chylomicrons
5. Lipoprotein lipase action
6. Formation of chylomicron remnants
7. Remnants are taken up by liver
All answers are correct Answer variants: 7
QUESTION N 543. Specify the correct order of stages of cholesterol transport from liver to peripheral tissues.
Question type: 3. Arranging the answers in the correct sequence
Question value (points): 1
Answer variants:
1. Cholesterol and its ester synthesis
2. Packing into VLDL (organ in which it is main formation)
3. Transport by blood
4. Formation of LDL
5. Lipoproteins are taken up by receptors of tissue
6. Cholesterol enters different tissues
All answers are correct Answer variants: 6
QUESTION N 544. Match the letter and the number. Compare the properties of lipases.
Question type: 4. Arranging the correspondences for the answers
Question value (points): 1
Answer variants:
1. Class Hydrolases
2. Class Lyases
3. Digests dietary fats
4. Digesrs TG into chylomicrons, VLDL
5. It is activated by glucagon, epinephrine
6. It is activated by bile salts
7. It is located in capillary walls
8. It is located in adipose tissue
Answer variants: 8
Correspondence variants:
A. Pancreatic lipase
B. TG lipase
C. LPL lipase
D. All of the above
E. None of the above
Correct answers: 1-D; 2-E; 3-A; 4-C; 5-B; 6-A; 7-C; 8-B Correspondence variants: 5
QUESTION N 545. Eicosanoids are produced from fatty acids released from membrane phospholipids by:
Question type: 1. Choosing the only correct answer
Question value (points): 1
Answer variants:
1. Phospholipase A1
2. Phospholipase A2
3. Phospholipase C
4. Phospholipase D
Correct answer: 2 Answer variants: 4
QUESTION N 546. Match the letter and the number. Pathways for eicosanoid synthesis.
Question type: 4. Arranging the correspondences for the answers
Question value (points): 1
Answer variants:
1. Cyclooxygenase pathway
2. Lipooxygenase pathway
Answer variants: 2
Correspondence variants:
A. Leikotrienes
B. Thromboxanes
C. Prostaglandins
D. Prostacyclin
Correct answers: 1-B, C, D; 2-A Correspondence variants: 4
QUESTION N 547. All of the following statements about eicosanoids are true EXCEPT:
Question type: 1. Choosing the only correct answer
Question value (points): 1
Answer variants:
1. They are derived from polyunsaturated fatty acids containing C20 atoms.
2. Many of them have very short half-lives in the range of minutes or less.
3. Many of them have half-lives in the range of minutes or hours.
4. They have a variety of extremely potent hormonelike actions.
5. They are not stored in cells.
6. They have different target cells and biological actions.
7. They interact with a specific receptor on the plasma membrane of a target cell.
Correct answer: 3 Answer variants: 7
QUESTION N 548. In the human, prostaglandins can be derived from:
Question type: 1. Choosing the only correct answer
Question value (points): 1
Answer variants:
1. glucose
2. acetyl CoA
3. arachidonic acid
4. oleic acid
5. leukotrienes
Correct answer: 3 Answer variants: 5
QUESTION N 549. A cyclooxygenase, which is inhibited by aspirin, is required for the production of:
Question type: 1. Choosing the only correct answer
Question value (points): 1
Answer variants:
1. thromboxanes from arachidonic acid
2. leukotrienes from arachidonic acid
3. phospholipids from arachidonic acid
4. arachidonic acid from linoleic acid
Correct answer: 1 Answer variants: 4
QUESTION N 550. Which one of the following statements about the conversation of fatty acids to ketone
bodies is TRUE?
Question type: 1. Choosing the only correct answer
Question value (points): 1
Answer variants:
1. Carnitine transports the fatty acid across the plasma membrane
2. Activation of the fatty acid is driven by the conversion of ATP to ADP
3. Thiolase cleaves HMG CoA
4. Acetoacetate and acetyl CoA are produced by cleavage of HMG CoA
5. The complete sequence of reactions occurs in all tissues of the body
Correct answer: 4 Answer variants: 5
QUESTION N 551. Match the amino acid abbreviations and their structural formulae
Question type: 4. Arranging the correspondences for the answers
Question value (points): 1
Answer variants:
1. Asp
2. Ser
3. Arg
4. Val
5. Ala
6. Lys
Answer variants: 6
Correspondence variants:
A. (A)
B. (B)
C. (C)
D. (D)
E. (E)
F. (F)
Correct answers: 1-D; 2-A; 3-E; 4-B; 5-F; 6-C Correspondence variants: 6
QUESTION N 552. Match the amino acids and the properties of their radicals
Question type: 4. Arranging the correspondences for the answers
Question value (points): 1
Answer variants:
1. Proline
2. Arginine
3. Glutamine
4. Aspartate
Answer variants: 4
Correspondence variants:
A. Conteins nonpolar radical
B. Conteins a carboxil group in its radical
C. Conteins an amide group in its side chain
D. Has positive charge at physiological pH
Correct answers: 1-A; 2-D; 3-C; 4-B Correspondence variants: 4
QUESTION N 553. Match the amino acids and the properties of their radicals
Question type: 4. Arranging the correspondences for the answers
Question value (points): 1
Answer variants:
1. Cysteine
2. Serine
3. Isoleucine
4. Glycine
Answer variants: 4
Correspondence variants:
A. Conteins a hydroxyl group in its side chain
B. Can form disulfide bonds
C. Contains the smallest side chain
D. Contains nonpolar radical
Correct answers: 1-B; 2-A; 3-D; 4-C Correspondence variants: 4
QUESTION N 554. Match the correct statements about peptides below
Question type: 4. Arranging the correspondences for the answers
Question value (points): 1
Answer variants:
1. Ile-Gly-Pro-Thr
2. Phe-Arg-Leu-Asp
3. Pro-Met-Val-Ala
4. Gln-Phe-Asn-Cys
Answer variants: 4
Correspondence variants:
A. All of its radicals are hydrophobic
B. It contains two amides
C. It contains C-terminal with a negatively side chain
D. It contains an amino acid with hydroxyl group
Correct answers: 1-D; 2-C; 3-A; 4-B Correspondence variants: 4
QUESTION N 555. Select the appropriate characteristics for protein structure
Question type: 4. Arranging the correspondences for the answers
Question value (points): 1
Answer variants:
1. Secondary structure
2. Tertiary structure
3. Both (in tertiary and secondary)
4. No correspondence
Answer variants: 4
Correspondence variants:
A. The order of sequence of amino acids in polypeptide chain
B. The spatial strucyure of protein
C. The conformation which is stabilized by interactions between amino acids radicals
D. The conformation of polypeptide chain as alfa-helix or beta sheeets
Correct answers: 1-D; 2-B; 3-C; 4-A Correspondence variants: 4
QUESTION N 556. CHOOSE ONE INCORRECT ANSWER. The spatial structure of a protein is formed by:
Question type: 1. Choosing the only correct answer
Question value (points): 1
Answer variants:
1. The bonds between the alfa-amino and alfa-carboxil groups of amino acids
2. Hydrogen bonds between the amino acid radicals
3. Hydrogen bonds between the atoms of peptide backbone
4. Hydrophobic interactions between the amino acid radicals
5. Interactions beetween the carboxyl and amino groups of amino acid radicals
Correct answer: 1 Answer variants: 5
QUESTION N 557. CHOOSE THE BEST ANSWER. Proteins are effective buffers because they contain:
Question type: 1. Choosing the only correct answer
Question value (points): 1
Answer variants:
1. A large number of amino acids
2. Amino acids residues with different pK
3. N-terminal and C-terminal residues that donate and accept protons
4. Peptide bond that is readily hydrolyzed, consuming hydrogen and hydroxyl ions
5. A large number of hydrogen bonds in alfa-helix
Correct answer: 2 Answer variants: 5
QUESTION N 558. Match properties to peptides
Question type: 4. Arranging the correspondences for the answers
Question value (points): 1
Answer variants:
1. Val-Phe-Ala-Gly
3. Phe-Leu-Arg-His
4. Gln-Gly-Asp-Ala
Answer variants: 3
Correspondence variants:
A. At pH=7.0 remains at the start in an electric field
B. Isoelectric point less then 7.0
C. At pH=7.0 it moves to the catode in an electric field
Correct answers: 1-A; 3-C; 4-B Correspondence variants: 3
QUESTION N 559. Match the correspondense
Question type: 4. Arranging the correspondences for the answers
Question value (points): 1
Answer variants:
1. Tyr-Phe-Glu-Ala-Asp
2. Arg-Thr-Val-Lys-Try
3. Both
4. None of them
Answer variants: 4
Correspondence variants:
A. It is soluble at pH=7.0
B. It's isoelectric point higher than 7.0
C. At pH=7.0 it can interact with Ca2+
D. Its isoelectric point is 7.0
Correct answers: 1-C; 2-B; 3-A; 4-D Correspondence variants: 4
QUESTION N 560. CHOOSE THE CORRECT ANSWER. Quaternary structure is defined as:
Question type: 1. Choosing the only correct answer
Question value (points): 1
Answer variants:
1. The ordered organization of secondary structure within a protein
2. The overall ordered array of amino acids within a protein
3. The structure obtained through interactions between different protomers
4. The structure obtained through interactions of secobdary structures within the protein
Correct answer: 3 Answer variants: 4
QUESTION N 561. CHOOSE THE CORRECT ANSWER. Increased affinity of each Hb protomer for O2 is due to:
Question type: 1. Choosing the only correct answer
Question value (points): 1
Answer variants:
1. Changes in tertiary structure of protomers
2. Changes in the bonds stabilizing the quaternary structure
3. Changes in relative position of the protomers
4. Cooperative changes in the conformation of the protomers
5. Changes in the localisation of an iron atom in the heme
Correct answer: 4 Answer variants: 5
QUESTION N 563. 2,3-bisphosphoglicerate regulates the fuction of Hb because (2):
Question type: 2. Choosing several possible correct answers
Question value (points): 1
Answer variants:
1. It interacts with Hb in the oxigen binding site
2. It interacts with Hb in allosteric site which doesn`t contain heme
3. It induces dissociation of the protomers
4. It changes the conformation of all four protomers
Correct answers: 2; 4 Answer variants: 4
QUESTION N 564. In what direction does this reaction proceed: 4O2 + Hb-BPG (left)<--->(right) Hb(O2)4 + BPG
Question type: 4. Arranging the correspondences for the answers
Question value (points): 1
Answer variants:
1. left
2. right
Answer variants: 2
Correspondence variants:
A. In tissue capillaries
B. In lung capillaries
Correct answers: 1-A; 2-B Correspondence variants: 2
QUESTION N 565. Where is the concentration of the Hb-BPG increased:
Question type: 1. Choosing the only correct answer
Question value (points): 1
Answer variants:
1. In resting muscule capillaries
2. In contracting muscle capillaries
Correct answer: 2 Answer variants: 2
QUESTION N 566. The affinity of Hb for O2 after adaptation of a man to high mountain condition
Question type: 1. Choosing the only correct answer
Question value (points): 1
Answer variants:
1. Increased
2. Decreased
Correct answer: 1 Answer variants: 2
QUESTION N 567. Whith the exception of glycine? all amino acids found in proteins are...
Question type: 1. Choosing the only correct answer
Question value (points): 1
Answer variants:
1. dextrorotatory
2. of the D configuration
3. optically inactive
4. of the L configuration
5. either L or D
Correct answer: 4 Answer variants: 5
QUESTION N 568. Which one of the following electrophoresis analitic procedures depends least on the
charge of the protein
Question type: 1. Choosing the only correct answer
Question value (points): 1
Answer variants:
1. Free zone capillary electrophoresis
2. Gel electophoresis
3. Polyacrylamide gel electrophoresis in sodium dodecyl sulfate
4. Isoelectric focusing
Correct answer: 3 Answer variants: 4
QUESTION N 569. Which one of the following statements about protein structure is correct?
Question type: 1. Choosing the only correct answer
Question value (points): 1
Answer variants:
1. The extended beta-configuration is not found in globular proteins
2. The stability of the alfa-helix is mainly due to hydrophobic interactions
3. Globular proteins tend to fold into configuration that keep hydrophobic side chains in the interior of the molecule
4. The protomers of polymeric proteins are linked by covalent bonds
5. The primery structure of a peptude does not influence the formation of the native three-dimentional configuration
Correct answer: 3 Answer variants: 5
QUESTION N 570. Compositional analyses from a large diversity of proteinsbfrom human reveals
Question type: 1. Choosing the only correct answer
Question value (points): 1
Answer variants:
1. that they are ALL made of ONLY 20 amino acids
2. the presence of a few D-amino acids
3. that none of the amino acids contains phosphate
4. that some of the glutamates from blood proteins have an additional carboxyl group
Correct answer: 4 Answer variants: 4
QUESTION N 571. Which interaction may play a role in the formation of quaternary structure by COVALENTLY
linking different polypeptide chains
Question type: 1. Choosing the only correct answer
Question value (points): 1
Answer variants:
1. Hydrogen bonds
2. Electrostatic interactions
3. Disulfide bridges
4. Peptide bonds
5. Hydrophobic interactions
Correct answer: 3 Answer variants: 5
QUESTION N 572. Select three amino acids that are hydrophobic and have a tendency to be located away
from the surface of proteins
Question type: 2. Choosing several possible correct answers
Question value (points): 1
Answer variants:
1. Leucine
2. Aspartate
3. Isoleucine
4. Cysteine
5. Proline
6. Arginine
7. Glutamate
8. Lysine
9. Valine
Correct answers: 1; 3; 9Answer variants: 9
QUESTION N 573. Select two amino acids that are basic and have a tendency to be found on the surface of
proteins
Question type: 2. Choosing several possible correct answers
Question value (points): 1
Answer variants:
1. Leucine
2. Aspartate
3. Isoleucine
4. Cysteine
5. Proline
6. Arginine
7. Glutamate
8. Lysine
9. Valine
Correct answers: 6; 8 Answer variants: 9
QUESTION N 575. Which one of the following characteristics best applies to an allosteric effector?
Question type: 1. Choosing the only correct answer
Question value (points): 1
Answer variants:
1. It competes with sustrate for the catalytic site
2. It binds to a site on the enzime molecule distinct from the catalytic site
3. It changes the nature of product formed
4. It covalently modifies the enzyme
Correct answer: 2 Answer variants: 4
QUESTION N 576. Why is ethanol an effective treatment for ethylene glycol poisoning?
Question type: 1. Choosing the only correct answer
Question value (points): 1
Answer variants:
1. ADH exibits a much higher affinity (Km) for ethanol then for ethylene glycol
2. Ethanol is an allosteric effector of ADH
3. Ethanol combines with the toxic compound formed by the reaction of ADH with ethylene glycol and randers it
harmless
4. Acedaldehyde is of therapeutic value
5. Ethanol induces another enzyme that is capable of metabolizing ethylene glycol
Correct answer: 1 Answer variants: 5
QUESTION N 577. If one compares Lineweaver-Burk plots for the reaction of ADH with ethanol and ethylene
glycol, which of the folloving would be observed?
Question type: 1. Choosing the only correct answer
Question value (points): 1
Answer variants:
1. They exhibit identical slopes
2. They exhibit identical Y-intercepts
3. They exhibit identical X-intercepts
4. Only the plot for the reaction of ethanol is linear
5. Only the plot for the reaction of ethylene glycol is linear
Correct answer: 2 Answer variants: 5
QUESTION N 578. The active site of an enzyme:
Question type: 1. Choosing the only correct answer
Question value (points): 1
Answer variants:
1. ... is formed only after addition of a specific substrate
2. ... is directly involved in binding of allosteric inhibitors
3. ... resides in a few adjacent amino acid residues in a primary sequence of a polypeptide chain
4. ... binds competitive inhibitors
Correct answer: 4 Answer variants: 4
QUESTION N 579. The Km for the enzyme is ...
Question type: 1. Choosing the only correct answer
Question value (points): 1
Answer variants:
1. 2 mM
2. 4 mM
3. 9 mM
4. 12 mM
5. 25 mM
Correct answer: 4 Answer variants: 5
QUESTION N 580. The velocity (V) of an enzyme-catalized reaction
Question type: 1. Choosing the only correct answer
Question value (points): 1
Answer variants:
1. ... decreases as the substrate concentration increases
2. ... is lowest when the enzyme is saturated with substrate
3. ... is related to the substrate concentrstion at 1/2 Vmax
4. ... is independent of the pH of the solution
Correct answer: 3 Answer variants: 4
QUESTION N 581. The concentration of fumarate required to give a velocity of 25 mcmol/min/mg protein is:
Question type: 1. Choosing the only correct answer
Question value (points): 1
Answer variants:
1. 2 mcM
2. 5 mcM
3. 10 micromol/l
4. 20 micromol/l
5. 50 micromol/l
Correct answer: 2 Answer variants: 5
QUESTION N 582. The fumarate concentration in mitochondria is approximately 2 mM. If the fumarate
concrntration dropped to 1 mM, the reaction rate would:
Question type: 1. Choosing the only correct answer
Question value (points): 1
Answer variants:
1. ... increase slightly
2. ... derease slightly
3. ... decrease in one half
4. ... stay exactly the same
Correct answer: 2 Answer variants: 4
QUESTION N 583. If a person who is fasting eats a high-carbohydrate meal, the velocity of glucokinase
reaction will:
Question type: 1. Choosing the only correct answer
Question value (points): 1
Answer variants:
1. ... remains at less then 50% Vmax
2. ... remains above 90% Vmax
3. ... increase from less than 50% Vmax to more than 50% Vmax
4. ... decrease from more than 50% Vmax to les than 50% Vmax
Correct answer: 3 Answer variants: 4
QUESTION N 584. A drug that is a competitive inhibitor of an enzyme
Question type: 1. Choosing the only correct answer
Question value (points): 1
Answer variants:
1. ... increases the apparent Km but does not affect Vmax
2. ... decreases the apparent Km but does not affect Vmax
3. ... increases Vmax but does not affect the apparent Km
4. ... decreases Vmax but does not affect the apparent Km
5. ... decreases both Vmax and Km
Correct answer: 1 Answer variants: 5
QUESTION N 585. The value of Km for the enzyme depicted by curve A is
Question type: 1. Choosing the only correct answer
Question value (points): 1
Answer variants:
1. 0.5 mM
2. 1 mM
3. 2 mM
Correct answer: 1 Answer variants: 3
QUESTION N 586. The value of Vmax for the enzyme depicted by curve A is
Question type: 1. Choosing the only correct answer
Question value (points): 1
Answer variants:
1. 0.1 mcmol/min/mg
2. 1 mcmol/min/mg
3. 10 mcmol/min/mg
4. 0.5 mmol/min/mg
5. 2 mmol/min/mg
Correct answer: 3 Answer variants: 5
QUESTION N 587. Curve B depicts the effect of an inhibitor on the system described by curve A. This inhibitor
...
Question type: 1. Choosing the only correct answer
Question value (points): 1
Answer variants:
1. ... is a competitive inhibitor
2. ... is a noncompetitive inhibitor
3. ... increases the Vmax
4. ... decreases the Km
Correct answer: 1 Answer variants: 4
QUESTION N 588. Curve C depicts the effect of an inhibitor on the system described by curve A. This inhibitor
...
Question type: 1. Choosing the only correct answer
Question value (points): 1
Answer variants:
1. ... is a competitive inhibitor
2. ... is a noncompetitive inhibitor
3. ... increases the Vmax
4. ... decreases the Km
Correct answer: 2 Answer variants: 4
QUESTION N 589. In the presence of the allosteric compound...
Question type: 1. Choosing the only correct answer
Question value (points): 1
Answer variants:
1. ... Km and Vmax both increase
2. ... Km and Vmax both decrease
3. ... Km increases and Vmax decreases
4. ... Km decreases and Vmax increases
5. ... Km decreases and Vmax stays the same
Correct answer: 5 Answer variants: 5
QUESTION N 590. The allosteric compound is ...
Question type: 1. Choosing the only correct answer
Question value (points): 1
Answer variants:
1. ... a competitive inhibitor
2. ... a noncompetitive inhibitor
3. ... an irreversible inhibitor
4. ... an activator
Correct answer: 4 Answer variants: 4
Questions: 577
The total of topics: 1

You might also like